Constant Deny syslog Cisco ASA

Did you know AVG CloudCare has Content Filtering? Content Filtering allows for increased productivity with advanced, real-time content filtering and the ability to block access to the websites that drain time and money from your business. 
Learn more about how Content Filtering can help you here: http://bit.ly/1Lrvqre

I have the following Query in Access and as it is, it works. I needed to add a new modifier to the "WHERE" clause of "Payroll.PayCode '37' so that it is not taking that paycode into account. Once I added that modifier, the entire query quit working stating that the query was to complex. I proceeded to struggle with the query for a while and eventually broke down and completely recreated the query in MS SQL where I have it working with the Modifier. Once I moved it to Access 2013 I get the same error that the Query is to complex. Could someone please help me out with this. Here is my SQL Query.SQLSELECT p.EmployeeNumber, e.Fname, e.Lname, e.Address, e.City, e.ST, e.Zip, e.DOH, e.DOT, e.DOB, e.[Mrtl Status],e.jobTitle, e.Dept, ROUND(SUM((p.PayUnit)/13),0) AS [13 Week Avg], eb.Eligible, eb.[Agreement Received], eb.[Benefits Term Date],...

Similar Messages

  • Cisco ASA Connection Denied syslog messages

    Hi,
    Could you please provide the connection denied syslog messages, I'm not able to differentiate the messages from syslog guide
    Regards,
    Shalendra

    Hi Shalendra,
    For TCP connection denied syslog , 106001 is the id.
    For protocol denied connection, 106002 is the id.
    For connection denies due to logging permit-hostdown policy, 414006 is the id.
    Refer to this link:
    http://www9.cisco.com/c/en/us/td/docs/security/asa/syslog-guide/syslogs/logsevp.html#13063
    Regards,
    Shrinkhala

  • Block / Deny ICMP Traffic cisco asa 5512-x

    hi expert
    I have cisco asa 5512x for configure as firewall and sslvpn.
    my customer want block/Deny icmp traffic from interface outside without block anything.
    i've configure form cli :
    icmp deny any outside
    but from outside can't open sslvpn url and asdm.

    Hi,
    Access for the Anyconnect/ASDM does not depend on the ICMP permit/deny commands on the ASA device.
    If you want to block the Pings to the ASA interface use the command:-
    icmp deny any outside etc.
    What do you mean by "i can ping from outside." Plzz explain.
    Thanks and Regards,
    Vibhor Amrodia

  • Cisco asa 5585 syslog options for ips?

    We have CISCO ASA 5585 with a separate module for IPS, I want to know what are the options for configuring syslog? Its nearly impossible to find ; and there are some forums on the internet which says that cisco ips stores logs in native / proprietary format and cannot be exported.
    Please elaborate
    Thanks.

    Some sensor-related events generate syslog messages. Those will be forwarded according to the parent ASA syslog settings.
    Detailed IPS events (signature triggers actions etc.) are stored locally and must be retrieved using the SDEE protocol (tcp-based). That requires use of a management system like Cisco Security Manager (CSM), IPS Manager Express (IME) etc. There is a good document here that explains SDEE in more detail.

  • Cisco ASA 5500x with FirePower logging & syslog Format/reference

    Hello everyone,
    Can anyone explain how Cisco ASA 5500x Firepower logging works?
    http://www.cisco.com/c/en/us/products/collateral/security/asa-5500-series-next-generation-firewalls/white_paper_c11-532091.html
    http://www.cisco.com/c/dam/en/us/solutions/collateral/enterprise/design-zone-smart-business-architecture/sbaSIEM_deployG.pdf
    I referred above links and found syslog for botnet filtering.
    ASA-4-338002: Dynamic filter permitted black listed TCP traffic from inside: 10.1.1.45/6798 (209.165.201.1/7890) to outside: 209.165.202.129/80 (209.165.202.129/80), destination 209.165.202.129 resolved from dynamic list: bad.example.com
    It is cisco asa 5500 log. is it same for Firepower? If yes, is Firepower generate syslog for all events like this?
    Please refer me syslog reference guide for Cisco ASA 5500x Firepower if exist.
    Thanks & Regards
    Revathi

    Firepower logging is to a Firesight management center (FMC) via https. It does not use SDEE.
    Just like the old IPS, syslog messages are only about the module status, not about actual IPS events.

  • Cisco ASA 5585-X SSP-20 8.4(2) - TCP Syslog problem

    Hi,
    We have a firewall service environment where logging is handled with UDP at the moment.
    Recently we have noticed that some messages get lost on the way to the server (Since the server doesnt seem to be under huge stress from syslog traffic). We decided to try sending the syslog via TCP.
    You can imagine my surprise when I enabled the "logging host <interface name> <server ip> tcp/1470" on an ASA Security context and find out that all the connections through that firewall are now being blocked. Granted, I could have checked the command reference for this specific command but I never even thought of the possibility of a logging command beeing able to stop all traffic on a firewall.
    The TCP syslog connection failing was caused by a missmatched TCP port on the server which got corrected quickly. Even though I could now view log messages from the firewall in question in real time, the only message logged was the blocking of new connections with the following syslog message:
    "%ASA-3-201008: Disallowing new connections."
    Here start my questions:
    - New connections are supposed to be blocked when the the TCP Syslog server aint reachable. How is it possible that I am seeing the TCP syslog sent to the server and the ASA Security Context is still blocking the traffic?
    - I configured the "logging permit-hostdown" after I found the command and it supposedly should prevent the above problem/situation from happening. Yet after issuing this command on the Security Context in question, connections were still being blocked with the same syslog message. Why is this?
    - Eventually I changed the logging back to UDP. This yet again caused no change to the situation. All the customer connections were still being blocked. Why is this?
    - After all the above I removed all possible logging configurations from the Security Context. This had absolutely no effect on the situation either.
    - As a last measure I changed to the system context of the ASA and totally removed the syslog interface from the Security Context. This also had absolutely no effect on the situation.
    At the end I was forced to save the configuration on the ASAs Flash -memory, remove the Security Context, create the SC again, attach the interfaces again and load the configuration from the flash into the Security Context. This in the end corrected the problem.
    Seems to me this is some sort of bug since the syslog server was receiving the syslog messages from the SC but the ASA was still blocking all new connections. Even the command "logging permit-hostdown" command didnt help or changing back to UDP.
    It seems the Security Context in question just simply got stuck and continued blocking all connections even though in the end it didnt have ANY logging configurations on.
    Seems to me that this is quite a risky configuration if you are possibly facing cutting all traffic for hundreds of customers when the syslog connection is lost or the above situation happens and isnt corrected by any of the above measures we took (like the command "logging permit-hostdown" which is supposed to avoid this situation alltogether).
    - Jouni

    Hi,
    I FINALLY had the time to look at this issue as I was testing something else in our lab too.
    In short, here is what I did:
    I configured the TCP logging in the same way as in the original post
    I configured the TCP logging giving the commands in different order
    Did some other tests related to the proble
    Device used: ASA 5585-X
    Software: 8.4(2)
    Original Device and software : ASA 5585-X running 8.4(1)9
    Heres the above scenarions and what actually happened
    Original situation
    Before doing any changes the test firewall context in question is working normally and the log sent by UDP/514 is arriving to the Syslog server as usual.
    I now change the syslog to TCP by giving a command "logging host tcp/1471" (actual port being TCP/1470)
    The firewall immediatly starts blocking all connections going through it.
    I change the configuration to the correct port TCP/1470 after which log starts appearing in my realtime view on the syslog server. The firewall context in question is still sending only the message "Disallowing new connections" even though the TCP -port on the Syslog server is clearly reachable and the connection is active.
    After this I try to do the suggest "clear local-host all" command. This has no effect on the firewall context. No connections are getting through. No connections/xlates are formed on the firewall. I can only see the firewall doing DNS queries with its outside interface (related to another configuration).
    After this I try to start correcting the situation the same way as before. I add "logging permit-hostdown" command which has no effect on the situation. I remove all logging configurations and it doesnt have any effect on the situation.
    After this I activate UDP logging and can see the logs arriving on the syslog server but again I can only see "Disallowing new connections" message.
    In the end I have no other option (to my knowledge) other than to delete the Security Context and create it again with same interfaces and with the configuration saved to the Flash -memory of the ASA.
    After this the connections work like usual. (UDP logging in the saved configuration)
    Giving the configurations in different order
    After I've created the firewall again and all is working I have another try in configuring the TCP Syslog while giving the commands in different order.
    First I add the command "logging permit-hostdown" command
    Then I add the command "logging host tcp/1470"
    After this logs start arriving on the syslog server and connections work as usual. Seems giving the "logging permit-hostdown" first before any other configurations is the right way to go.
    Removing the "logging permit-hostdown" command
    After I saw that everything was working I tried to remove the "logging permit-hostdown" command and see what happens. Everything worked fine.
    Configuring wrong TCP port to "logging host" command
    I decide to try and change the TCP port used to a wrong one and see if anything happens. (logging permit-hostdown is active). Firewall works as usual. Naturally no logs can be viewed at the syslog server.
    Configuring the TCP Syslogging without "logging permit-hostdown" but with correct port
    Finally I tried to configure the TCP Syslogging on ASA with the correct TCP port without issuing the "logging permit-hostdown" command. Everything seemed to work fine after this.
    So in conclusion it seems that IF you don't have the "logging permit-hostdown" command issued before you start configuring "logging host tcp/xxxx" , you might run into problems IF you don't have matching settings on the ASA sending the log and the Syslog server receiving the log.
    There doesnt seem to be any easy way to correct the situation (with the connections getting blocked) after you have once messed up the configurations. Seems your only option is to reconfigure the Security Context (which is easy) or if this problem exists in the same way in a single ASA you will have to reboot the device which means longer downtime than reconfiguring a context.
    There would still be a couple of things to test but at the moment I have no more time for this. I will update if there is any new information.
    - Jouni

  • Cisco ASA 5505 - IPsec Tunnel issue

    Issue with IPsec Child SA
    Hi,
    I have a site to site VPN tunnel setup with a Cisco ASA5505 and a Checkpoint Firewall. The version of software is 9.22. I am using IKEv2 for Phase 1 encryption. The following is my cisco asa configuration:
    hostname GARPR-COM1-WF01
    xlate per-session deny tcp any4 any4
    xlate per-session deny tcp any4 any6
    xlate per-session deny tcp any6 any4
    xlate per-session deny tcp any6 any6
    xlate per-session deny udp any4 any4 eq domain
    xlate per-session deny udp any4 any6 eq domain
    xlate per-session deny udp any6 any4 eq domain
    xlate per-session deny udp any6 any6 eq domain
    names
    interface Ethernet0/0
     description Failover Link
     switchport access vlan 950
    interface Ethernet0/1
     description Outside FW Link
     switchport access vlan 999
    interface Ethernet0/2
     description Inside FW Link
     switchport access vlan 998
    interface Ethernet0/3
     description Management Link
     switchport access vlan 6
    interface Ethernet0/4
     shutdown
    interface Ethernet0/5
     shutdown
    interface Ethernet0/6
     shutdown
    interface Ethernet0/7
     shutdown
    interface Vlan1
     no nameif
     no security-level
     no ip address
    interface Vlan6
     nameif management
     security-level 100
     ip address 10.65.1.20 255.255.255.240
    interface Vlan950
     description LAN Failover Interface
    interface Vlan998
     nameif inside
     security-level 100
     ip address 10.65.1.5 255.255.255.252
    interface Vlan999
     nameif outside
     security-level 0
     ip address ************* 255.255.255.248
    boot system disk0:/asa922-4-k8.bin
    ftp mode passive
    dns server-group DefaultDNS
     domain-name ***************
    object network North_American_LAN
     subnet 10.73.0.0 255.255.0.0
     description North American LAN
    object network Queretaro_LAN
     subnet 10.74.0.0 255.255.0.0
     description Queretaro_LAN
    object network Tor_LAN
     subnet 10.75.0.0 255.255.0.0
     description Tor LAN
    object network Mor_LAN
     subnet 10.76.0.0 255.255.0.0
     description Mor LAN
    object network Tus_LAN
     subnet 10.79.128.0 255.255.128.0
     description North American LAN
    object network Mtl_LAN
     subnet 10.88.0.0 255.255.0.0
     description Mtl LAN
    object network Wic_LAN
     subnet 10.90.0.0 255.254.0.0
     description Wic LAN
    object network Wic_LAN_172
     subnet 172.18.0.0 255.255.0.0
     description Wic Servers/Legacy Client LAN
    object network Mtl_LAN_172
     subnet 172.19.0.0 255.255.0.0
     description Mtl Servers/Legacy Client LAN
    object network Tor_LAN_172
     subnet 172.20.0.0 255.255.0.0
     description Tor Servers/Legacy Client LAN
    object network Bridge_LAN_172
     subnet 172.23.0.0 255.255.0.0
     description Bridge Servers/Legacy Client LAN
    object network Mtl_WLAN
     subnet 10.114.0.0 255.255.0.0
     description Mtl Wireless LAN
    object network Bel_WLAN
     subnet 10.115.0.0 255.255.0.0
     description Bel Wireless LAN
    object network Wic_WLAN
     subnet 10.116.0.0 255.255.0.0
     description Wic Wireless LAN
    object network Mtl_Infrastructure_10
     subnet 10.96.0.0 255.255.0.0
     description Mtl Infrastructre LAN
    object network BA_Small_Site_Blocks
     subnet 10.68.0.0 255.255.0.0
     description BA Small Sites Blocks
    object network Bel_LAN
     subnet 10.92.0.0 255.255.0.0
     description Bel LAN 10 Network
    object network LAN_172
     subnet 172.25.0.0 255.255.0.0
     description  LAN 172 Network
    object network Gar_LAN
     subnet 10.65.1.0 255.255.255.0
     description Gar LAN
    object network garpr-com1-wf01.net.aero.bombardier.net
     host **************
     description Garching Firewall
    object-group network BA_Sites
     description Internal Networks
     network-object object BA_Small_Site_Blocks
     network-object object Bel_LAN
     network-object object Bel_LAN_172
     network-object object Bel_WLAN
     network-object object Bridge_LAN_172
     network-object object Mtl_Infrastructure_10
     network-object object Mtl_LAN
     network-object object Mtl_LAN_172
     network-object object Mtl_WLAN
     network-object object Mor_LAN
     network-object object North_American_LAN
     network-object object Queretaro_LAN
     network-object object Tor_LAN
     network-object object Tor_LAN_172
     network-object object Tus_LAN
     network-object object Wic_LAN
     network-object object Wic_LAN_172
     network-object object Wic_WLAN
    access-list 101 extended permit ip object garpr-com1-wf01.net.aero.bombardier.net object Bel_LAN_172
    access-list 101 extended permit ip object Garching_LAN object-group BA_Sites
    pager lines 24
    logging enable
    logging timestamp
    logging buffered warnings
    logging trap informational
    logging asdm informational
    logging host outside 172.25.5.102
    mtu management 1500
    mtu inside 1500
    mtu outside 1500
    failover
    failover lan unit primary
    failover lan interface Failover_Link Vlan950
    failover polltime interface msec 500 holdtime 5
    failover key *****
    failover interface ip Failover_Link 192.168.124.1 255.255.255.0 standby 192.168.124.2
    icmp unreachable rate-limit 1 burst-size 1
    asdm image disk0:/asdm-731-101.bin
    asdm history enable
    arp timeout 14400
    no arp permit-nonconnected
    nat (inside,outside) source static Gar_LAN Gar_LAN destination static BA_Sites BA_Sites no-proxy-arp route-lookup
    route outside 0.0.0.0 0.0.0.0 ************* 1
    route inside 10.65.1.0 255.255.255.255 10.65.1.6 1
    route inside 10.65.1.16 255.255.255.240 10.65.1.6 1
    route inside 10.65.1.32 255.255.255.240 10.65.1.6 1
    route inside 10.65.1.48 255.255.255.240 10.65.1.6 1
    route inside 10.65.1.64 255.255.255.240 10.65.1.6 1
    route inside 10.65.1.128 255.255.255.128 10.65.1.6 1
    timeout xlate 3:00:00
    timeout pat-xlate 0:00:30
    timeout conn 1:00:00 half-closed 0:10:00 udp 0:02:00 icmp 0:00:02
    timeout sunrpc 0:10:00 h323 0:05:00 h225 1:00:00 mgcp 0:05:00 mgcp-pat 0:05:00
    timeout sip 0:30:00 sip_media 0:02:00 sip-invite 0:03:00 sip-disconnect 0:02:00
    timeout sip-provisional-media 0:02:00 uauth 0:05:00 absolute
    timeout tcp-proxy-reassembly 0:01:00
    timeout floating-conn 0:00:00
    dynamic-access-policy-record DfltAccessPolicy
    aaa-server TACACS+ protocol tacacs+
    aaa-server RADIUS protocol radius
    user-identity default-domain LOCAL
    aaa authentication ssh console LOCAL
    aaa authentication telnet console LOCAL
    http server enable
    http 10.65.1.0 255.255.255.0 inside
    http 172.25.5.0 255.255.255.0 inside
    http 10.65.1.21 255.255.255.255 management
    snmp-server host inside 172.25.49.0 community ***** udp-port 161
    snmp-server host outside 172.25.49.0 community *****
    snmp-server host inside 172.25.5.101 community ***** udp-port 161
    snmp-server host outside 172.25.5.101 community *****
    snmp-server host inside 172.25.81.88 poll community *****
    snmp-server host outside 172.25.81.88 poll community *****
    snmp-server location:
    snmp-server contact
    snmp-server community *****
    snmp-server enable traps syslog
    crypto ipsec ikev2 ipsec-proposal aes256
     protocol esp encryption aes-256
     protocol esp integrity sha-1
    crypto ipsec security-association lifetime seconds 3600
    crypto ipsec security-association pmtu-aging infinite
    crypto map GARCH 10 match address 101
    crypto map GARCH 10 set pfs group19
    crypto map GARCH 10 set peer *******************
    crypto map GARCH 10 set ikev2 ipsec-proposal aes256
    crypto map GARCH 10 set security-association lifetime seconds 3600
    crypto map GARCH interface outside
    crypto ca trustpool policy
    no crypto isakmp nat-traversal
    crypto ikev2 policy 10
     encryption aes-256
     integrity sha256
     group 19
     prf sha256
     lifetime seconds 86400
    crypto ikev2 enable outside
    telnet 10.65.1.6 255.255.255.255 inside
    telnet timeout 5
    ssh stricthostkeycheck
    ssh 172.25.5.0 255.255.255.0 inside
    ssh 172.19.9.49 255.255.255.255 inside
    ssh 172.25.5.0 255.255.255.0 outside
    ssh 172.19.9.49 255.255.255.255 outside
    ssh timeout 30
    ssh version 2
    ssh key-exchange group dh-group1-sha1
    console timeout 30
    management-access inside
    dhcprelay server 172.25.81.1 outside
    dhcprelay server 172.25.49.1 outside
    dhcprelay enable inside
    dhcprelay timeout 60
    threat-detection basic-threat
    threat-detection statistics access-list
    no threat-detection statistics tcp-intercept
    ntp server 172.19.109.41
    ntp server 172.19.109.42
    ntp server 172.19.9.49 source outside
    tunnel-group ********* type ipsec-l2l
    tunnel-group ********* ipsec-attributes
     ikev2 remote-authentication pre-shared-key *****
     ikev2 local-authentication pre-shared-key *****
    class-map inspection_default
     match default-inspection-traffic
    policy-map type inspect dns preset_dns_map
     parameters
      message-length maximum client auto
      message-length maximum 512
    policy-map global_policy
     class inspection_default
      inspect dns preset_dns_map
      inspect ftp
      inspect h323 h225
      inspect h323 ras
      inspect ip-options
      inspect netbios
      inspect rsh
      inspect rtsp
      inspect skinny
      inspect esmtp
      inspect sqlnet
      inspect sunrpc
      inspect tftp
      inspect sip
      inspect xdmcp
    service-policy global_policy global
    prompt hostname context
    no call-home reporting anonymous
    call-home
     profile CiscoTAC-1
      no active
      destination address http https://tools.cisco.com/its/service/oddce/services/DDCEService
      destination address email [email protected]
      destination transport-method http
      subscribe-to-alert-group diagnostic
      subscribe-to-alert-group environment
      subscribe-to-alert-group inventory periodic monthly
      subscribe-to-alert-group configuration periodic monthly
      subscribe-to-alert-group telemetry periodic daily
    Cryptochecksum:25ad9bf6db66a31e840ad96f49cd7e37
    : end
    I believe when a VPN tunnel is setup there should be one Child sa per subnet. The internal network of 10.65.1.0/24 should be setup with a child sa to the networks that were specified above depending on if there is traffic destined for them. What I am seeing is multiple child sa setup for the same subnet like the example below:
    GARPR-COM1-WF01# sh crypto ikev2 sa | i 172.19
              remote selector 172.19.0.0/0 - 172.19.255.255/65535
              remote selector 172.19.0.0/0 - 172.19.255.255/65535
              remote selector 172.19.0.0/0 - 172.19.255.255/65535
              remote selector 172.19.0.0/0 - 172.19.255.255/65535
              remote selector 172.19.0.0/0 - 172.19.255.255/65535
              remote selector 172.19.0.0/0 - 172.19.255.255/65535
              remote selector 172.19.0.0/0 - 172.19.255.255/65535
              remote selector 172.19.0.0/0 - 172.19.255.255/65535
              remote selector 172.19.0.0/0 - 172.19.255.255/65535
              remote selector 172.19.0.0/0 - 172.19.255.255/65535
    where for destination network 10.92.0.0/16 there is only one child sa:
    GARPR-COM1-WF01# sh crypto ikev2 sa | i 10.92
              remote selector 10.92.0.0/0 - 10.92.255.255/6553
    Should this be the case or does anyone have any idea why there is multiple child sa setup for the same subnet?
    Thanks
    Jonathan

    Hi there,
    I had same issue with PIX 506E and it was not even a circuit issue and I got ride of it and problem got fixed with PIX515E
    I don't know, the device is too old to stay alive.
    thanks

  • S2S between Cisco ASA 5505 and Sonicwall TZ-170 but not able to ping across

    Hi,
    I am helping out a friend of mine with his Site-to-Site VPN between his companies Cisco ASA another company's SonicWall TZ-170.  I have checked the screenshots proivded by the other end and tried to match with ours.  The Tunnel shows but we are not able to Ping resources on the other end.  The other side insists that the problem is on our end but I am not sure where the issue resides.  Please take a look at our config and let me know if there is anything that I have missed.  I am pretty sure I didn't but extra eyes may be of need here.
    Our LAN is 10.200.x.x /16 and theirs is 192.168.9.0 /24
    ASA Version 8.2(2)
    terminal width 300
    hostname company-asa
    domain-name Company.com
    no names
    name 10.1.0.0 sacramento-network
    name 10.3.0.0 irvine-network
    name 10.2.0.0 portland-network
    name x.x.x.x MailLive
    name 192.168.9.0 revit-vpn-remote-subnet
    dns-guard
    interface Ethernet0/0
    nameif outside
    security-level 0
    ip address x.x.x.x 255.255.255.128
    interface Ethernet0/1
    nameif inside
    security-level 100
    ip address 10.200.200.1 255.255.0.0
    interface Ethernet0/2
    nameif dmz
    security-level 50
    ip address 172.22.22.1 255.255.255.0
    interface Ethernet0/3
    description Internal Wireless
    shutdown
    nameif Wireless
    security-level 100
    ip address 10.201.201.1 255.255.255.0
    interface Management0/0
    shutdown
    nameif management
    security-level 100
    no ip address
    management-only
    boot system disk0:/asa822-k8.bin
    ftp mode passive
    clock timezone PST -8
    clock summer-time PDT recurring
    dns domain-lookup outside
    dns server-group DefaultDNS
    domain-name company.com
    same-security-traffic permit inter-interface
    same-security-traffic permit intra-interface
    object-group network local_net_group
    network-object 10.1.0.0 255.255.0.0
    network-object 10.2.0.0 255.255.0.0
    network-object 10.200.0.0 255.255.0.0
    network-object 10.3.0.0 255.255.0.0
    network-object 10.4.0.0 255.255.0.0
    network-object 10.5.0.0 255.255.0.0
    network-object 10.6.0.0 255.255.0.0
    network-object 10.7.0.0 255.255.0.0
    network-object 192.168.200.0 255.255.255.0
    object-group network NACIO123
    network-object 1.1.1.1 255.255.255.224
    object-group service MAIL_HTTPS_BORDERWARE tcp
    port-object eq smtp
    port-object eq https
    port-object eq 10101
    object-group service SYSLOG_SNMP_NETFLOW udp
    port-object eq syslog
    port-object eq snmp
    port-object eq 2055
    object-group service HTTP_HTTPS tcp
    port-object eq www
    port-object eq https
    object-group network OUTSIDECO_SERVERS
    network-object host x.x.x.34
    network-object host x.x.x.201
    network-object host x.x.x.63
    object-group network NO-LOG
    network-object host 10.200.200.13
    network-object host 10.200.200.25
    network-object host 10.200.200.32
    object-group service iPhoneSync-Services-TCP tcp
    port-object eq 993
    port-object eq 990
    port-object eq 998
    port-object eq 5678
    port-object eq 5721
    port-object eq 26675
    object-group service termserv tcp
    description terminal services
    port-object eq 3389
    object-group protocol TCPUDP
    protocol-object udp
    protocol-object tcp
    object-group service DTI tcp
    description DCS CONTROL PROTOCOL
    port-object eq 3333
    object-group service H.245 tcp
    description h.245 signaling
    port-object range 1024 4999
    object-group service RAS udp
    port-object eq 1719
    port-object range 1718 1720
    object-group service XML tcp
    port-object range 3336 3341
    object-group service mpi tcp
    port-object eq 2010
    object-group service mvp_control tcp
    port-object eq 2946
    object-group service rpc tcp-udp
    port-object eq 1809
    object-group service tcp8080 tcp
    port-object eq 8080
    object-group service tcp8011 tcp
    port-object eq 8011
    object-group service rtp_rtcp_udp udp
    port-object range 1024 65535
    object-group service ecs_xml tcp-udp
    port-object eq 3271
    object-group service rtp20000 udp
    description 10000-65535
    port-object range 20000 25000
    port-object range 10000 65535
    object-group service tcp5222 tcp
    port-object range 5222 5269
    object-group service tcp7070 tcp
    port-object eq 7070
    object-group network videoco
    network-object host x.x.x.144
    network-object host x.x.x.145
    object-group service video tcp
    port-object range 1718 h323
    object-group service XML2 tcp-udp
    port-object range 3336 3345
    object-group service tcp_tls tcp
    port-object eq 5061
    object-group service Autodesk tcp
    port-object eq 2080
    port-object range 27000 27009
    access-list outside_policy remark ====== Begin Mail From Postini Network ======
    access-list outside_policy extended permit tcp x.x.x.x 255.255.240.0 host x.x.x.x eq smtp
    access-list outside_policy extended permit tcp x.x.x.x 255.255.255.240 host x.x.x.x eq smtp
    access-list outside_policy extended permit tcp x.x.x.0 255.255.240.0 host x.x.x.x eq smtp
    access-list outside_policy remark ****** End Mail From Postini Network ******
    access-list outside_policy remark ====== Begin Inbound Web Mail Access ======
    access-list outside_policy extended permit tcp any host x.x.x.x object-group HTTP_HTTPS
    access-list outside_policy remark ****** End Inbound Web Mail Access ******
    access-list outside_policy remark ====== Begin iPhone Sync Rules to Mail Server ======
    access-list outside_policy extended permit tcp any host x.x.x.x object-group iPhoneSync-Services-TCP
    access-list outside_policy remark ****** End iPhone Sync Rules to Mail Server ******
    access-list outside_policy remark ====== Begin MARS Monitoring ======
    access-list outside_policy extended permit udp x.x.x.x 255.255.255.128 host x.x.x.x object-group SYSLOG_SNMP_NETFLOW
    access-list outside_policy extended permit icmp x.x.x.x 255.255.255.128 host x.x.x.x
    access-list outside_policy remark ****** End MARS Monitoring ******
    access-list outside_policy extended permit tcp object-group NACIO123 host x.x.x.141 eq ssh
    access-list outside_policy extended permit tcp any host x.x.x.x eq www
    access-list outside_policy extended permit tcp any host x.x.x.x eq https
    access-list outside_policy extended permit tcp any host x.x.x.x eq h323
    access-list outside_policy extended permit tcp any host x.x.x.x range 60000 60001
    access-list outside_policy extended permit udp any host x.x.x.x range 60000 60007
    access-list outside_policy remark radvision 5110   port 80 both
    access-list outside_policy extended permit object-group TCPUDP any object-group videoco eq www
    access-list outside_policy remark radvision
    access-list outside_policy extended permit tcp any object-group videoco object-group termserv
    access-list outside_policy remark radvision 5110  port21 out
    access-list outside_policy extended permit tcp any object-group videoco eq ftp
    access-list outside_policy remark rad5110   port22 both
    access-list outside_policy extended permit tcp any object-group videoco eq ssh
    access-list outside_policy remark rad 5110  port161 udp both
    access-list outside_policy extended permit udp any object-group videoco eq snmp
    access-list outside_policy remark rad5110 port443 both
    access-list outside_policy extended permit tcp any object-group videoco eq https
    access-list outside_policy remark rad5110 port 1024-4999  both
    access-list outside_policy extended permit tcp any object-group videoco object-group H.245
    access-list outside_policy remark rad5110 port 1719 udp both
    access-list outside_policy extended permit udp any object-group videoco object-group RAS
    access-list outside_policy remark rad5110 port 1720 both
    access-list outside_policy extended permit tcp any any eq h323
    access-list outside_policy remark RAD 5110 port 3333 tcp both
    access-list outside_policy extended permit tcp any object-group videoco object-group DTI
    access-list outside_policy remark rad5110 port 3336-3341 both
    access-list outside_policy extended permit object-group TCPUDP any object-group videoco object-group XML2
    access-list outside_policy remark port 5060 tcp/udp
    access-list outside_policy extended permit object-group TCPUDP any object-group videoco eq sip
    access-list outside_policy remark rad 5110port 1809 rpc both
    access-list outside_policy extended permit object-group TCPUDP any object-group videoco object-group rpc
    access-list outside_policy remark rad 5110 port 2010 both
    access-list outside_policy extended permit tcp any object-group videoco object-group mpi
    access-list outside_policy remark rad 5110 port 2946 both
    access-list outside_policy extended permit tcp any object-group videoco object-group mvp_control
    access-list outside_policy extended permit tcp any object-group videoco object-group tcp8080
    access-list outside_policy extended permit tcp any object-group videoco object-group tcp8011
    access-list outside_policy remark 1024-65535
    access-list outside_policy extended permit udp any object-group videoco object-group rtp_rtcp_udp
    access-list outside_policy extended permit object-group TCPUDP any object-group videoco object-group ecs_xml
    access-list outside_policy extended permit udp any object-group videoco object-group rtp20000
    access-list outside_policy extended permit tcp any object-group videoco eq telnet
    access-list outside_policy remark port 53 dns
    access-list outside_policy extended permit object-group TCPUDP any object-group videoco eq domain
    access-list outside_policy remark 7070
    access-list outside_policy extended permit tcp any object-group videoco object-group tcp7070
    access-list outside_policy remark 5222-5269 tcp
    access-list outside_policy extended permit tcp any object-group videoco range 5222 5269
    access-list outside_policy extended permit tcp any object-group videoco object-group video
    access-list outside_policy extended permit tcp any object-group videoco object-group tcp_tls
    access-list outside_policy remark ====== Begin Autodesk Activation access ======
    access-list outside_policy extended permit tcp any any object-group Autodesk
    access-list outside_policy remark ****** End Autodesk Activation access ******
    access-list outside_policy extended permit tcp x.x.x.x 255.255.255.248 host x.x.x.x eq smtp
    access-list outside_policy remark ****** End Autodesk Activation access ******
    access-list inside_policy extended deny tcp host 10.200.200.25 10.1.0.0 255.255.0.0 eq 2967 log disable
    access-list inside_policy extended deny tcp host 10.200.200.25 10.3.0.0 255.255.0.0 eq 2967 log disable
    access-list inside_policy extended deny tcp host 10.200.200.25 10.2.0.0 255.255.0.0 eq 2967 log disable
    access-list inside_policy extended deny tcp host 10.200.200.25 10.4.0.0 255.255.0.0 eq 2967 log disable
    access-list inside_policy extended deny tcp host 10.200.200.25 10.5.0.0 255.255.0.0 eq 2967 log disable
    access-list inside_policy extended deny udp object-group NO-LOG any eq 2967 log disable
    access-list inside_policy extended deny tcp object-group NO-LOG any eq 2967 log disable
    access-list inside_policy remark ====== Begin Outbound Mail Server Rules ======
    access-list inside_policy extended permit udp host 10.200.200.222 any eq 5679
    access-list inside_policy extended permit tcp host 10.200.200.222 any eq smtp
    access-list inside_policy remark ****** End Outbound Mail Server Rules ******
    access-list inside_policy extended permit ip object-group local_net_group any
    access-list inside_policy extended permit icmp object-group local_net_group any
    access-list OUTSIDECO_VPN extended permit ip host x.x.x.x object-group OUTSIDECO_SERVERS
    access-list company-split-tunnel standard permit 10.1.0.0 255.255.0.0
    access-list company-split-tunnel standard permit 10.2.0.0 255.255.0.0
    access-list company-split-tunnel standard permit 10.3.0.0 255.255.0.0
    access-list company-split-tunnel standard permit 10.4.0.0 255.255.0.0
    access-list company-split-tunnel standard permit 10.200.0.0 255.255.0.0
    access-list company-split-tunnel standard permit 10.5.0.0 255.255.0.0
    access-list company-split-tunnel standard permit 10.6.0.0 255.255.0.0
    access-list company-split-tunnel standard permit 10.7.0.0 255.255.0.0
    access-list company-split-tunnel standard permit 172.22.22.0 255.255.255.0
    access-list company-split-tunnel remark Video
    access-list company-split-tunnel standard permit 192.168.0.0 255.255.0.0
    access-list SSL_SPLIT standard permit 10.1.0.0 255.255.0.0
    access-list SSL_SPLIT standard permit 10.2.0.0 255.255.0.0
    access-list SSL_SPLIT standard permit 10.3.0.0 255.255.0.0
    access-list SSL_SPLIT standard permit 10.200.0.0 255.255.0.0
    access-list SSL_SPLIT standard permit 10.4.0.0 255.255.0.0
    access-list SSL_SPLIT standard permit 10.5.0.0 255.255.0.0
    access-list SSL_SPLIT standard permit 10.6.0.0 255.255.0.0
    access-list SSL_SPLIT standard permit 10.7.0.0 255.255.0.0
    access-list SSL_SPLIT standard permit 172.22.22.0 255.255.255.0
    access-list SSL_SPLIT remark Video
    access-list SSL_SPLIT standard permit 192.168.0.0 255.255.0.0
    access-list NONAT_SSL extended permit ip object-group local_net_group 172.20.20.0 255.255.255.0
    access-list NONAT_SSL extended permit ip 10.200.0.0 255.255.0.0 192.168.9.0 255.255.255.0
    access-list tom extended permit tcp host x.x.x.x any eq smtp
    access-list tom extended permit tcp host 10.200.200.222 any eq smtp
    access-list tom extended permit tcp any host x.x.x.x
    access-list aaron extended permit tcp any any eq 2967
    access-list capauth extended permit ip host 10.200.200.1 host 10.200.200.220
    access-list capauth extended permit ip host 10.200.200.220 host 10.200.200.1
    access-list DMZ extended permit icmp any any
    access-list dmz_access_in extended permit tcp any eq 51024 any eq 3336
    access-list dmz_access_in extended permit icmp any any
    access-list dmz_access_in extended permit tcp any any eq ftp
    access-list dmz_access_in extended permit tcp any any eq https
    access-list dmz_access_in remark rad5110 port 162 out
    access-list dmz_access_in extended permit udp any any eq snmptrap
    access-list dmz_access_in remark port 23 out
    access-list dmz_access_in extended permit tcp any any eq telnet
    access-list dmz_access_in remark port 53 dns out
    access-list dmz_access_in extended permit object-group TCPUDP any any eq domain
    access-list dmz_access_in extended permit object-group TCPUDP any any eq www
    access-list dmz_access_in extended permit tcp any any eq h323
    access-list dmz_access_in extended permit tcp any any object-group XML
    access-list dmz_access_in extended permit udp any any object-group RAS
    access-list dmz_access_in extended permit tcp any any range 1718 h323
    access-list dmz_access_in extended permit tcp any any object-group H.245
    access-list dmz_access_in extended permit object-group TCPUDP any any eq sip
    access-list dmz_access_in extended permit udp any any object-group rtp_rtcp_udp
    access-list dmz_access_in extended permit object-group TCPUDP any any object-group XML2
    access-list dmz_access_in extended permit ip object-group local_net_group any
    access-list dmz_access_in remark port 5061
    access-list dmz_access_in extended permit tcp any any object-group tcp_tls
    access-list outside_cryptomap extended permit ip 10.200.0.0 255.255.0.0 192.168.9.0 255.255.255.0
    pager lines 24
    logging enable
    logging buffered warnings
    logging trap informational
    logging history informational
    logging asdm warnings
    logging host outside x.x.x.x
    mtu outside 1500
    mtu inside 1500
    mtu dmz 1500
    mtu Wireless 1500
    mtu management 1500
    ip local pool SSL_VPN_POOL 172.20.20.1-172.20.20.75 mask 255.255.255.0
    ip verify reverse-path interface outside
    no failover
    icmp unreachable rate-limit 1 burst-size 1
    asdm image disk0:/asdm-631.bin
    no asdm history enable
    arp timeout 14400
    global (outside) 1 interface
    nat (inside) 0 access-list NONAT_SSL
    nat (inside) 1 0.0.0.0 0.0.0.0
    static (inside,outside) x.x.x.12 10.200.200.15 netmask 255.255.255.255
    static (inside,outside) x.x.x.15 10.5.0.11 netmask 255.255.255.255
    static (inside,outside) x.x.x.13 10.200.200.240 netmask 255.255.255.255
    static (inside,outside) x.x.x.16 10.200.200.222 netmask 255.255.255.255
    static (inside,outside) x.x.x.14 10.200.200.155 netmask 255.255.255.255
    static (inside,dmz) 10.200.200.0 10.200.200.0 netmask 255.255.255.0
    static (inside,dmz) 10.4.0.0 10.4.0.0 netmask 255.255.0.0
    static (dmz,outside) x.x.x.18 172.22.22.15 netmask 255.255.255.255
    static (dmz,outside) x.x.x.19 172.22.22.16 netmask 255.255.255.255
    static (inside,dmz) 10.3.0.0 10.3.0.0 netmask 255.255.0.0
    static (inside,dmz) 10.2.0.0 10.2.0.0 netmask 255.255.0.0
    static (inside,dmz) 10.1.0.0 10.1.0.0 netmask 255.255.0.0
    static (inside,dmz) 10.6.0.0 10.6.0.0 netmask 255.255.0.0
    static (inside,dmz) 10.7.0.0 10.7.0.0 netmask 255.255.0.0
    static (inside,dmz) 10.5.0.0 10.5.0.0 netmask 255.255.0.0
    access-group outside_policy in interface outside
    access-group inside_policy in interface inside
    access-group dmz_access_in in interface dmz
    route outside 0.0.0.0 0.0.0.0 x.x.x.12 1
    route inside 10.1.0.0 255.255.0.0 10.200.200.254 1
    route inside 10.2.0.0 255.255.0.0 10.200.200.254 1
    route inside 10.3.0.0 255.255.0.0 10.200.200.254 1
    route inside 10.4.0.0 255.255.0.0 10.200.200.254 1
    route inside 10.5.0.0 255.255.0.0 10.200.200.254 1
    route inside 10.6.0.0 255.255.0.0 10.200.200.254 1
    route inside 10.7.0.0 255.255.0.0 10.200.200.150 1
    route inside x.x.x.0 255.255.255.0 10.200.200.2 1
    route inside x.x.x.0 255.255.255.0 10.200.200.2 1
    route inside 192.168.1.0 255.255.255.0 10.200.200.254 1
    route inside 192.168.2.0 255.255.255.0 10.200.200.254 1
    route inside 192.168.3.0 255.255.255.0 10.200.200.254 1
    route inside 192.168.4.0 255.255.255.0 10.200.200.254 1
    route inside 192.168.5.0 255.255.255.0 10.200.200.254 1
    route inside 192.168.6.0 255.255.255.0 10.200.200.254 1
    route inside 192.168.7.0 255.255.255.0 10.200.200.254 1
    route inside 192.168.200.0 255.255.255.0 10.200.200.254 1
    route inside 192.168.201.0 255.255.255.0 10.200.200.254 1
    timeout xlate 3:00:00
    timeout conn 1:00:00 half-closed 0:10:00 udp 0:02:00 icmp 0:00:02
    timeout sunrpc 0:10:00 h323 2:00:00 h225 1:00:00 mgcp 0:05:00 mgcp-pat 0:05:00
    timeout sip 0:30:00 sip_media 0:02:00 sip-invite 0:03:00 sip-disconnect 0:02:00
    timeout sip-provisional-media 0:02:00 uauth 0:05:00 absolute
    timeout tcp-proxy-reassembly 0:01:00
    dynamic-access-policy-record DfltAccessPolicy
    aaa-server COMPANY-NT-AUTH protocol nt
    aaa-server COMPANY-NT-AUTH (inside) host 10.200.200.220
    nt-auth-domain-controller DC
    aaa authentication ssh console LOCAL
    aaa authorization command LOCAL
    http server enable
    http 10.200.200.0 255.255.255.0 inside
    http 10.200.0.0 255.255.0.0 inside
    http 10.3.0.0 255.255.0.0 inside
    no snmp-server location
    no snmp-server contact
    snmp-server community *****
    snmp-server enable traps snmp authentication linkup linkdown coldstart
    crypto ipsec transform-set AES256-SHA esp-aes-256 esp-sha-hmac
    crypto ipsec transform-set asa2transform esp-3des esp-sha-hmac
    crypto ipsec transform-set 3DES-SHA esp-3des esp-sha-hmac
    crypto ipsec transform-set ESP-3DES-SHA esp-3des esp-sha-hmac
    crypto ipsec security-association lifetime seconds 28800
    crypto ipsec security-association lifetime kilobytes 4608000
    crypto dynamic-map outside_dyn_map 20 set transform-set 3DES-SHA
    crypto dynamic-map outside_dyn_map 20 set security-association lifetime seconds 28800
    crypto dynamic-map outside_dyn_map 20 set security-association lifetime kilobytes 4608000
    crypto map OUTSIDE_MAP 5 match address outside_cryptomap
    crypto map OUTSIDE_MAP 5 set pfs
    crypto map OUTSIDE_MAP 5 set peer x.x.x.53
    crypto map OUTSIDE_MAP 5 set transform-set 3DES-SHA
    crypto map OUTSIDE_MAP 5 set security-association lifetime seconds 28800
    crypto map OUTSIDE_MAP 10 match address OUTSIDECO_VPN
    crypto map OUTSIDE_MAP 10 set peer x.x.x.25
    crypto map OUTSIDE_MAP 10 set transform-set AES256-SHA
    crypto map OUTSIDE_MAP 10 set security-association lifetime seconds 28800
    crypto map OUTSIDE_MAP 10 set security-association lifetime kilobytes 4608000
    crypto map OUTSIDE_MAP 65535 ipsec-isakmp dynamic outside_dyn_map
    crypto map OUTSIDE_MAP interface outside
    crypto isakmp identity address
    crypto isakmp enable outside
    crypto isakmp policy 5
    authentication pre-share
    encryption 3des
    hash sha
    group 2
    lifetime 28800
    crypto isakmp policy 10
    authentication pre-share
    encryption des
    hash sha
    group 2
    lifetime 86400
    telnet timeout 5
    ssh 0.0.0.0 0.0.0.0 outside
    ssh 0.0.0.0 0.0.0.0 inside
    ssh timeout 20
    console timeout 0
    dhcpd dns 10.200.200.220 10.200.200.225
    dhcpd wins 10.200.200.220 10.200.200.225
    dhcpd lease 18000
    dhcpd domain company.com
    dhcpd dns 10.200.200.220 10.200.200.225 interface Wireless
    dhcpd wins 10.200.200.220 10.200.200.225 interface Wireless
    dhcpd lease 18000 interface Wireless
    dhcpd domain company.com interface Wireless
    threat-detection basic-threat
    threat-detection statistics access-list
    no threat-detection statistics tcp-intercept
    ntp server 192.5.41.40 source outside prefer
    ssl trust-point vpn.company.com outside
    webvpn
    enable outside
    anyconnect-essentials
    svc image disk0:/anyconnect-win-2.5.0217-k9.pkg 1
    svc image disk0:/anyconnect-macosx-i386-2.5.2017-k9.pkg 2
    svc enable
    tunnel-group-list enable
    group-policy SSL_Client_Policy internal
    group-policy SSL_Client_Policy attributes
    wins-server value 10.200.200.220
    dns-server value 10.200.200.220
    vpn-tunnel-protocol IPSec svc webvpn
    split-tunnel-policy tunnelspecified
    split-tunnel-network-list value SSL_SPLIT
    default-domain value company.com
    webvpn
      sso-server none
      auto-signon allow uri * auth-type all
    group-policy no-split-test internal
    group-policy no-split-test attributes
    banner value Welcome to company and Associates
    banner value Welcome to company and Associates
    dns-server value 10.200.200.220
    vpn-tunnel-protocol IPSec
    ipsec-udp enable
    split-tunnel-policy tunnelall
    default-domain value company.com
    group-policy DfltGrpPolicy attributes
    dns-server value 10.200.200.220
    default-domain value company.com
    group-policy company internal
    group-policy company attributes
    banner value Welcome to company and Associates
    banner value Welcome to company and Associates
    dns-server value 10.200.200.220
    vpn-tunnel-protocol IPSec
    ipsec-udp enable
    split-tunnel-policy tunnelspecified
    split-tunnel-network-list value SSL_SPLIT
    default-domain value company.com
    username ciscoadmin password xxxxxxxxxxx encrypted privilege 15
    tunnel-group DefaultWEBVPNGroup general-attributes
    address-pool SSL_VPN_POOL
    authentication-server-group COMPANY-NT-AUTH
    default-group-policy SSL_Client_Policy
    tunnel-group DefaultWEBVPNGroup webvpn-attributes
    group-alias company_SSL_VPN enable
    tunnel-group company_group type remote-access
    tunnel-group company_group general-attributes
    address-pool SSL_VPN_POOL
    authentication-server-group COMPANY-NT-AUTH LOCAL
    default-group-policy company
    tunnel-group company_group ipsec-attributes
    pre-shared-key *****
    tunnel-group x.x.x.53 type ipsec-l2l
    tunnel-group x.x.x.53 ipsec-attributes
    pre-shared-key *****
    class-map inspection_default
    match default-inspection-traffic
    policy-map global_policy
    class inspection_default
      inspect tftp
      inspect esmtp
      inspect ftp
      inspect icmp
      inspect ip-options
      inspect netbios
      inspect rsh
      inspect skinny
      inspect sqlnet
      inspect sunrpc
      inspect xdmcp
      inspect mgcp
      inspect h323 h225
      inspect h323 ras
      inspect sip
    service-policy global_policy global
    privilege cmd level 5 mode exec command ping
    privilege cmd level 6 mode exec command write
    privilege show level 5 mode exec command running-config
    privilege show level 5 mode exec command version
    privilege show level 5 mode exec command conn
    privilege show level 5 mode exec command memory
    privilege show level 5 mode exec command cpu
    privilege show level 5 mode exec command xlate
    privilege show level 5 mode exec command traffic
    privilege show level 5 mode exec command interface
    privilege show level 5 mode exec command clock
    privilege show level 5 mode exec command ip
    privilege show level 5 mode exec command failover
    privilege show level 5 mode exec command arp
    privilege show level 5 mode exec command route
    privilege show level 5 mode exec command blocks
    prompt hostname context
    call-home
    profile CiscoTAC-1
      no active
      destination address http https://tools.cisco.com/its/service/oddce/services/DDCEService
      destination address email [email protected]
      destination transport-method http
      subscribe-to-alert-group diagnostic
      subscribe-to-alert-group environment
      subscribe-to-alert-group inventory periodic monthly
      subscribe-to-alert-group configuration periodic monthly
      subscribe-to-alert-group telemetry periodic daily
    Cryptochecksum:a0689b4c837c79a51e7a0cfed591dec9
    : end
    COMPANY-asa#

    Hi Sian,
    Yes on their end the PFS is enabled for DH Group 2.
    Here is the information that you requested:
    company-asa# sh crypto isakmp sa
       Active SA: 3
        Rekey SA: 0 (A tunnel will report 1 Active and 1 Rekey SA during rekey)
    Total IKE SA: 3
    1   IKE Peer: x.x.x.87
        Type    : user            Role    : responder
        Rekey   : no              State   : AM_ACTIVE
    2   IKE Peer: x.x.x.53
        Type    : L2L             Role    : initiator
        Rekey   : no              State   : MM_ACTIVE
    3   IKE Peer: x.x.x.25
        Type    : user            Role    : initiator
        Rekey   : no              State   : MM_WAIT_MSG4
    company-asa# sh crypto ipsec sa
    interface: outside
        Crypto map tag: OUTSIDE_MAP, seq num: 5, local addr: x.x.x.13
          access-list outside_cryptomap extended permit ip 10.200.0.0 255.255.0.0 192.168.9.0 255.255.255.0
          local ident (addr/mask/prot/port): (10.200.0.0/255.255.0.0/0/0)
          remote ident (addr/mask/prot/port): (192.168.9.0/255.255.255.0/0/0)
          current_peer: x.x.x.53
          #pkts encaps: 0, #pkts encrypt: 0, #pkts digest: 0
          #pkts decaps: 10744, #pkts decrypt: 10744, #pkts verify: 10744
          #pkts compressed: 0, #pkts decompressed: 0
          #pkts not compressed: 0, #pkts comp failed: 0, #pkts decomp failed: 0
          #pre-frag successes: 0, #pre-frag failures: 0, #fragments created: 0
          #PMTUs sent: 0, #PMTUs rcvd: 0, #decapsulated frgs needing reassembly: 0
          #send errors: 0, #recv errors: 0
          local crypto endpt.: x.x.x.13, remote crypto endpt.: x.x.x.53
          path mtu 1500, ipsec overhead 58, media mtu 1500
          current outbound spi: 500EC8BF
          current inbound spi : 8DAE3436
        inbound esp sas:
          spi: 0x8DAE3436 (2377004086)
             transform: esp-3des esp-sha-hmac no compression
             in use settings ={L2L, Tunnel, PFS Group 2, }
             slot: 0, conn_id: 32768, crypto-map: OUTSIDE_MAP
             sa timing: remaining key lifetime (kB/sec): (3914946/24388)
             IV size: 8 bytes
             replay detection support: Y
             Anti replay bitmap:
              0xFFFFFFFF 0xFFFFFFFF
        outbound esp sas:
          spi: 0x500EC8BF (1343146175)
             transform: esp-3des esp-sha-hmac no compression
             in use settings ={L2L, Tunnel, PFS Group 2, }
             slot: 0, conn_id: 32768, crypto-map: OUTSIDE_MAP
             sa timing: remaining key lifetime (kB/sec): (3915000/24388)
             IV size: 8 bytes
             replay detection support: Y
             Anti replay bitmap:
              0x00000000 0x00000001
        Crypto map tag: outside_dyn_map, seq num: 20, local addr: x.x.x.13
          local ident (addr/mask/prot/port): (0.0.0.0/0.0.0.0/0/0)
          remote ident (addr/mask/prot/port): (172.20.20.8/255.255.255.255/0/0)
          current_peer: x.x.x.87, username: ewebb
          dynamic allocated peer ip: 172.20.20.8
          #pkts encaps: 16434, #pkts encrypt: 16464, #pkts digest: 16464
          #pkts decaps: 19889, #pkts decrypt: 19889, #pkts verify: 19889
          #pkts compressed: 0, #pkts decompressed: 0
          #pkts not compressed: 16434, #pkts comp failed: 0, #pkts decomp failed: 0
          #pre-frag successes: 30, #pre-frag failures: 0, #fragments created: 60
          #PMTUs sent: 0, #PMTUs rcvd: 0, #decapsulated frgs needing reassembly: 60
          #send errors: 0, #recv errors: 0
          local crypto endpt.: x.x.x.13/4500, remote crypto endpt.: x.x.x.87/2252
          path mtu 1500, ipsec overhead 66, media mtu 1500
          current outbound spi: 2D712C9F
          current inbound spi : 0EDB79C8
        inbound esp sas:
          spi: 0x0EDB79C8 (249264584)
             transform: esp-3des esp-sha-hmac no compression
             in use settings ={RA, Tunnel,  NAT-T-Encaps, }
             slot: 0, conn_id: 65536, crypto-map: outside_dyn_map
             sa timing: remaining key lifetime (sec): 18262
             IV size: 8 bytes
             replay detection support: Y
             Anti replay bitmap:
              0xFFFFFFFF 0xFFFFFFFF
        outbound esp sas:
          spi: 0x2D712C9F (762391711)
             transform: esp-3des esp-sha-hmac no compression
             in use settings ={RA, Tunnel,  NAT-T-Encaps, }
             slot: 0, conn_id: 65536, crypto-map: outside_dyn_map
             sa timing: remaining key lifetime (sec): 18261
             IV size: 8 bytes
             replay detection support: Y
             Anti replay bitmap:
              0x00000000 0x00000001

  • Cisco ASA 5505 site to site Multiple subnet.

    Hi. I need some help configuring my cisco asa 5505.
    I've set up a VPN tunnel between two ASA 5505
    Site 1:
    Subnet 192.168.77.0
    Site 2:
    Have multiple vlans and now the tunnel goes to vlan400 - 192.168.1.0
    What I need help with:
    From site 1 i need to be able to reach another vlan on site 2. vlan480 - 192.168.20.0
    And from site 1 I need to reach 192.168.77.0 subnet from vlan480 - 192.168.20.0
    Vlan480 is used for phones. In vlan480 we have a PABX central.
    Is this possible to do?
    Any help would be greatfully appreciated!
    Config site 2:
    : Saved
    ASA Version 7.2(2)
    hostname ciscoasa
    domain-name default.domain.invalid
    enable password x encrypted
    names
    name 192.168.1.250 DomeneServer
    name 192.168.1.10 NotesServer
    name 192.168.1.90 OvServer
    name 192.168.1.97 TerminalServer
    name 192.168.1.98 w8-eyeshare
    name 192.168.50.10 w8-print
    name 192.168.1.94 w8-app
    name 192.168.1.89 FonnaFlyMedia
    interface Vlan1
    nameif Vlan1
    security-level 100
    ip address 192.168.200.100 255.255.255.0
    ospf cost 10
    interface Vlan2
    nameif outside
    security-level 0
    ip address 79.x.x.226 255.255.255.224
    ospf cost 10
    interface Vlan400
    nameif vlan400
    security-level 100
    ip address 192.168.1.1 255.255.255.0
    ospf cost 10
    interface Vlan450
    nameif Vlan450
    security-level 100
    ip address 192.168.210.1 255.255.255.0
    ospf cost 10
    interface Vlan460
    nameif Vlan460-SuldalHotell
    security-level 100
    ip address 192.168.2.1 255.255.255.0
    ospf cost 10
    interface Vlan461
    nameif Vlan461-SuldalHotellGjest
    security-level 100
    ip address 192.168.3.1 255.255.255.0
    ospf cost 10
    interface Vlan462
    nameif Vlan462-Suldalsposten
    security-level 100
    ip address 192.168.4.1 255.255.255.0
    ospf cost 10
    interface Vlan470
    nameif vlan470-Kyrkjekontoret
    security-level 100
    ip address 192.168.202.1 255.255.255.0
    ospf cost 10
    interface Vlan480
    nameif vlan480-Telefoni
    security-level 100
    ip address 192.168.20.1 255.255.255.0
    ospf cost 10
    interface Vlan490
    nameif Vlan490-QNapBackup
    security-level 100
    ip address 192.168.10.1 255.255.255.0
    ospf cost 10
    interface Vlan500
    nameif Vlan500-HellandBadlands
    security-level 100
    ip address 192.168.30.1 255.255.255.0
    ospf cost 10
    interface Vlan510
    nameif Vlan510-IsTak
    security-level 100
    ip address 192.168.40.1 255.255.255.0
    ospf cost 10
    interface Vlan600
    nameif Vlan600-SafeQ
    security-level 100
    ip address 192.168.50.1 255.255.255.0
    ospf cost 10
    interface Ethernet0/0
    switchport access vlan 2
    interface Ethernet0/1
    interface Ethernet0/2
    switchport access vlan 500
    switchport trunk allowed vlan 400,450,460-462,470,480,500,510,600,610
    switchport mode trunk
    interface Ethernet0/3
    switchport access vlan 490
    interface Ethernet0/4
    interface Ethernet0/5
    interface Ethernet0/6
    interface Ethernet0/7
    passwd x encrypted
    ftp mode passive
    clock timezone WAT 1
    dns server-group DefaultDNS
    domain-name default.domain.invalid
    same-security-traffic permit inter-interface
    same-security-traffic permit intra-interface
    object-group service Lotus_Notes_Utgaaande tcp
    description Frim Notes og ut til alle
    port-object eq domain
    port-object eq ftp
    port-object eq www
    port-object eq https
    port-object eq lotusnotes
    port-object eq pop3
    port-object eq pptp
    port-object eq smtp
    object-group service Lotus_Notes_inn tcp
    description From alle og inn til Notes
    port-object eq www
    port-object eq lotusnotes
    port-object eq pop3
    port-object eq smtp
    object-group service Reisebyraa tcp-udp
    port-object range 3702 3702
    port-object range 5500 5500
    port-object range 9876 9876
    object-group service Remote_Desktop tcp-udp
    description Tilgang til Remote Desktop
    port-object range 3389 3389
    object-group service Sand_Servicenter_50000 tcp-udp
    description Program tilgang til Sand Servicenter AS
    port-object range 50000 50000
    object-group service VNC_Remote_Admin tcp
    description Frå oss til alle
    port-object range 5900 5900
    object-group service Printer_Accept tcp-udp
    port-object range 9100 9100
    port-object eq echo
    object-group icmp-type Echo_Ping
    icmp-object echo
    icmp-object echo-reply
    object-group service Print tcp
    port-object range 9100 9100
    object-group service FTP_NADA tcp
    description Suldalsposten NADA tilgang
    port-object eq ftp
    port-object eq ftp-data
    object-group service Telefonsentral tcp
    description Hoftun
    port-object eq ftp
    port-object eq ftp-data
    port-object eq www
    port-object eq https
    port-object eq telnet
    object-group service Printer_inn_800 tcp
    description Fra 800  nettet og inn til 400 port 7777
    port-object range 7777 7777
    object-group service Suldalsposten tcp
    description Sending av mail vha Mac Mail programmet - åpner smtp
    port-object eq pop3
    port-object eq smtp
    object-group service http2 tcp
    port-object range 81 81
    object-group service DMZ_FTP_PASSIVE tcp-udp
    port-object range 55536 56559
    object-group service DMZ_FTP tcp-udp
    port-object range 20 21
    object-group service DMZ_HTTPS tcp-udp
    port-object range 443 443
    object-group service DMZ_HTTP tcp-udp
    port-object range 8080 8080
    object-group service DNS_Query tcp
    port-object range domain domain
    object-group service DUETT_SQL_PORT tcp-udp
    description For kobling mellom andre nett og duett server
    port-object range 54659 54659
    access-list outside_access_in extended permit ip any any
    access-list outside_access_out extended permit ip any any
    access-list vlan400_access_in extended deny ip any host 149.20.56.34
    access-list vlan400_access_in extended deny ip any host 149.20.56.32
    access-list vlan400_access_in extended permit ip any any
    access-list Vlan450_access_in extended deny ip any host 149.20.56.34
    access-list Vlan450_access_in extended deny ip any host 149.20.56.32
    access-list Vlan450_access_in extended permit ip any any
    access-list Vlan460_access_in extended deny ip any host 149.20.56.34
    access-list Vlan460_access_in extended deny ip any host 149.20.56.32
    access-list Vlan460_access_in extended permit ip any any
    access-list vlan400_access_out extended permit icmp any any object-group Echo_Ping
    access-list vlan400_access_out extended permit tcp any host NotesServer object-group Lotus_Notes_Utgaaande
    access-list vlan400_access_out extended permit tcp any host DomeneServer object-group Remote_Desktop
    access-list vlan400_access_out extended permit tcp any host TerminalServer object-group Remote_Desktop
    access-list vlan400_access_out extended permit tcp any host OvServer object-group http2
    access-list vlan400_access_out extended permit tcp any host NotesServer object-group Lotus_Notes_inn
    access-list vlan400_access_out extended permit tcp any host NotesServer object-group Remote_Desktop
    access-list vlan400_access_out extended permit tcp any host w8-eyeshare object-group Remote_Desktop
    access-list vlan400_access_out extended permit tcp any host w8-app object-group Remote_Desktop
    access-list vlan400_access_out extended permit tcp any host FonnaFlyMedia range 8400 8600
    access-list vlan400_access_out extended permit udp any host FonnaFlyMedia range 9000 9001
    access-list vlan400_access_out extended permit tcp 192.168.4.0 255.255.255.0 host DomeneServer
    access-list vlan400_access_out extended permit tcp 192.168.4.0 255.255.255.0 host w8-app object-group DUETT_SQL_PORT
    access-list Vlan500_access_in extended deny ip any host 149.20.56.34
    access-list Vlan500_access_in extended deny ip any host 149.20.56.32
    access-list Vlan500_access_in extended permit ip any any
    access-list vlan470_access_in extended deny ip any host 149.20.56.34
    access-list vlan470_access_in extended deny ip any host 149.20.56.32
    access-list vlan470_access_in extended permit ip any any
    access-list Vlan490_access_in extended deny ip any host 149.20.56.34
    access-list Vlan490_access_in extended deny ip any host 149.20.56.32
    access-list Vlan490_access_in extended permit ip any any
    access-list Vlan450_access_out extended permit icmp any any object-group Echo_Ping
    access-list Vlan1_access_out extended permit ip any any
    access-list Vlan1_access_out extended permit tcp any host w8-print object-group Remote_Desktop
    access-list Vlan1_access_out extended deny ip any any
    access-list Vlan1_access_out extended permit icmp any any echo-reply
    access-list Vlan460_access_out extended permit icmp any any object-group Echo_Ping
    access-list Vlan490_access_out extended permit icmp any any object-group Echo_Ping
    access-list Vlan490_access_out extended permit tcp any host 192.168.10.10 object-group DMZ_FTP
    access-list Vlan490_access_out extended permit tcp any host 192.168.10.10 object-group DMZ_FTP_PASSIVE
    access-list Vlan490_access_out extended permit tcp any host 192.168.10.10 object-group DMZ_HTTPS
    access-list Vlan490_access_out extended permit tcp any host 192.168.10.10 object-group DMZ_HTTP
    access-list Vlan500_access_out extended permit icmp any any object-group Echo_Ping
    access-list vlan470_access_out extended permit icmp any any object-group Echo_Ping
    access-list vlan470_access_out extended permit tcp any host 192.168.202.10 object-group Remote_Desktop
    access-list Vlan510_access_out extended permit icmp any any object-group Echo_Ping
    access-list vlan480_access_out extended permit ip any any
    access-list Vlan510_access_in extended permit ip any any
    access-list Vlan600_access_in extended permit ip any any
    access-list Vlan600_access_out extended permit icmp any any
    access-list Vlan600_access_out extended permit tcp any host w8-print object-group Remote_Desktop
    access-list Vlan600_access_out extended permit tcp 192.168.1.0 255.255.255.0 host w8-print eq www
    access-list Vlan600_access_out extended permit tcp 192.168.202.0 255.255.255.0 host w8-print eq www
    access-list Vlan600_access_out extended permit tcp 192.168.210.0 255.255.255.0 host w8-print eq www
    access-list Vlan600_access_in_1 extended permit ip any any
    access-list Vlan461_access_in extended permit ip any any
    access-list Vlan461_access_out extended permit icmp any any object-group Echo_Ping
    access-list vlan400_nat0_outbound extended permit ip 192.168.1.0 255.255.255.0 192.168.77.0 255.255.255.0
    access-list outside_20_cryptomap_1 extended permit ip 192.168.1.0 255.255.255.0 192.168.77.0 255.255.255.0
    access-list outside_20_cryptomap extended permit ip 192.168.1.0 255.255.255.0 192.168.77.0 255.255.255.0
    access-list Vlan462-Suldalsposten_access_in extended permit ip any any
    access-list Vlan462-Suldalsposten_access_out extended permit icmp any any echo-reply
    access-list Vlan462-Suldalsposten_access_out_1 extended permit icmp any any echo-reply
    access-list Vlan462-Suldalsposten_access_in_1 extended permit ip any any
    pager lines 24
    logging enable
    logging asdm informational
    mtu Vlan1 1500
    mtu outside 1500
    mtu vlan400 1500
    mtu Vlan450 1500
    mtu Vlan460-SuldalHotell 1500
    mtu Vlan461-SuldalHotellGjest 1500
    mtu vlan470-Kyrkjekontoret 1500
    mtu vlan480-Telefoni 1500
    mtu Vlan490-QNapBackup 1500
    mtu Vlan500-HellandBadlands 1500
    mtu Vlan510-IsTak 1500
    mtu Vlan600-SafeQ 1500
    mtu Vlan462-Suldalsposten 1500
    no failover
    monitor-interface Vlan1
    monitor-interface outside
    monitor-interface vlan400
    monitor-interface Vlan450
    monitor-interface Vlan460-SuldalHotell
    monitor-interface Vlan461-SuldalHotellGjest
    monitor-interface vlan470-Kyrkjekontoret
    monitor-interface vlan480-Telefoni
    monitor-interface Vlan490-QNapBackup
    monitor-interface Vlan500-HellandBadlands
    monitor-interface Vlan510-IsTak
    monitor-interface Vlan600-SafeQ
    monitor-interface Vlan462-Suldalsposten
    icmp unreachable rate-limit 1 burst-size 1
    asdm image disk0:/asdm-522.bin
    no asdm history enable
    arp timeout 14400
    global (outside) 1 interface
    nat (vlan400) 0 access-list vlan400_nat0_outbound
    nat (vlan400) 1 0.0.0.0 0.0.0.0 dns
    nat (Vlan450) 1 0.0.0.0 0.0.0.0 dns
    nat (Vlan460-SuldalHotell) 1 0.0.0.0 0.0.0.0
    nat (Vlan461-SuldalHotellGjest) 1 0.0.0.0 0.0.0.0
    nat (vlan470-Kyrkjekontoret) 1 0.0.0.0 0.0.0.0
    nat (Vlan490-QNapBackup) 1 0.0.0.0 0.0.0.0 dns
    nat (Vlan500-HellandBadlands) 1 0.0.0.0 0.0.0.0
    nat (Vlan510-IsTak) 1 0.0.0.0 0.0.0.0
    nat (Vlan600-SafeQ) 1 0.0.0.0 0.0.0.0
    nat (Vlan462-Suldalsposten) 1 0.0.0.0 0.0.0.0
    static (vlan400,outside) 79.x.x.x DomeneServer netmask 255.255.255.255
    static (vlan470-Kyrkjekontoret,outside) 79.x.x.x 192.168.202.10 netmask 255.255.255.255
    static (vlan400,outside) 79.x.x.x NotesServer netmask 255.255.255.255 dns
    static (vlan400,outside) 79.x.x.231 TerminalServer netmask 255.255.255.255
    static (vlan400,outside) 79.x.x.234 OvServer netmask 255.255.255.255
    static (vlan400,outside) 79.x.x.232 w8-eyeshare netmask 255.255.255.255
    static (Vlan490-QNapBackup,outside) 79.x.x.233 192.168.10.10 netmask 255.255.255.255 dns
    static (Vlan600-SafeQ,outside) 79.x.x.235 w8-print netmask 255.255.255.255
    static (vlan400,outside) 79.x.x.236 w8-app netmask 255.255.255.255
    static (Vlan450,vlan400) 192.168.210.0 192.168.210.0 netmask 255.255.255.0
    static (Vlan500-HellandBadlands,vlan400) 192.168.30.0 192.168.30.0 netmask 255.255.255.0
    static (vlan400,Vlan500-HellandBadlands) 192.168.1.0 192.168.1.0 netmask 255.255.255.0
    static (vlan400,Vlan450) 192.168.1.0 192.168.1.0 netmask 255.255.255.0
    static (vlan400,outside) 79.x.x.252 FonnaFlyMedia netmask 255.255.255.255
    static (Vlan462-Suldalsposten,vlan400) 192.168.4.0 192.168.4.0 netmask 255.255.255.0
    static (vlan400,Vlan462-Suldalsposten) 192.168.1.0 192.168.1.0 netmask 255.255.255.0
    static (vlan400,Vlan600-SafeQ) 192.168.1.0 192.168.1.0 netmask 255.255.255.0
    static (Vlan600-SafeQ,vlan400) 192.168.50.0 192.168.50.0 netmask 255.255.255.0
    static (Vlan600-SafeQ,Vlan450) 192.168.50.0 192.168.50.0 netmask 255.255.255.0
    static (Vlan600-SafeQ,vlan470-Kyrkjekontoret) 192.168.50.0 192.168.50.0 netmask 255.255.255.0
    static (Vlan450,Vlan600-SafeQ) 192.168.210.0 192.168.210.0 netmask 255.255.255.0
    static (vlan470-Kyrkjekontoret,Vlan600-SafeQ) 192.168.202.0 192.168.202.0 netmask 255.255.255.0
    access-group Vlan1_access_out out interface Vlan1
    access-group outside_access_in in interface outside
    access-group outside_access_out out interface outside
    access-group vlan400_access_in in interface vlan400
    access-group vlan400_access_out out interface vlan400
    access-group Vlan450_access_in in interface Vlan450
    access-group Vlan450_access_out out interface Vlan450
    access-group Vlan460_access_in in interface Vlan460-SuldalHotell
    access-group Vlan460_access_out out interface Vlan460-SuldalHotell
    access-group Vlan461_access_in in interface Vlan461-SuldalHotellGjest
    access-group Vlan461_access_out out interface Vlan461-SuldalHotellGjest
    access-group vlan470_access_in in interface vlan470-Kyrkjekontoret
    access-group vlan470_access_out out interface vlan470-Kyrkjekontoret
    access-group vlan480_access_out out interface vlan480-Telefoni
    access-group Vlan490_access_in in interface Vlan490-QNapBackup
    access-group Vlan490_access_out out interface Vlan490-QNapBackup
    access-group Vlan500_access_in in interface Vlan500-HellandBadlands
    access-group Vlan500_access_out out interface Vlan500-HellandBadlands
    access-group Vlan510_access_in in interface Vlan510-IsTak
    access-group Vlan510_access_out out interface Vlan510-IsTak
    access-group Vlan600_access_in_1 in interface Vlan600-SafeQ
    access-group Vlan600_access_out out interface Vlan600-SafeQ
    access-group Vlan462-Suldalsposten_access_in_1 in interface Vlan462-Suldalsposten
    access-group Vlan462-Suldalsposten_access_out_1 out interface Vlan462-Suldalsposten
    route outside 0.0.0.0 0.0.0.0 79.x.x.225 1
    timeout xlate 3:00:00
    timeout conn 1:00:00 half-closed 0:10:00 udp 0:02:00 icmp 0:00:02
    timeout sunrpc 0:10:00 h323 0:05:00 h225 1:00:00 mgcp 0:05:00 mgcp-pat 0:05:00
    timeout sip 0:30:00 sip_media 0:02:00 sip-invite 0:03:00 sip-disconnect 0:02:00
    timeout uauth 0:05:00 absolute
    username x password x encrypted privilege 15
    aaa authentication ssh console LOCAL
    http server enable
    http 192.168.210.0 255.255.255.0 Vlan450
    http 192.168.200.0 255.255.255.0 Vlan1
    http 192.168.1.0 255.255.255.0 vlan400
    no snmp-server location
    no snmp-server contact
    snmp-server community public
    snmp-server enable traps snmp authentication linkup linkdown coldstart
    crypto ipsec transform-set ESP-3DES-SHA esp-3des esp-sha-hmac
    crypto map outside_map 20 match address outside_20_cryptomap_1
    crypto map outside_map 20 set pfs
    crypto map outside_map 20 set peer 62.92.159.137
    crypto map outside_map 20 set transform-set ESP-3DES-SHA
    crypto map outside_map interface outside
    crypto isakmp enable outside
    crypto isakmp enable vlan400
    crypto isakmp policy 10
    authentication pre-share
    encryption 3des
    hash sha
    group 2
    lifetime 86400
    tunnel-group 62.92.159.137 type ipsec-l2l
    tunnel-group 62.92.159.137 ipsec-attributes
    pre-shared-key *
    telnet 192.168.200.0 255.255.255.0 Vlan1
    telnet 192.168.1.0 255.255.255.0 vlan400
    telnet timeout 5
    ssh 171.68.225.216 255.255.255.255 outside
    ssh timeout 5
    console timeout 0
    dhcpd update dns both
    dhcpd option 6 ip 81.167.36.3 81.167.36.11 interface Vlan1
    dhcpd option 6 ip 81.167.36.3 81.167.36.11 interface outside
    dhcpd address 192.168.1.100-192.168.1.225 vlan400
    dhcpd option 6 ip DomeneServer 81.167.36.11 interface vlan400
    dhcpd option 3 ip 192.168.1.1 interface vlan400
    dhcpd enable vlan400
    dhcpd address 192.168.210.100-192.168.210.200 Vlan450
    dhcpd option 6 ip 81.167.36.3 81.167.36.11 interface Vlan450
    dhcpd option 3 ip 192.168.210.1 interface Vlan450
    dhcpd enable Vlan450
    dhcpd address 192.168.2.100-192.168.2.150 Vlan460-SuldalHotell
    dhcpd option 6 ip 81.167.36.3 81.167.36.11 interface Vlan460-SuldalHotell
    dhcpd option 3 ip 192.168.2.1 interface Vlan460-SuldalHotell
    dhcpd enable Vlan460-SuldalHotell
    dhcpd address 192.168.3.100-192.168.3.200 Vlan461-SuldalHotellGjest
    dhcpd option 6 ip 81.167.36.3 81.167.36.11 interface Vlan461-SuldalHotellGjest
    dhcpd option 3 ip 192.168.3.1 interface Vlan461-SuldalHotellGjest
    dhcpd enable Vlan461-SuldalHotellGjest
    dhcpd address 192.168.202.100-192.168.202.199 vlan470-Kyrkjekontoret
    dhcpd option 3 ip 192.168.202.1 interface vlan470-Kyrkjekontoret
    dhcpd option 6 ip 81.167.36.3 81.167.36.11 interface vlan470-Kyrkjekontoret
    dhcpd enable vlan470-Kyrkjekontoret
    dhcpd option 3 ip 192.168.20.1 interface vlan480-Telefoni
    dhcpd option 6 ip 81.167.36.3 81.167.36.11 interface vlan480-Telefoni
    dhcpd address 192.168.10.80-192.168.10.90 Vlan490-QNapBackup
    dhcpd option 6 ip 81.167.36.3 81.167.36.11 interface Vlan490-QNapBackup
    dhcpd option 3 ip 192.168.10.1 interface Vlan490-QNapBackup
    dhcpd address 192.168.30.100-192.168.30.199 Vlan500-HellandBadlands
    dhcpd option 6 ip 81.167.36.3 81.167.36.11 interface Vlan500-HellandBadlands
    dhcpd option 3 ip 192.168.30.1 interface Vlan500-HellandBadlands
    dhcpd enable Vlan500-HellandBadlands
    dhcpd address 192.168.40.100-192.168.40.150 Vlan510-IsTak
    dhcpd option 6 ip 81.167.36.3 81.167.36.11 interface Vlan510-IsTak
    dhcpd option 3 ip 192.168.40.1 interface Vlan510-IsTak
    dhcpd enable Vlan510-IsTak
    dhcpd address 192.168.50.150-192.168.50.199 Vlan600-SafeQ
    dhcpd option 6 ip 81.167.36.3 81.167.36.11 interface Vlan600-SafeQ
    dhcpd enable Vlan600-SafeQ
    dhcpd address 192.168.4.100-192.168.4.150 Vlan462-Suldalsposten
    dhcpd option 6 ip DomeneServer 81.167.36.11 interface Vlan462-Suldalsposten
    dhcpd option 3 ip 192.168.4.1 interface Vlan462-Suldalsposten
    dhcpd enable Vlan462-Suldalsposten
    policy-map type inspect dns preset_dns_map
    parameters
      message-length maximum 512
    prompt hostname context
    Cryptochecksum:x
    : end
    Config site 1:
    : Saved
    ASA Version 7.2(4)
    hostname ciscoasa
    domain-name default.domain.invalid
    enable password x encrypted
    passwd x encrypted
    names
    interface Vlan1
    nameif inside
    security-level 100
    ip address 192.168.77.1 255.255.255.0
    interface Vlan2
    nameif outside
    security-level 0
    pppoe client vpdn group Telenor
    ip address pppoe setroute
    interface Ethernet0/0
    switchport access vlan 2
    interface Ethernet0/1
    interface Ethernet0/2
    interface Ethernet0/3
    interface Ethernet0/4
    interface Ethernet0/5
    switchport access vlan 15
    interface Ethernet0/6
    interface Ethernet0/7
    ftp mode passive
    dns server-group DefaultDNS
    domain-name default.domain.invalid
    access-list outside_access_in extended permit icmp any any echo-reply log disable
    access-list outside_1_cryptomap extended permit ip 192.168.77.0 255.255.255.0 192.168.1.0 255.255.255.0
    access-list inside_nat0_outbound extended permit ip 192.168.77.0 255.255.255.0 192.168.1.0 255.255.255.0
    pager lines 24
    logging asdm informational
    mtu inside 1500
    mtu outside 1500
    icmp unreachable rate-limit 1 burst-size 1
    asdm image disk0:/asdm-524.bin
    no asdm history enable
    arp timeout 14400
    global (outside) 1 interface
    nat (inside) 0 access-list inside_nat0_outbound
    nat (inside) 1 0.0.0.0 0.0.0.0
    access-group outside_access_in in interface outside
    timeout xlate 3:00:00
    timeout conn 1:00:00 half-closed 0:10:00 udp 0:02:00 icmp 0:00:02
    timeout sunrpc 0:10:00 h323 0:05:00 h225 1:00:00 mgcp 0:05:00 mgcp-pat 0:05:00
    timeout sip 0:30:00 sip_media 0:02:00 sip-invite 0:03:00 sip-disconnect 0:02:00
    timeout sip-provisional-media 0:02:00 uauth 0:05:00 absolute
    http server enable
    http 192.168.77.0 255.255.255.0 inside
    http 192.168.1.0 255.255.255.0 inside
    no snmp-server location
    no snmp-server contact
    snmp-server enable traps snmp authentication linkup linkdown coldstart
    crypto ipsec transform-set ESP-3DES-SHA esp-3des esp-sha-hmac
    crypto map outside_map 1 match address outside_1_cryptomap
    crypto map outside_map 1 set pfs
    crypto map outside_map 1 set peer 79.160.252.226
    crypto map outside_map 1 set transform-set ESP-3DES-SHA
    crypto map outside_map interface outside
    crypto isakmp enable inside
    crypto isakmp enable outside
    crypto isakmp policy 10
    authentication pre-share
    encryption 3des
    hash sha
    group 2
    lifetime 86400
    telnet 192.168.77.0 255.255.255.0 inside
    telnet timeout 5
    ssh timeout 5
    console timeout 0
    vpdn group Telenor request dialout pppoe
    vpdn group Telenor localname x
    vpdn group Telenor ppp authentication chap
    vpdn username x password x store-local
    dhcpd auto_config outside
    dhcpd address 192.168.77.100-192.168.77.130 inside
    dhcpd dns 192.168.77.1 interface inside
    dhcpd option 6 ip 130.67.15.198 193.213.112.4 interface inside
    dhcpd enable inside
    dhcpd option 6 ip 130.67.15.198 193.213.112.4 interface outside
    tunnel-group 79.160.252.226 type ipsec-l2l
    tunnel-group 79.160.252.226 ipsec-attributes
    pre-shared-key *
    class-map inspection_default
    match default-inspection-traffic
    policy-map type inspect dns preset_dns_map
    parameters
      message-length maximum 512
    policy-map global_policy
    class inspection_default
      inspect dns preset_dns_map
      inspect ftp
      inspect h323 h225
      inspect h323 ras
      inspect rsh
      inspect rtsp
      inspect esmtp
      inspect sqlnet
      inspect skinny
      inspect sunrpc
      inspect xdmcp
      inspect sip
      inspect netbios
      inspect tftp
    service-policy global_policy global
    prompt hostname context
    Cryptochecksum:x
    : end

    Hi,
    The addition of a new network to the existing L2L VPN should be a pretty simple process.
    Essentially you will have to add the network to the Crypto ACL present in the "crypto map" configurations. You will also have to configure the NAT0 configuration for it in the proper interfaces of the ASA. These configurations are all done on both ends of the L2L VPN connection.
    Looking at your above configurations it would seem that you will need the following configurations
    SITE 1
    We add the new network to both the crypto ACL and the NAT0 ACL
    access-list outside_1_cryptomap extended permit ip 192.168.77.0 255.255.255.0 192.168.20.0 255.255.255.0
    access-list inside_nat0_outbound extended permit ip 192.168.77.0 255.255.255.0 192.168.20.0 255.255.255.0
    SITE 2
    We add the new network to the crypto ACL
    We create a new NAT0 configuration for the Vlan480 interface as it has no previous NAT0 configuration
    access-list outside_20_cryptomap_1 extended permit ip 192.168.20.0 255.255.255.0 192.168.77.0 255.255.255.0
    access-list VLAN480-NAT0 remark NAT0 for VPN
    access-list VLAN480-NAT0 permit ip 192.168.20.0 255.255.255.0 192.168.77.0 255.255.255.0
    nat (vlan480-Telefoni) 0 access-list VLAN480-NAT0
    These configurations should pretty much do the trick.
    Let me know if it worked
    - Jouni

  • Cisco ASA 5505 Site to Site VPN

    Hello All,
    First time posting to the forums. I've been working with Cisco ASA 5505 for a number of months and recently I purchased a 2nd ASA with the goal of setting up Site to Site VPN tunnel. It look so simple from the number of videos that I have watched on the internet. But when I have done it suprise suprise it didn't work for me ... I have deleted the tunnels a number of times and attempted to recreate them. I am using the VPN wizard in the ADM to create the tunnel. Both the asa are 5505 and have the same same firmware etc.
    I would appreciate any help that can be directed towards this issue please.  Slowly losing my mind
    Please see details below:
    Both ADM are 7.1
    IOS
    ASA 1
    aved
    ASA Version 9.0(1)
    hostname PAYBACK
    enable password HSMurh79NVmatjY0 encrypted
    xlate per-session deny tcp any4 any4
    xlate per-session deny tcp any4 any6
    xlate per-session deny tcp any6 any4
    xlate per-session deny tcp any6 any6
    xlate per-session deny udp any4 any4 eq domain
    xlate per-session deny udp any4 any6 eq domain
    xlate per-session deny udp any6 any4 eq domain
    xlate per-session deny udp any6 any6 eq domain
    passwd 2KFQnbNIdI.2KYOU encrypted
    names
    ip local pool VPN1 192.168.50.1-192.168.50.254 mask 255.255.255.0
    interface Ethernet0/0
    switchport access vlan 2
    speed 100
    duplex full
    interface Ethernet0/1
    description Trunk link to SW1
    switchport trunk allowed vlan 1,10,20,30,40
    switchport trunk native vlan 1
    switchport mode trunk
    interface Ethernet0/2
    interface Ethernet0/3
    interface Ethernet0/4
    interface Ethernet0/5
    interface Ethernet0/6
    interface Ethernet0/7
    interface Vlan1
    no nameif
    no security-level
    no ip address
    interface Vlan2
    nameif outside
    security-level 0
    ip address 92.51.193.158 255.255.255.252
    interface Vlan10
    nameif inside
    security-level 100
    ip address 192.168.10.1 255.255.255.0
    interface Vlan20
    nameif servers
    security-level 100
    ip address 192.168.20.1 255.255.255.0
    interface Vlan30
    nameif printers
    security-level 100
    ip address 192.168.30.1 255.255.255.0
    interface Vlan40
    nameif wireless
    security-level 100
    ip address 192.168.40.1 255.255.255.0
    banner login line Welcome to Payback Loyalty Systems
    boot system disk0:/asa901-k8.bin
    ftp mode passive
    clock summer-time GMT/IDT recurring last Sun Mar 1:00 last Sun Oct 2:00
    dns domain-lookup outside
    dns domain-lookup inside
    dns domain-lookup servers
    dns domain-lookup printers
    dns domain-lookup wireless
    dns server-group DefaultDNS
    name-server 83.147.160.2
    name-server 83.147.160.130
    same-security-traffic permit inter-interface
    object network obj_any
    subnet 0.0.0.0 0.0.0.0
    object network ftp_server
    object network Internal_Report_Server
    host 192.168.20.21
    description Automated Report Server Internal Address
    object network Report_Server
    host 89.234.126.9
    description Automated Report Server
    object service RDP
    service tcp destination eq 3389
    description RDP to Server
    object network Host_QA_Server
    host 89.234.126.10
    description QA Host External Address
    object network Internal_Host_QA
    host 192.168.20.22
    description Host of VM machine for QA
    object network Internal_QA_Web_Server
    host 192.168.20.23
    description Web Server in QA environment
    object network Web_Server_QA_VM
    host 89.234.126.11
    description Web server in QA environment
    object service SQL_Server
    service tcp destination eq 1433
    object network Demo_Server
    host 89.234.126.12
    description Server set up to Demo Product
    object network Internal_Demo_Server
    host 192.168.20.24
    description Internal IP Address of Demo Server
    object network NETWORK_OBJ_192.168.20.0_24
    subnet 192.168.20.0 255.255.255.0
    object network NETWORK_OBJ_192.168.50.0_26
    subnet 192.168.50.0 255.255.255.192
    object network NETWORK_OBJ_192.168.0.0_16
    subnet 192.168.0.0 255.255.0.0
    object service MSSQL
    service tcp destination eq 1434
    description MSSQL port
    object network VPN-network
    subnet 192.168.50.0 255.255.255.0
    object network NETWORK_OBJ_192.168.50.0_24
    subnet 192.168.50.0 255.255.255.0
    object service TS
    service tcp destination eq 4400
    object service TS_Return
    service tcp source eq 4400
    object network External_QA_3
    host 89.234.126.13
    object network Internal_QA_3
    host 192.168.20.25
    object network Dev_WebServer
    host 192.168.20.27
    object network External_Dev_Web
    host 89.234.126.14
    object network CIX_Subnet
    subnet 192.168.100.0 255.255.255.0
    object network NETWORK_OBJ_192.168.10.0_24
    subnet 192.168.10.0 255.255.255.0
    object network NETWORK_OBJ_84.39.233.50
    host 84.39.233.50
    object network NETWORK_OBJ_92.51.193.158
    host 92.51.193.158
    object network NETWORK_OBJ_192.168.100.0_24
    subnet 192.168.100.0 255.255.255.0
    object network NETWORK_OBJ_192.168.1.0_24
    subnet 192.168.1.0 255.255.255.0
    object-group service DM_INLINE_SERVICE_1
    service-object tcp destination eq ftp
    service-object tcp destination eq netbios-ssn
    service-object tcp destination eq smtp
    service-object object TS
    object-group network Payback_Internal
    network-object 192.168.10.0 255.255.255.0
    network-object 192.168.20.0 255.255.255.0
    network-object 192.168.40.0 255.255.255.0
    object-group service DM_INLINE_SERVICE_3
    service-object tcp destination eq www
    service-object tcp destination eq https
    service-object object TS
    service-object object TS_Return
    object-group service DM_INLINE_SERVICE_4
    service-object object RDP
    service-object tcp destination eq www
    service-object tcp destination eq https
    object-group service DM_INLINE_SERVICE_5
    service-object object MSSQL
    service-object object RDP
    service-object object TS
    object-group protocol TCPUDP
    protocol-object udp
    protocol-object tcp
    object-group service DM_INLINE_SERVICE_6
    service-object object TS
    service-object object TS_Return
    service-object tcp destination eq www
    service-object tcp destination eq https
    access-list outside_access_in remark This rule is allowing from internet to interal server.
    access-list outside_access_in remark Allowed:
    access-list outside_access_in remark FTP
    access-list outside_access_in remark RDP
    access-list outside_access_in remark SMTP
    access-list outside_access_in remark Net Bios
    access-list outside_access_in remark SQL
    access-list outside_access_in remark TS - 4400
    access-list outside_access_in extended permit object-group DM_INLINE_SERVICE_1 any4 object Internal_Report_Server
    access-list outside_access_in remark Access rule to internal host QA
    access-list outside_access_in remark Allowed:
    access-list outside_access_in remark HTTP
    access-list outside_access_in remark RDP
    access-list outside_access_in extended permit tcp any4 object Internal_Host_QA eq www
    access-list outside_access_in remark Access to INternal Web Server:
    access-list outside_access_in remark Allowed:
    access-list outside_access_in remark HTTP
    access-list outside_access_in remark RDP
    access-list outside_access_in extended permit object-group DM_INLINE_SERVICE_3 any4 object Internal_QA_Web_Server
    access-list outside_access_in remark Rule for allowing access to Demo server
    access-list outside_access_in remark Allowed:
    access-list outside_access_in remark RDP
    access-list outside_access_in remark MSSQL
    access-list outside_access_in extended permit object-group DM_INLINE_SERVICE_4 any4 object Internal_Demo_Server
    access-list outside_access_in extended permit object-group DM_INLINE_SERVICE_5 any object Internal_QA_3
    access-list outside_access_in remark Access for Development WebServer
    access-list outside_access_in extended permit object-group DM_INLINE_SERVICE_6 any object Dev_WebServer
    access-list AnyConnect_Client_Local_Print extended deny ip any4 any4
    access-list AnyConnect_Client_Local_Print extended permit tcp any4 any4 eq lpd
    access-list AnyConnect_Client_Local_Print remark IPP: Internet Printing Protocol
    access-list AnyConnect_Client_Local_Print extended permit tcp any4 any4 eq 631
    access-list AnyConnect_Client_Local_Print remark Windows' printing port
    access-list AnyConnect_Client_Local_Print extended permit tcp any4 any4 eq 9100
    access-list AnyConnect_Client_Local_Print remark mDNS: multicast DNS protocol
    access-list AnyConnect_Client_Local_Print extended permit udp any4 host 224.0.0.251 eq 5353
    access-list AnyConnect_Client_Local_Print remark LLMNR: Link Local Multicast Name Resolution protocol
    access-list AnyConnect_Client_Local_Print extended permit udp any4 host 224.0.0.252 eq 5355
    access-list AnyConnect_Client_Local_Print remark TCP/NetBIOS protocol
    access-list AnyConnect_Client_Local_Print extended permit tcp any4 any4 eq 137
    access-list AnyConnect_Client_Local_Print extended permit udp any4 any4 eq netbios-ns
    access-list Payback_VPN_splitTunnelAcl standard permit 192.168.20.0 255.255.255.0
    access-list outside_cryptomap extended permit ip 192.168.10.0 255.255.255.0 192.168.100.0 255.255.255.0
    pager lines 24
    logging enable
    logging console informational
    logging asdm informational
    logging from-address
    [email protected]
    logging recipient-address
    [email protected]
    level alerts
    mtu outside 1500
    mtu inside 1500
    mtu servers 1500
    mtu printers 1500
    mtu wireless 1500
    no failover
    icmp unreachable rate-limit 1 burst-size 1
    asdm image disk0:/asdm-711-52.bin
    no asdm history enable
    arp timeout 14400
    no arp permit-nonconnected
    nat (inside,outside) source dynamic any interface
    nat (wireless,outside) source dynamic any interface
    nat (servers,outside) source dynamic any interface
    nat (servers,outside) source static Internal_Report_Server Report_Server
    nat (servers,outside) source static Internal_Host_QA Host_QA_Server
    nat (servers,outside) source static Internal_QA_Web_Server Web_Server_QA_VM
    nat (servers,outside) source static Internal_Demo_Server Demo_Server
    nat (servers,outside) source static NETWORK_OBJ_192.168.20.0_24 NETWORK_OBJ_192.168.20.0_24 destination static NETWORK_OBJ_192.168.50.0_24 NETWORK_OBJ_192.168.50.0_24 no-proxy-arp route-lookup
    nat (servers,outside) source static Internal_QA_3 External_QA_3
    nat (servers,outside) source static Dev_WebServer External_Dev_Web
    nat (inside,outside) source static NETWORK_OBJ_192.168.10.0_24 NETWORK_OBJ_192.168.10.0_24 destination static NETWORK_OBJ_192.168.1.0_24 NETWORK_OBJ_192.168.1.0_24 no-proxy-arp route-lookup
    nat (inside,outside) source static NETWORK_OBJ_192.168.10.0_24 NETWORK_OBJ_192.168.10.0_24 destination static NETWORK_OBJ_192.168.100.0_24 NETWORK_OBJ_192.168.100.0_24 no-proxy-arp route-lookup
    access-group outside_access_in in interface outside
    route outside 0.0.0.0 0.0.0.0 92.51.193.157 1
    timeout xlate 3:00:00
    timeout pat-xlate 0:00:30
    timeout conn 1:00:00 half-closed 0:10:00 udp 0:02:00 icmp 0:00:02
    timeout sunrpc 0:10:00 h323 0:05:00 h225 1:00:00 mgcp 0:05:00 mgcp-pat 0:05:00
    timeout sip 0:30:00 sip_media 0:02:00 sip-invite 0:03:00 sip-disconnect 0:02:00
    timeout sip-provisional-media 0:02:00 uauth 0:05:00 absolute
    timeout tcp-proxy-reassembly 0:01:00
    timeout floating-conn 0:00:00
    dynamic-access-policy-record DfltAccessPolicy
    user-identity default-domain LOCAL
    aaa authentication ssh console LOCAL
    http server enable
    http 192.168.10.0 255.255.255.0 inside
    http 192.168.40.0 255.255.255.0 wireless
    no snmp-server location
    no snmp-server contact
    snmp-server enable traps snmp authentication linkup linkdown coldstart
    crypto ipsec ikev1 transform-set ESP-AES-256-MD5 esp-aes-256 esp-md5-hmac
    crypto ipsec ikev1 transform-set ESP-DES-SHA esp-des esp-sha-hmac
    crypto ipsec ikev1 transform-set ESP-DES-MD5 esp-des esp-md5-hmac
    crypto ipsec ikev1 transform-set ESP-AES-192-MD5 esp-aes-192 esp-md5-hmac
    crypto ipsec ikev1 transform-set ESP-3DES-MD5 esp-3des esp-md5-hmac
    crypto ipsec ikev1 transform-set ESP-AES-256-SHA esp-aes-256 esp-sha-hmac
    crypto ipsec ikev1 transform-set ESP-AES-128-SHA esp-aes esp-sha-hmac
    crypto ipsec ikev1 transform-set ESP-AES-192-SHA esp-aes-192 esp-sha-hmac
    crypto ipsec ikev1 transform-set ESP-AES-128-MD5 esp-aes esp-md5-hmac
    crypto ipsec ikev1 transform-set ESP-3DES-SHA esp-3des esp-sha-hmac
    crypto ipsec ikev1 transform-set ESP-AES-128-SHA-TRANS esp-aes esp-sha-hmac
    crypto ipsec ikev1 transform-set ESP-AES-128-SHA-TRANS mode transport
    crypto ipsec ikev1 transform-set ESP-AES-128-MD5-TRANS esp-aes esp-md5-hmac
    crypto ipsec ikev1 transform-set ESP-AES-128-MD5-TRANS mode transport
    crypto ipsec ikev1 transform-set ESP-AES-192-SHA-TRANS esp-aes-192 esp-sha-hmac
    crypto ipsec ikev1 transform-set ESP-AES-192-SHA-TRANS mode transport
    crypto ipsec ikev1 transform-set ESP-AES-192-MD5-TRANS esp-aes-192 esp-md5-hmac
    crypto ipsec ikev1 transform-set ESP-AES-192-MD5-TRANS mode transport
    crypto ipsec ikev1 transform-set ESP-AES-256-SHA-TRANS esp-aes-256 esp-sha-hmac
    crypto ipsec ikev1 transform-set ESP-AES-256-SHA-TRANS mode transport
    crypto ipsec ikev1 transform-set ESP-AES-256-MD5-TRANS esp-aes-256 esp-md5-hmac
    crypto ipsec ikev1 transform-set ESP-AES-256-MD5-TRANS mode transport
    crypto ipsec ikev1 transform-set ESP-3DES-SHA-TRANS esp-3des esp-sha-hmac
    crypto ipsec ikev1 transform-set ESP-3DES-SHA-TRANS mode transport
    crypto ipsec ikev1 transform-set ESP-3DES-MD5-TRANS esp-3des esp-md5-hmac
    crypto ipsec ikev1 transform-set ESP-3DES-MD5-TRANS mode transport
    crypto ipsec ikev1 transform-set ESP-DES-SHA-TRANS esp-des esp-sha-hmac
    crypto ipsec ikev1 transform-set ESP-DES-SHA-TRANS mode transport
    crypto ipsec ikev1 transform-set ESP-DES-MD5-TRANS esp-des esp-md5-hmac
    crypto ipsec ikev1 transform-set ESP-DES-MD5-TRANS mode transport
    crypto ipsec ikev2 ipsec-proposal AES256
    protocol esp encryption aes-256
    protocol esp integrity sha-1 md5
    crypto ipsec ikev2 ipsec-proposal AES192
    protocol esp encryption aes-192
    protocol esp integrity sha-1 md5
    crypto ipsec ikev2 ipsec-proposal AES
    protocol esp encryption aes
    protocol esp integrity sha-1 md5
    crypto ipsec ikev2 ipsec-proposal 3DES
    protocol esp encryption 3des
    protocol esp integrity sha-1 md5
    crypto ipsec ikev2 ipsec-proposal DES
    protocol esp encryption des
    protocol esp integrity sha-1 md5
    crypto ipsec security-association pmtu-aging infinite
    crypto map outside_map 1 match address outside_cryptomap
    crypto map outside_map 1 set pfs
    crypto map outside_map 1 set peer 84.39.233.50
    crypto map outside_map 1 set ikev1 transform-set ESP-AES-128-SHA ESP-AES-128-MD5 ESP-AES-192-SHA ESP-AES-192-MD5 ESP-AES-256-SHA ESP-AES-256-MD5 ESP-3DES-SHA ESP-3DES-MD5 ESP-DES-SHA ESP-DES-MD5
    crypto map outside_map 1 set ikev2 ipsec-proposal AES256 AES192 AES 3DES DES
    crypto map outside_map interface outside
    crypto ca trustpool policy
    crypto ikev2 policy 1
    encryption aes-256
    integrity sha
    group 5
    prf sha
    lifetime seconds 86400
    crypto ikev2 policy 10
    encryption aes-192
    integrity sha
    group 5
    prf sha
    lifetime seconds 86400
    crypto ikev2 policy 20
    encryption aes
    integrity sha
    group 5
    prf sha
    lifetime seconds 86400
    crypto ikev2 policy 30
    encryption 3des
    integrity sha
    group 5
    prf sha
    lifetime seconds 86400
    crypto ikev2 policy 40
    encryption des
    integrity sha
    group 5
    prf sha
    lifetime seconds 86400
    crypto ikev2 enable outside client-services port 443
    crypto ikev1 enable outside
    crypto ikev1 policy 10
    authentication pre-share
    encryption 3des
    hash sha
    group 2
    lifetime 86400
    telnet timeout 5
    ssh 77.75.100.208 255.255.255.240 outside
    ssh 192.168.10.0 255.255.255.0 inside
    ssh 192.168.40.0 255.255.255.0 wireless
    ssh timeout 5
    console timeout 0
    dhcpd dns 192.168.0.1
    dhcpd auto_config outside
    dhcpd address 192.168.10.21-192.168.10.240 inside
    dhcpd dns 192.168.20.21 83.147.160.2 interface inside
    dhcpd option 15 ascii paybackloyalty.com interface inside
    dhcpd enable inside
    dhcpd address 192.168.40.21-192.168.40.240 wireless
    dhcpd dns 192.168.20.21 83.147.160.2 interface wireless
    dhcpd update dns interface wireless
    dhcpd option 15 ascii paybackloyalty.com interface wireless
    dhcpd enable wireless
    threat-detection basic-threat
    threat-detection statistics access-list
    no threat-detection statistics tcp-intercept
    group-policy Payback_VPN internal
    group-policy Payback_VPN attributes
    vpn-simultaneous-logins 10
    vpn-tunnel-protocol ikev1
    split-tunnel-policy tunnelspecified
    split-tunnel-network-list value Payback_VPN_splitTunnelAcl
    group-policy DfltGrpPolicy attributes
    dns-server value 83.147.160.2 83.147.160.130
    vpn-tunnel-protocol ikev1 ikev2 ssl-clientless
    group-policy GroupPolicy_84.39.233.50 internal
    group-policy GroupPolicy_84.39.233.50 attributes
    vpn-tunnel-protocol ikev1 ikev2
    username Noelle password XB/IpvYaATP.2QYm encrypted
    username Noelle attributes
    vpn-group-policy Payback_VPN
    service-type remote-access
    username Eanna password vXILR9ZZQIsd1Naw encrypted privilege 0
    username Eanna attributes
    vpn-group-policy Payback_VPN
    service-type remote-access
    username Michael password qpbleUqUEchRrgQX encrypted
    username Michael attributes
    vpn-group-policy Payback_VPN
    service-type remote-access
    username Danny password .7fEXdzESUk6S/cC encrypted privilege 0
    username Danny attributes
    vpn-group-policy Payback_VPN
    service-type remote-access
    username Aileen password tytrelqvV5VRX2pz encrypted privilege 0
    username Aileen attributes
    vpn-group-policy Payback_VPN
    service-type remote-access
    username Aidan password aDu6YH0V5XaxpEPg encrypted privilege 0
    username Aidan attributes
    vpn-group-policy Payback_VPN
    service-type remote-access
    username gordon password 6e6Djaz3W/XH59zX encrypted privilege 15
    username shane.c password iqGMoWOnfO6YKXbw encrypted
    username shane.c attributes
    vpn-group-policy Payback_VPN
    service-type remote-access
    username Shane password uYePLcrFadO9pBZx encrypted
    username Shane attributes
    vpn-group-policy Payback_VPN
    service-type remote-access
    username James password TdYPv1pvld/hPM0d encrypted
    username James attributes
    vpn-group-policy Payback_VPN
    service-type remote-access
    username mark password yruxpddqfyNb.qFn encrypted
    username mark attributes
    service-type admin
    username Mary password XND5FTEiyu1L1zFD encrypted
    username Mary attributes
    vpn-group-policy Payback_VPN
    service-type remote-access
    username Massimo password vs65MMo4rM0l4rVu encrypted privilege 0
    username Massimo attributes
    vpn-group-policy Payback_VPN
    service-type remote-access
    tunnel-group Payback_VPN type remote-access
    tunnel-group Payback_VPN general-attributes
    address-pool VPN1
    default-group-policy Payback_VPN
    tunnel-group Payback_VPN ipsec-attributes
    ikev1 pre-shared-key *****
    tunnel-group 84.39.233.50 type ipsec-l2l
    tunnel-group 84.39.233.50 general-attributes
    default-group-policy GroupPolicy_84.39.233.50
    tunnel-group 84.39.233.50 ipsec-attributes
    ikev1 pre-shared-key *****
    ikev2 remote-authentication pre-shared-key *****
    ikev2 local-authentication pre-shared-key *****
    class-map global-class
    match default-inspection-traffic
    policy-map global-policy
    class global-class
      inspect dns
      inspect ftp
      inspect h323 h225
      inspect h323 ras
      inspect ip-options
      inspect netbios
      inspect pptp
      inspect rsh
      inspect rtsp
      inspect sip
      inspect snmp
      inspect sqlnet
      inspect sunrpc
      inspect tftp
      inspect xdmcp
      inspect icmp error
      inspect icmp
    service-policy global-policy global
    smtp-server 192.168.20.21
    prompt hostname context
    no call-home reporting anonymous
    call-home
    profile CiscoTAC-1
      no active
      destination address http https://tools.cisco.com/its/service/oddce/services/DDCEService
      destination address email [email protected]
      destination transport-method http
      subscribe-to-alert-group diagnostic
      subscribe-to-alert-group environment
      subscribe-to-alert-group inventory periodic monthly
      subscribe-to-alert-group configuration periodic monthly
      subscribe-to-alert-group telemetry periodic daily
    Cryptochecksum:d06974501eb0327a5ed229c8445f4fe1
    ASA 2
    ASA Version 9.0(1)
    hostname Payback-CIX
    enable password HSMurh79NVmatjY0 encrypted
    passwd 2KFQnbNIdI.2KYOU encrypted
    names
    interface Ethernet0/0
    switchport access vlan 2
    speed 100
    duplex full
    interface Ethernet0/1
    description This port connects to VLAN 100
    switchport access vlan 100
    interface Ethernet0/2
    interface Ethernet0/3
    switchport access vlan 100
    interface Ethernet0/4
    switchport access vlan 100
    interface Ethernet0/5
    switchport access vlan 100
    interface Ethernet0/6
    switchport access vlan 100
    interface Ethernet0/7
    switchport access vlan 100
    interface Vlan2
    nameif outside
    security-level 0
    ip address 84.39.233.50 255.255.255.240
    interface Vlan100
    nameif inside
    security-level 100
    ip address 192.168.100.1 255.255.255.0
    banner login line Welcome to Payback Loyalty - CIX
    ftp mode passive
    clock summer-time gmt/idt recurring last Sun Mar 1:00 last Sun Oct 2:00
    dns domain-lookup outside
    dns domain-lookup inside
    dns server-group defaultDNS
    name-server 8.8.8.8
    name-server 8.8.4.4
    same-security-traffic permit inter-interface
    object network obj_any
    subnet 0.0.0.0 0.0.0.0
    object network CIX-Host-1
    host 192.168.100.2
    description This is the host machine of the VM servers
    object network External_CIX-Host-1
    host 84.39.233.51
    description This is the external IP address of the host server for the VM server
    object service RDP
    service tcp source range 1 65535 destination eq 3389
    object network Payback_Office
    host 92.51.193.158
    object service MSQL
    service tcp destination eq 1433
    object network Development_OLTP
    host 192.168.100.10
    description VM for Eiresoft
    object network External_Development_OLTP
    host 84.39.233.52
    description This is the external IP address for the VM for Eiresoft
    object network Eiresoft
    host 146.66.160.70
    description DBA Contractor
    object network External_TMC_Web
    host 84.39.233.53
    description Public Address of TMC Webserver
    object network TMC_Webserver
    host 192.168.100.19
    description Internal Address of TMC Webserver
    object network External_TMC_OLTP
    host 84.39.233.54
    description Targets OLTP external IP
    object network TMC_OLTP
    host 192.168.100.18
    description Targets interal IP address
    object network External_OLTP_Failover
    host 84.39.233.55
    description Public IP of OLTP Failover
    object network OLTP_Failover
    host 192.168.100.60
    description Server for OLTP failover
    object network Servers
    subnet 192.168.20.0 255.255.255.0
    object network Wired
    subnet 192.168.10.0 255.255.255.0
    object network Wireless
    subnet 192.168.40.0 255.255.255.0
    object network NETWORK_OBJ_192.168.100.0_24
    subnet 192.168.100.0 255.255.255.0
    object network NETWORK_OBJ_192.168.10.0_24
    subnet 192.168.10.0 255.255.255.0
    object network Eiresoft_2nd
    host 137.117.217.29
    description Eiresoft 2nd IP
    object network Dev_Test_Webserver
    host 192.168.100.12
    description Dev Test Webserver Internal Address
    object network External_Dev_Test_Webserver
    host 84.39.233.56
    description This is the PB Dev Test Webserver
    object network NETWORK_OBJ_192.168.1.0_24
    subnet 192.168.1.0 255.255.255.0
    object-group service DM_INLINE_SERVICE_1
    service-object object MSQL
    service-object object RDP
    object-group service DM_INLINE_SERVICE_2
    service-object object MSQL
    service-object object RDP
    object-group service DM_INLINE_SERVICE_3
    service-object object MSQL
    service-object object RDP
    object-group service DM_INLINE_SERVICE_4
    service-object object MSQL
    service-object object RDP
    service-object tcp destination eq ftp
    object-group service DM_INLINE_SERVICE_5
    service-object object MSQL
    service-object object RDP
    service-object tcp destination eq ftp
    object-group service DM_INLINE_SERVICE_6
    service-object object MSQL
    service-object object RDP
    object-group network Payback_Intrernal
    network-object object Servers
    network-object object Wired
    network-object object Wireless
    object-group service DM_INLINE_SERVICE_7
    service-object object MSQL
    service-object object RDP
    object-group service DM_INLINE_SERVICE_8
    service-object object MSQL
    service-object object RDP
    object-group service DM_INLINE_SERVICE_9
    service-object object MSQL
    service-object object RDP
    object-group service DM_INLINE_SERVICE_10
    service-object object MSQL
    service-object object RDP
    service-object tcp destination eq ftp
    object-group service DM_INLINE_SERVICE_11
    service-object object RDP
    service-object tcp destination eq ftp
    access-list outside_access_in extended permit object-group DM_INLINE_SERVICE_1 object Payback_Office object CIX-Host-1
    access-list outside_access_in remark Development OLTP from Payback Office
    access-list outside_access_in extended permit object-group DM_INLINE_SERVICE_2 object Payback_Office object Development_OLTP
    access-list outside_access_in remark Access for Eiresoft
    access-list outside_access_in extended permit object-group DM_INLINE_SERVICE_3 object Eiresoft object Development_OLTP
    access-list outside_access_in extended permit object-group DM_INLINE_SERVICE_4 object Payback_Office object TMC_Webserver
    access-list outside_access_in remark Access to OLTP for target from Payback Office
    access-list outside_access_in extended permit object-group DM_INLINE_SERVICE_5 object Payback_Office object TMC_OLTP
    access-list outside_access_in extended permit object-group DM_INLINE_SERVICE_6 object Payback_Office object OLTP_Failover
    access-list outside_access_in remark This is allowing access from Eiresoft to the OLTP Failover server
    access-list outside_access_in extended permit object-group DM_INLINE_SERVICE_7 object Eiresoft object OLTP_Failover
    access-list outside_access_in remark Access for the 2nd IP from Eiresoft
    access-list outside_access_in extended permit object-group DM_INLINE_SERVICE_8 object Eiresoft_2nd object Development_OLTP
    access-list outside_access_in remark Access from the 2nd Eiresoft IP
    access-list outside_access_in extended permit object-group DM_INLINE_SERVICE_9 object Eiresoft_2nd object OLTP_Failover
    access-list outside_access_in extended permit object-group DM_INLINE_SERVICE_10 object Payback_Office object Dev_Test_Webserver
    access-list outside_access_in extended permit object-group DM_INLINE_SERVICE_11 object Payback_Office object External_TMC_OLTP
    access-list outside_cryptomap extended permit ip 192.168.100.0 255.255.255.0 192.168.10.0 255.255.255.0
    pager lines 24
    logging enable
    logging asdm informational
    mtu outside 1500
    mtu inside 1500
    icmp unreachable rate-limit 1 burst-size 1
    no asdm history enable
    arp timeout 14400
    no arp permit-nonconnected
    nat (inside,outside) source dynamic any interface
    nat (inside,outside) source static CIX-Host-1 External_CIX-Host-1
    nat (inside,outside) source static Development_OLTP External_Development_OLTP
    nat (inside,outside) source static TMC_Webserver External_TMC_Web
    nat (inside,outside) source static TMC_OLTP External_TMC_OLTP
    nat (inside,outside) source static OLTP_Failover External_OLTP_Failover
    nat (inside,outside) source static Dev_Test_Webserver External_Dev_Test_Webserver
    nat (inside,outside) source static NETWORK_OBJ_192.168.100.0_24 NETWORK_OBJ_192.168.100.0_24 destination static NETWORK_OBJ_192.168.10.0_24 NETWORK_OBJ_192.168.10.0_24 no-proxy-arp route-lookup
    nat (inside,outside) source static NETWORK_OBJ_192.168.100.0_24 NETWORK_OBJ_192.168.100.0_24 destination static NETWORK_OBJ_192.168.1.0_24 NETWORK_OBJ_192.168.1.0_24 no-proxy-arp route-lookup
    access-group outside_access_in in interface outside
    route outside 0.0.0.0 0.0.0.0 84.39.233.49 1
    timeout xlate 3:00:00
    timeout pat-xlate 0:00:30
    timeout conn 1:00:00 half-closed 0:10:00 udp 0:02:00 icmp 0:00:02
    timeout sunrpc 0:10:00 h323 0:05:00 h225 1:00:00 mgcp 0:05:00 mgcp-pat 0:05:00
    timeout sip 0:30:00 sip_media 0:02:00 sip-invite 0:03:00 sip-disconnect 0:02:00
    timeout sip-provisional-media 0:02:00 uauth 0:05:00 absolute
    timeout tcp-proxy-reassembly 0:01:00
    timeout floating-conn 0:00:00
    dynamic-access-policy-record DfltAccessPolicy
    user-identity default-domain LOCAL
    aaa authentication ssh console LOCAL
    http server enable
    http 92.51.193.156 255.255.255.252 outside
    no snmp-server location
    no snmp-server contact
    snmp-server enable traps snmp authentication linkup linkdown coldstart warmstart
    crypto ipsec ikev1 transform-set ESP-AES-128-SHA esp-aes esp-sha-hmac
    crypto ipsec ikev1 transform-set ESP-AES-128-MD5 esp-aes esp-md5-hmac
    crypto ipsec ikev1 transform-set ESP-AES-192-SHA esp-aes-192 esp-sha-hmac
    crypto ipsec ikev1 transform-set ESP-AES-192-MD5 esp-aes-192 esp-md5-hmac
    crypto ipsec ikev1 transform-set ESP-AES-256-SHA esp-aes-256 esp-sha-hmac
    crypto ipsec ikev1 transform-set ESP-AES-256-MD5 esp-aes-256 esp-md5-hmac
    crypto ipsec ikev1 transform-set ESP-AES-128-SHA-TRANS esp-aes esp-sha-hmac
    crypto ipsec ikev1 transform-set ESP-AES-128-SHA-TRANS mode transport
    crypto ipsec ikev1 transform-set ESP-AES-128-MD5-TRANS esp-aes esp-md5-hmac
    crypto ipsec ikev1 transform-set ESP-AES-128-MD5-TRANS mode transport
    crypto ipsec ikev1 transform-set ESP-AES-192-SHA-TRANS esp-aes-192 esp-sha-hmac
    crypto ipsec ikev1 transform-set ESP-AES-192-SHA-TRANS mode transport
    crypto ipsec ikev1 transform-set ESP-AES-192-MD5-TRANS esp-aes-192 esp-md5-hmac
    crypto ipsec ikev1 transform-set ESP-AES-192-MD5-TRANS mode transport
    crypto ipsec ikev1 transform-set ESP-AES-256-SHA-TRANS esp-aes-256 esp-sha-hmac
    crypto ipsec ikev1 transform-set ESP-AES-256-SHA-TRANS mode transport
    crypto ipsec ikev1 transform-set ESP-AES-256-MD5-TRANS esp-aes-256 esp-md5-hmac
    crypto ipsec ikev1 transform-set ESP-AES-256-MD5-TRANS mode transport
    crypto ipsec ikev1 transform-set ESP-3DES-SHA esp-3des esp-sha-hmac
    crypto ipsec ikev1 transform-set ESP-3DES-MD5 esp-3des esp-md5-hmac
    crypto ipsec ikev1 transform-set ESP-3DES-SHA-TRANS esp-3des esp-sha-hmac
    crypto ipsec ikev1 transform-set ESP-3DES-SHA-TRANS mode transport
    crypto ipsec ikev1 transform-set ESP-3DES-MD5-TRANS esp-3des esp-md5-hmac
    crypto ipsec ikev1 transform-set ESP-3DES-MD5-TRANS mode transport
    crypto ipsec ikev1 transform-set ESP-DES-SHA esp-des esp-sha-hmac
    crypto ipsec ikev1 transform-set ESP-DES-MD5 esp-des esp-md5-hmac
    crypto ipsec ikev1 transform-set ESP-DES-SHA-TRANS esp-des esp-sha-hmac
    crypto ipsec ikev1 transform-set ESP-DES-SHA-TRANS mode transport
    crypto ipsec ikev1 transform-set ESP-DES-MD5-TRANS esp-des esp-md5-hmac
    crypto ipsec ikev1 transform-set ESP-DES-MD5-TRANS mode transport
    crypto ipsec ikev2 ipsec-proposal DES
    protocol esp encryption des
    protocol esp integrity sha-1 md5
    crypto ipsec ikev2 ipsec-proposal 3DES
    protocol esp encryption 3des
    protocol esp integrity sha-1 md5
    crypto ipsec ikev2 ipsec-proposal AES
    protocol esp encryption aes
    protocol esp integrity sha-1 md5
    crypto ipsec ikev2 ipsec-proposal AES192
    protocol esp encryption aes-192
    protocol esp integrity sha-1 md5
    crypto ipsec ikev2 ipsec-proposal AES256
    protocol esp encryption aes-256
    protocol esp integrity sha-1 md5
    crypto ipsec security-association pmtu-aging infinite
    crypto map outside_map 1 match address outside_cryptomap
    crypto map outside_map 1 set pfs
    crypto map outside_map 1 set peer 92.51.193.158
    crypto map outside_map 1 set ikev1 transform-set ESP-AES-128-SHA ESP-AES-128-MD5 ESP-AES-192-SHA ESP-AES-192-MD5 ESP-AES-256-SHA ESP-AES-256-MD5 ESP-3DES-SHA ESP-3DES-MD5 ESP-DES-SHA ESP-DES-MD5
    crypto map outside_map 1 set ikev2 ipsec-proposal DES 3DES AES AES192 AES256
    crypto map outside_map interface outside
    crypto ca trustpool policy
    crypto ikev2 policy 1
    encryption aes-256
    integrity sha
    group 5 2
    prf sha
    lifetime seconds 86400
    crypto ikev2 policy 10
    encryption aes-192
    integrity sha
    group 5 2
    prf sha
    lifetime seconds 86400
    crypto ikev2 policy 20
    encryption aes
    integrity sha
    group 5 2
    prf sha
    lifetime seconds 86400
    crypto ikev2 policy 30
    encryption 3des
    integrity sha
    group 5 2
    prf sha
    lifetime seconds 86400
    crypto ikev2 policy 40
    encryption des
    integrity sha
    group 5 2
    prf sha
    lifetime seconds 86400
    crypto ikev2 enable outside
    crypto ikev1 enable outside
    crypto ikev1 policy 10
    authentication crack
    encryption aes-256
    hash sha
    group 2
    lifetime 86400
    crypto ikev1 policy 20
    authentication rsa-sig
    encryption aes-256
    hash sha
    group 2
    lifetime 86400
    crypto ikev1 policy 30
    authentication pre-share
    encryption aes-256
    hash sha
    group 2
    lifetime 86400
    crypto ikev1 policy 40
    authentication crack
    encryption aes-192
    hash sha
    group 2
    lifetime 86400
    crypto ikev1 policy 50
    authentication rsa-sig
    encryption aes-192
    hash sha
    group 2
    lifetime 86400
    crypto ikev1 policy 60
    authentication pre-share
    encryption aes-192
    hash sha
    group 2
    lifetime 86400
    crypto ikev1 policy 70
    authentication crack
    encryption aes
    hash sha
    group 2
    lifetime 86400
    crypto ikev1 policy 80
    authentication rsa-sig
    encryption aes
    hash sha
    group 2
    lifetime 86400
    crypto ikev1 policy 90
    authentication pre-share
    encryption aes
    hash sha
    group 2
    lifetime 86400
    crypto ikev1 policy 100
    authentication crack
    encryption 3des
    hash sha
    group 2
    lifetime 86400
    crypto ikev1 policy 110
    authentication rsa-sig
    encryption 3des
    hash sha
    group 2
    lifetime 86400
    crypto ikev1 policy 120
    authentication pre-share
    encryption 3des
    hash sha
    group 2
    lifetime 86400
    crypto ikev1 policy 130
    authentication crack
    encryption des
    hash sha
    group 2
    lifetime 86400
    crypto ikev1 policy 140
    authentication rsa-sig
    encryption des
    hash sha
    group 2
    lifetime 86400
    crypto ikev1 policy 150
    authentication pre-share
    encryption des
    hash sha
    group 2
    lifetime 86400
    telnet timeout 5
    ssh 77.75.100.208 255.255.255.240 outside
    ssh 92.51.193.156 255.255.255.252 outside
    ssh timeout 5
    console timeout 0
    dhcpd auto_config outside
    threat-detection basic-threat
    threat-detection statistics access-list
    no threat-detection statistics tcp-intercept
    group-policy GroupPolicy_92.51.193.158 internal
    group-policy GroupPolicy_92.51.193.158 attributes
    vpn-tunnel-protocol ikev1 ikev2
    username gordon password 6e6Djaz3W/XH59zX encrypted privilege 15
    tunnel-group 92.51.193.158 type ipsec-l2l
    tunnel-group 92.51.193.158 general-attributes
    default-group-policy GroupPolicy_92.51.193.158
    tunnel-group 92.51.193.158 ipsec-attributes
    ikev1 pre-shared-key *****
    ikev2 remote-authentication pre-shared-key *****
    ikev2 local-authentication pre-shared-key *****
    class-map inspection_default
    match default-inspection-traffic
    policy-map type inspect dns preset_dns_map
    parameters
      message-length maximum client auto
      message-length maximum 512
    policy-map global_policy
    class inspection_default
      inspect dns preset_dns_map
      inspect ftp
      inspect h323 h225
      inspect h323 ras
      inspect rsh
      inspect rtsp
      inspect esmtp
      inspect sqlnet
      inspect skinny
      inspect sunrpc
      inspect xdmcp
      inspect sip
      inspect netbios
      inspect tftp
      inspect ip-options
      inspect icmp
    service-policy global_policy global
    prompt hostname context
    no call-home reporting anonymous
    Cryptochecksum:83b2069fa311e6037163ae74f9b2bec2
    : end

    Hi,
    Thanks for the help to date
    I now have the Site to Site working but there is one little issue I have. If I try to RD to a server through the tunnel it will not allow connection on the first attempt however if I ping that host and then attempt to RD it will allow the connection. It looks like the host is asleep until it receives traffic through the tunnel. Is this thje correct behaviour.
    See below the details:
    ASA1:
    hostname PAYBACK
    enable password HSMurh79NVmatjY0 encrypted
    xlate per-session deny tcp any4 any4
    xlate per-session deny tcp any4 any6
    xlate per-session deny tcp any6 any4
    xlate per-session deny tcp any6 any6
    xlate per-session deny udp any4 any4 eq domain
    xlate per-session deny udp any4 any6 eq domain
    xlate per-session deny udp any6 any4 eq domain
    xlate per-session deny udp any6 any6 eq domain
    passwd 2KFQnbNIdI.2KYOU encrypted
    names
    ip local pool VPN1 192.168.50.1-192.168.50.254 mask 255.255.255.0
    interface Ethernet0/0
    switchport access vlan 2
    speed 100
    duplex full
    interface Ethernet0/1
    description Trunk link to SW1
    switchport trunk allowed vlan 1,10,20,30,40
    switchport trunk native vlan 1
    switchport mode trunk
    interface Ethernet0/2
    interface Ethernet0/3
    interface Ethernet0/4
    interface Ethernet0/5
    interface Ethernet0/6
    interface Ethernet0/7
    interface Vlan1
    no nameif
    no security-level
    no ip address
    interface Vlan2
    nameif outside
    security-level 0
    ip address XX.XX.XX.XX 255.255.255.252
    interface Vlan10
    nameif inside
    security-level 100
    ip address 192.168.10.1 255.255.255.0
    interface Vlan20
    nameif servers
    security-level 100
    ip address 192.168.20.1 255.255.255.0
    interface Vlan30
    nameif printers
    security-level 100
    ip address 192.168.30.1 255.255.255.0
    interface Vlan40
    nameif wireless
    security-level 100
    ip address 192.168.40.1 255.255.255.0
    banner login line Welcome to Payback Loyalty Systems
    boot system disk0:/asa901-k8.bin
    ftp mode passive
    clock summer-time GMT/IDT recurring last Sun Mar 1:00 last Sun Oct 2:00
    dns domain-lookup outside
    dns domain-lookup inside
    dns domain-lookup servers
    dns domain-lookup printers
    dns domain-lookup wireless
    dns server-group DefaultDNS
    name-server 83.147.160.2
    name-server 83.147.160.130
    same-security-traffic permit inter-interface
    object network obj_any
    subnet 0.0.0.0 0.0.0.0
    object network ftp_server
    object network Internal_Report_Server
    host 192.168.20.21
    description Automated Report Server Internal Address
    object network Report_Server
    host 89.234.126.9
    description Automated Report Server
    object service RDP
    service tcp destination eq 3389
    description RDP to Server
    object network Host_QA_Server
    host 89.234.126.10
    description QA Host External Address
    object network Internal_Host_QA
    host 192.168.20.22
    description Host of VM machine for QA
    object network Internal_QA_Web_Server
    host 192.168.20.23
    description Web Server in QA environment
    object network Web_Server_QA_VM
    host 89.234.126.11
    description Web server in QA environment
    object service SQL_Server
    service tcp destination eq 1433
    object network Demo_Server
    host 89.234.126.12
    description Server set up to Demo Product
    object network Internal_Demo_Server
    host 192.168.20.24
    description Internal IP Address of Demo Server
    object network NETWORK_OBJ_192.168.20.0_24
    subnet 192.168.20.0 255.255.255.0
    object network NETWORK_OBJ_192.168.50.0_26
    subnet 192.168.50.0 255.255.255.192
    object network NETWORK_OBJ_192.168.0.0_16
    subnet 192.168.0.0 255.255.0.0
    object service MSSQL
    service tcp destination eq 1434
    description MSSQL port
    object network VPN-network
    subnet 192.168.50.0 255.255.255.0
    object network NETWORK_OBJ_192.168.50.0_24
    subnet 192.168.50.0 255.255.255.0
    object service TS
    service tcp destination eq 4400
    object service TS_Return
    service tcp source eq 4400
    object network External_QA_3
    host 89.234.126.13
    object network Internal_QA_3
    host 192.168.20.25
    object network Dev_WebServer
    host 192.168.20.27
    object network External_Dev_Web
    host 89.234.126.14
    object network NETWORK_OBJ_192.168.100.0_24
    subnet 192.168.100.0 255.255.255.0
    object network Wireless
    subnet 192.168.40.0 255.255.255.0
    description Wireless network
    object network Servers
    subnet 192.168.20.0 255.255.255.0
    object-group service DM_INLINE_SERVICE_1
    service-object tcp destination eq ftp
    service-object tcp destination eq netbios-ssn
    service-object tcp destination eq smtp
    service-object object TS
    service-object object SQL_Server
    object-group service DM_INLINE_SERVICE_3
    service-object tcp destination eq www
    service-object tcp destination eq https
    service-object object TS
    service-object object TS_Return
    object-group service DM_INLINE_SERVICE_4
    service-object object RDP
    service-object tcp destination eq www
    service-object tcp destination eq https
    object-group service DM_INLINE_SERVICE_5
    service-object object MSSQL
    service-object object RDP
    service-object object TS
    object-group protocol TCPUDP
    protocol-object udp
    protocol-object tcp
    object-group service DM_INLINE_SERVICE_6
    service-object object TS
    service-object object TS_Return
    service-object tcp destination eq www
    service-object tcp destination eq https
    object-group network DM_INLINE_NETWORK_1
    network-object 192.168.10.0 255.255.255.0
    network-object 192.168.20.0 255.255.255.0
    network-object 192.168.40.0 255.255.255.0
    object-group network Payback_Internal
    network-object 192.168.10.0 255.255.255.0
    network-object 192.168.20.0 255.255.255.0
    network-object 192.168.40.0 255.255.255.0
    access-list outside_access_in remark This rule is allowing from internet to interal server.
    access-list outside_access_in remark Allowed:
    access-list outside_access_in remark FTP
    access-list outside_access_in remark RDP
    access-list outside_access_in remark SMTP
    access-list outside_access_in remark Net Bios
    access-list outside_access_in remark SQL
    access-list outside_access_in remark TS - 4400
    access-list outside_access_in extended permit object-group DM_INLINE_SERVICE_1 any4 object Internal_Report_Server
    access-list outside_access_in remark Access rule to internal host QA
    access-list outside_access_in remark Allowed:
    access-list outside_access_in remark HTTP
    access-list outside_access_in remark RDP
    access-list outside_access_in extended permit tcp any4 object Internal_Host_QA eq www
    access-list outside_access_in remark Access to INternal Web Server:
    access-list outside_access_in remark Allowed:
    access-list outside_access_in remark HTTP
    access-list outside_access_in remark RDP
    access-list outside_access_in extended permit object-group DM_INLINE_SERVICE_3 any4 object Internal_QA_Web_Server
    access-list outside_access_in remark Rule for allowing access to Demo server
    access-list outside_access_in remark Allowed:
    access-list outside_access_in remark RDP
    access-list outside_access_in remark MSSQL
    access-list outside_access_in extended permit object-group DM_INLINE_SERVICE_4 any4 object Internal_Demo_Server
    access-list outside_access_in extended permit object-group DM_INLINE_SERVICE_5 any object Internal_QA_3
    access-list outside_access_in remark Access for Development WebServer
    access-list outside_access_in extended permit object-group DM_INLINE_SERVICE_6 any object Dev_WebServer
    access-list AnyConnect_Client_Local_Print extended deny ip any4 any4
    access-list AnyConnect_Client_Local_Print extended permit tcp any4 any4 eq lpd
    access-list AnyConnect_Client_Local_Print remark IPP: Internet Printing Protocol
    access-list AnyConnect_Client_Local_Print extended permit tcp any4 any4 eq 631
    access-list AnyConnect_Client_Local_Print remark Windows' printing port
    access-list AnyConnect_Client_Local_Print extended permit tcp any4 any4 eq 9100
    access-list AnyConnect_Client_Local_Print remark mDNS: multicast DNS protocol
    access-list AnyConnect_Client_Local_Print extended permit udp any4 host 224.0.0.251 eq 5353
    access-list AnyConnect_Client_Local_Print remark LLMNR: Link Local Multicast Name Resolution protocol
    access-list AnyConnect_Client_Local_Print extended permit udp any4 host 224.0.0.252 eq 5355
    access-list AnyConnect_Client_Local_Print remark TCP/NetBIOS protocol
    access-list AnyConnect_Client_Local_Print extended permit tcp any4 any4 eq 137
    access-list AnyConnect_Client_Local_Print extended permit udp any4 any4 eq netbios-ns
    access-list Payback_VPN_splitTunnelAcl standard permit 192.168.20.0 255.255.255.0
    access-list outside_cryptomap extended permit ip object-group DM_INLINE_NETWORK_1 192.168.100.0 255.255.255.0
    pager lines 24
    logging enable
    logging console informational
    logging asdm informational
    logging from-address [email protected]
    logging recipient-address [email protected] level alerts
    mtu outside 1500
    mtu inside 1500
    mtu servers 1500
    mtu printers 1500
    mtu wireless 1500
    no failover
    icmp unreachable rate-limit 1 burst-size 1
    asdm image disk0:/asdm-711-52.bin
    no asdm history enable
    arp timeout 14400
    no arp permit-nonconnected
    nat (inside,outside) source static DM_INLINE_NETWORK_1 DM_INLINE_NETWORK_1 destination static NETWORK_OBJ_192.168.100.0_24 NETWORK_OBJ_192.168.100.0_24 no-proxy-arp route-lookup
    nat (wireless,outside) source static Wireless Wireless destination static NETWORK_OBJ_192.168.100.0_24 NETWORK_OBJ_192.168.100.0_24 no-proxy-arp route-lookup
    nat (servers,outside) source static Servers Servers destination static NETWORK_OBJ_192.168.100.0_24 NETWORK_OBJ_192.168.100.0_24 no-proxy-arp route-lookup
    nat (inside,outside) source dynamic any interface
    nat (wireless,outside) source dynamic any interface
    nat (servers,outside) source dynamic any interface
    nat (servers,outside) source static Internal_Report_Server Report_Server
    nat (servers,outside) source static Internal_Host_QA Host_QA_Server
    nat (servers,outside) source static Internal_QA_Web_Server Web_Server_QA_VM
    nat (servers,outside) source static Internal_Demo_Server Demo_Server
    nat (servers,outside) source static NETWORK_OBJ_192.168.20.0_24 NETWORK_OBJ_192.168.20.0_24 destination static NETWORK_OBJ_192.168.50.0_24 NETWORK_OBJ_192.168.50.0_24 no-proxy-arp route-lookup
    nat (servers,outside) source static Internal_QA_3 External_QA_3
    nat (servers,outside) source static Dev_WebServer External_Dev_Web
    access-group outside_access_in in interface outside
    route outside 0.0.0.0 0.0.0.0 92.51.193.157 1
    timeout xlate 3:00:00
    timeout pat-xlate 0:00:30
    timeout conn 1:00:00 half-closed 0:10:00 udp 0:02:00 icmp 0:00:02
    timeout sunrpc 0:10:00 h323 0:05:00 h225 1:00:00 mgcp 0:05:00 mgcp-pat 0:05:00
    timeout sip 0:30:00 sip_media 0:02:00 sip-invite 0:03:00 sip-disconnect 0:02:00
    timeout sip-provisional-media 0:02:00 uauth 0:05:00 absolute
    timeout tcp-proxy-reassembly 0:01:00
    timeout floating-conn 0:00:00
    dynamic-access-policy-record DfltAccessPolicy
    user-identity default-domain LOCAL
    aaa authentication ssh console LOCAL
    http server enable
    http 192.168.10.0 255.255.255.0 inside
    no snmp-server location
    no snmp-server contact
    snmp-server enable traps snmp authentication linkup linkdown coldstart
    crypto ipsec ikev1 transform-set ESP-AES-256-MD5 esp-aes-256 esp-md5-hmac
    crypto ipsec ikev1 transform-set ESP-DES-SHA esp-des esp-sha-hmac
    crypto ipsec ikev1 transform-set ESP-DES-MD5 esp-des esp-md5-hmac
    crypto ipsec ikev1 transform-set ESP-AES-192-MD5 esp-aes-192 esp-md5-hmac
    crypto ipsec ikev1 transform-set ESP-3DES-MD5 esp-3des esp-md5-hmac
    crypto ipsec ikev1 transform-set ESP-AES-256-SHA esp-aes-256 esp-sha-hmac
    crypto ipsec ikev1 transform-set ESP-AES-128-SHA esp-aes esp-sha-hmac
    crypto ipsec ikev1 transform-set ESP-AES-192-SHA esp-aes-192 esp-sha-hmac
    crypto ipsec ikev1 transform-set ESP-AES-128-MD5 esp-aes esp-md5-hmac
    crypto ipsec ikev1 transform-set ESP-3DES-SHA esp-3des esp-sha-hmac
    crypto ipsec ikev1 transform-set ESP-AES-128-SHA-TRANS esp-aes esp-sha-hmac
    crypto ipsec ikev1 transform-set ESP-AES-128-SHA-TRANS mode transport
    crypto ipsec ikev1 transform-set ESP-AES-128-MD5-TRANS esp-aes esp-md5-hmac
    crypto ipsec ikev1 transform-set ESP-AES-128-MD5-TRANS mode transport
    crypto ipsec ikev1 transform-set ESP-AES-192-SHA-TRANS esp-aes-192 esp-sha-hmac
    crypto ipsec ikev1 transform-set ESP-AES-192-SHA-TRANS mode transport
    crypto ipsec ikev1 transform-set ESP-AES-192-MD5-TRANS esp-aes-192 esp-md5-hmac
    crypto ipsec ikev1 transform-set ESP-AES-192-MD5-TRANS mode transport
    crypto ipsec ikev1 transform-set ESP-AES-256-SHA-TRANS esp-aes-256 esp-sha-hmac
    crypto ipsec ikev1 transform-set ESP-AES-256-SHA-TRANS mode transport
    crypto ipsec ikev1 transform-set ESP-AES-256-MD5-TRANS esp-aes-256 esp-md5-hmac
    crypto ipsec ikev1 transform-set ESP-AES-256-MD5-TRANS mode transport
    crypto ipsec ikev1 transform-set ESP-3DES-SHA-TRANS esp-3des esp-sha-hmac
    crypto ipsec ikev1 transform-set ESP-3DES-SHA-TRANS mode transport
    crypto ipsec ikev1 transform-set ESP-3DES-MD5-TRANS esp-3des esp-md5-hmac
    crypto ipsec ikev1 transform-set ESP-3DES-MD5-TRANS mode transport
    crypto ipsec ikev1 transform-set ESP-DES-SHA-TRANS esp-des esp-sha-hmac
    crypto ipsec ikev1 transform-set ESP-DES-SHA-TRANS mode transport
    crypto ipsec ikev1 transform-set ESP-DES-MD5-TRANS esp-des esp-md5-hmac
    crypto ipsec ikev1 transform-set ESP-DES-MD5-TRANS mode transport
    crypto ipsec ikev2 ipsec-proposal DES
    protocol esp encryption des
    protocol esp integrity sha-1 md5
    crypto ipsec ikev2 ipsec-proposal 3DES
    protocol esp encryption 3des
    protocol esp integrity sha-1 md5
    crypto ipsec ikev2 ipsec-proposal AES
    protocol esp encryption aes
    protocol esp integrity sha-1 md5
    crypto ipsec ikev2 ipsec-proposal AES192
    protocol esp encryption aes-192
    protocol esp integrity sha-1 md5
    crypto ipsec ikev2 ipsec-proposal AES256
    protocol esp encryption aes-256
    protocol esp integrity sha-1 md5
    crypto ipsec security-association pmtu-aging infinite
    crypto dynamic-map SYSTEM_DEFAULT_CRYPTO_MAP 65535 set pfs group1
    crypto dynamic-map SYSTEM_DEFAULT_CRYPTO_MAP 65535 set ikev1 transform-set ESP-AES-128-SHA ESP-AES-128-MD5 ESP-AES-192-SHA ESP-AES-192-MD5 ESP-AES-256-SHA ESP-AES-256-MD5 ESP-3DES-SHA ESP-3DES-MD5 ESP-DES-SHA ESP-DES-MD5
    crypto dynamic-map SYSTEM_DEFAULT_CRYPTO_MAP 65535 set ikev2 ipsec-proposal AES256 AES192 AES 3DES DES
    crypto map outside_map 1 match address outside_cryptomap
    crypto map outside_map 1 set pfs
    crypto map outside_map 1 set peer XX.XX.XX.XX
    crypto map outside_map 1 set ikev1 transform-set ESP-AES-128-SHA ESP-AES-128-MD5 ESP-AES-192-SHA ESP-AES-192-MD5 ESP-AES-256-SHA ESP-AES-256-MD5 ESP-3DES-SHA ESP-3DES-MD5 ESP-DES-SHA ESP-DES-MD5
    crypto map outside_map 1 set ikev2 ipsec-proposal DES 3DES AES AES192 AES256
    crypto map outside_map 65535 ipsec-isakmp dynamic SYSTEM_DEFAULT_CRYPTO_MAP
    crypto map outside_map interface outside
    crypto map inside_map 65535 ipsec-isakmp dynamic SYSTEM_DEFAULT_CRYPTO_MAP
    crypto map inside_map interface inside
    crypto map servers_map 65535 ipsec-isakmp dynamic SYSTEM_DEFAULT_CRYPTO_MAP
    crypto map servers_map interface servers
    crypto ca trustpool policy
    crypto ikev2 policy 1
    encryption aes-256
    integrity sha
    group 5
    prf sha
    lifetime seconds 86400
    crypto ikev2 policy 10
    encryption aes-192
    integrity sha
    group 5
    prf sha
    lifetime seconds 86400
    crypto ikev2 policy 20
    encryption aes
    integrity sha
    group 5
    prf sha
    lifetime seconds 86400
    crypto ikev2 policy 30
    encryption 3des
    integrity sha
    group 5
    prf sha
    lifetime seconds 86400
    crypto ikev2 policy 40
    encryption des
    integrity sha
    group 5
    prf sha
    lifetime seconds 86400
    crypto ikev2 enable outside client-services port 443
    crypto ikev2 enable inside client-services port 443
    crypto ikev1 enable outside
    crypto ikev1 enable inside
    crypto ikev1 enable servers
    crypto ikev1 policy 10
    authentication pre-share
    encryption 3des
    hash sha
    group 2
    lifetime 86400
    telnet timeout 5
    ssh 192.168.10.0 255.255.255.0 inside
    ssh timeout 5
    console timeout 0
    dhcpd dns 192.168.0.1
    dhcpd auto_config outside
    dhcpd address 192.168.10.21-192.168.10.240 inside
    dhcpd dns 192.168.20.21 83.147.160.2 interface inside
    dhcpd option 15 ascii paybackloyalty.com interface inside
    dhcpd enable inside
    dhcpd address 192.168.40.21-192.168.40.240 wireless
    dhcpd dns 192.168.20.21 83.147.160.2 interface wireless
    dhcpd update dns interface wireless
    dhcpd option 15 ascii paybackloyalty.com interface wireless
    dhcpd enable wireless
    threat-detection basic-threat
    threat-detection statistics access-list
    no threat-detection statistics tcp-intercept
    group-policy Payback_VPN internal
    group-policy Payback_VPN attributes
    vpn-simultaneous-logins 10
    vpn-tunnel-protocol ikev1
    split-tunnel-policy tunnelspecified
    split-tunnel-network-list value Payback_VPN_splitTunnelAcl
    group-policy DfltGrpPolicy attributes
    dns-server value 83.147.160.2 83.147.160.130
    vpn-tunnel-protocol ikev1 ikev2 ssl-clientless
    group-policy GroupPolicy_84.39.233.50 internal
    group-policy GroupPolicy_84.39.233.50 attributes
    vpn-tunnel-protocol ikev1 ikev2
    username Noelle password XB/IpvYaATP.2QYm encrypted
    username Noelle attributes
    vpn-group-policy Payback_VPN
    service-type remote-access
    username Eanna password vXILR9ZZQIsd1Naw encrypted privilege 0
    username Eanna attributes
    vpn-group-policy Payback_VPN
    service-type remote-access
    username Michael password qpbleUqUEchRrgQX encrypted
    username Michael attributes
    vpn-group-policy Payback_VPN
    service-type remote-access
    username Danny password .7fEXdzESUk6S/cC encrypted privilege 0
    username Danny attributes
    vpn-group-policy Payback_VPN
    service-type remote-access
    username niamh password MlFlIlEiy8vismE0 encrypted
    username niamh attributes
    service-type admin
    username Aileen password tytrelqvV5VRX2pz encrypted privilege 0
    username Aileen attributes
    vpn-group-policy Payback_VPN
    service-type remote-access
    username Aidan password aDu6YH0V5XaxpEPg encrypted privilege 0
    username Aidan attributes
    vpn-group-policy Payback_VPN
    service-type remote-access
    username gordon password 6e6Djaz3W/XH59zX encrypted privilege 15
    username shane.c password iqGMoWOnfO6YKXbw encrypted
    username shane.c attributes
    vpn-group-policy Payback_VPN
    service-type remote-access
    username Shane password yQeVtvLLKqapoUje encrypted privilege 0
    username Shane attributes
    vpn-group-policy Payback_VPN
    service-type remote-access
    username James password TdYPv1pvld/hPM0d encrypted
    username James attributes
    vpn-group-policy Payback_VPN
    service-type remote-access
    username mark password yruxpddqfyNb.qFn encrypted
    username mark attributes
    service-type admin
    username Mary password XND5FTEiyu1L1zFD encrypted
    username Mary attributes
    vpn-group-policy Payback_VPN
    service-type remote-access
    username Massimo password vs65MMo4rM0l4rVu encrypted privilege 0
    username Massimo attributes
    vpn-group-policy Payback_VPN
    service-type remote-access
    tunnel-group Payback_VPN type remote-access
    tunnel-group Payback_VPN general-attributes
    address-pool VPN1
    default-group-policy Payback_VPN
    tunnel-group Payback_VPN ipsec-attributes
    ikev1 pre-shared-key *****
    tunnel-group 84.39.233.50 type ipsec-l2l
    tunnel-group 84.39.233.50 general-attributes
    default-group-policy GroupPolicy_84.39.233.50
    tunnel-group 84.39.233.50 ipsec-attributes
    ikev1 pre-shared-key *****
    ikev2 remote-authentication pre-shared-key *****
    ikev2 local-authentication pre-shared-key *****
    class-map global-class
    match default-inspection-traffic
    policy-map global-policy
    class global-class
      inspect dns
      inspect ftp
      inspect h323 h225
      inspect h323 ras
      inspect ip-options
      inspect netbios
      inspect pptp
      inspect rsh
      inspect rtsp
      inspect sip
      inspect snmp
      inspect sqlnet
      inspect sunrpc
      inspect tftp
      inspect xdmcp
      inspect icmp error
      inspect icmp
    service-policy global-policy global
    smtp-server 192.168.20.21
    prompt hostname context
    no call-home reporting anonymous
    call-home
    profile CiscoTAC-1
      no active
      destination address http https://tools.cisco.com/its/service/oddce/services/DDCEService
      destination address email [email protected]
      destination transport-method http
      subscribe-to-alert-group diagnostic
      subscribe-to-alert-group environment
      subscribe-to-alert-group inventory periodic monthly
      subscribe-to-alert-group configuration periodic monthly
      subscribe-to-alert-group telemetry periodic daily
    Cryptochecksum:83fa7ce1d93375645205f6e79b526381
    ASA2:
    ASA Version 9.0(1)
    hostname Payback-CIX
    enable password HSMurh79NVmatjY0 encrypted
    passwd 2KFQnbNIdI.2KYOU encrypted
    names
    interface Ethernet0/0
    switchport access vlan 2
    speed 100
    duplex full
    interface Ethernet0/1
    description This port connects to VLAN 100
    switchport access vlan 100
    interface Ethernet0/2
    interface Ethernet0/3
    switchport access vlan 100
    interface Ethernet0/4
    switchport access vlan 100
    interface Ethernet0/5
    switchport access vlan 100
    interface Ethernet0/6
    switchport access vlan 100
    interface Ethernet0/7
    switchport access vlan 100
    interface Vlan2
    nameif outside
    security-level 0
    ip address X.X.X.X 255.255.255.240
    interface Vlan100
    nameif inside
    security-level 100
    ip address 192.168.100.1 255.255.255.0
    banner login line Welcome to Payback Loyalty - CIX
    ftp mode passive
    clock timezone GMT 0
    clock summer-time gmt/idt recurring last Sun Mar 1:00 last Sun Oct 2:00
    dns domain-lookup outside
    dns domain-lookup inside
    dns server-group defaultDNS
    name-server 8.8.8.8
    name-server 8.8.4.4
    same-security-traffic permit inter-interface
    object network obj_any
    subnet 0.0.0.0 0.0.0.0
    object network CIX-Host-1
    host 192.168.100.2
    description This is the host machine of the VM servers
    object network External_CIX-Host-1
    host 84.39.233.51
    description This is the external IP address of the host server for the VM server
    object service RDP
    service tcp source range 1 65535 destination eq 3389
    object network Payback_Office
    host 92.51.193.158
    object service MSQL
    service tcp destination eq 1433
    object network Development_OLTP
    host 192.168.100.10
    description VM for Eiresoft
    object network External_Development_OLTP
    host 84.39.233.52
    description This is the external IP address for the VM for Eiresoft
    object network External_TMC_Web
    host 84.39.233.53
    description Public Address of TMC Webserver
    object network TMC_Webserver
    host 192.168.100.19
    description Internal Address of TMC Webserver
    object network External_TMC_OLTP
    host 84.39.233.54
    description Targets OLTP external IP
    object network TMC_OLTP
    host 192.168.100.18
    description Targets interal IP address
    object network External_OLTP_Failover
    host 84.39.233.55
    description Public IP of OLTP Failover
    object network OLTP_Failover
    host 192.168.100.60
    description Server for OLTP failover
    object network Servers
    subnet 192.168.20.0 255.255.255.0
    object network Wired
    subnet 192.168.10.0 255.255.255.0
    object network Wireless
    subnet 192.168.40.0 255.255.255.0
    object network NETWORK_OBJ_192.168.100.0_24
    subnet 192.168.100.0 255.255.255.0
    object network NETWORK_OBJ_192.168.10.0_24
    subnet 192.168.10.0 255.255.255.0
    object network Eiresoft_2nd
    host 137.117.217.29
    description Eiresoft 2nd IP
    object network Dev_Test_Webserver
    host 192.168.100.12
    description Dev Test Webserver Internal Address
    object network External_Dev_Test_Webserver
    host 84.39.233.56
    description This is the PB Dev Test Webserver
    object network NETWORK_OBJ_192.168.1.0_24
    subnet 192.168.1.0 255.255.255.0
    object network LAN
    subnet 192.168.100.0 255.255.255.0
    object network REMOTE-LAN
    subnet 192.168.10.0 255.255.255.0
    object network TargetMC
    host 83.71.194.145
    description This is Target Location that will be accessing the Webserver
    object network Rackspace_OLTP
    host 162.13.34.56
    description This is the IP address of production OLTP
    object service DB
    service tcp destination eq 5022
    object network Topaz_Target_VM
    host 82.198.151.168
    description This is Topaz IP that will be accessing Targets VM
    object service DB_2
    service tcp destination eq 5023
    object network EireSoft_NEW_IP
    host 146.66.161.3
    description Eiresoft latest IP form ISP DHCP
    object-group service DM_INLINE_SERVICE_1
    service-object object MSQL
    service-object object RDP
    service-object icmp echo
    service-object icmp echo-reply
    object-group service DM_INLINE_SERVICE_2
    service-object object MSQL
    service-object object RDP
    object-group service DM_INLINE_SERVICE_4
    service-object object MSQL
    service-object object RDP
    service-object tcp destination eq ftp
    service-object tcp destination eq www
    object-group service DM_INLINE_SERVICE_5
    service-object object MSQL
    service-object object RDP
    service-object tcp destination eq ftp
    object-group service DM_INLINE_SERVICE_6
    service-object object MSQL
    service-object object RDP
    object-group network Payback_Intrernal
    network-object object Servers
    network-object object Wired
    network-object object Wireless
    object-group service DM_INLINE_SERVICE_8
    service-object object MSQL
    service-object object RDP
    object-group service DM_INLINE_SERVICE_9
    service-object object MSQL
    service-object object RDP
    object-group service DM_INLINE_SERVICE_10
    service-object object MSQL
    service-object object RDP
    service-object tcp destination eq ftp
    service-object icmp echo
    service-object icmp echo-reply
    service-object object DB
    object-group service DM_INLINE_SERVICE_11
    service-object object RDP
    service-object tcp destination eq ftp
    object-group service DM_INLINE_SERVICE_12
    service-object object MSQL
    service-object icmp echo
    service-object icmp echo-reply
    service-object object DB
    service-object object DB_2
    object-group service DM_INLINE_SERVICE_13
    service-object object MSQL
    service-object object RDP
    object-group service DM_INLINE_SERVICE_14
    service-object object MSQL
    service-object object RDP
    access-list outside_access_in extended permit object-group DM_INLINE_SERVICE_1 object Payback_Office object CIX-Host-1
    access-list outside_access_in remark Development OLTP from Payback Office
    access-list outside_access_in extended permit object-group DM_INLINE_SERVICE_2 object Payback_Office object Development_OLTP
    access-list outside_access_in extended permit object-group DM_INLINE_SERVICE_4 object Payback_Office object TMC_Webserver
    access-list outside_access_in remark Access to OLTP for target from Payback Office
    access-list outside_access_in extended permit object-group DM_INLINE_SERVICE_5 object Payback_Office object TMC_OLTP
    access-list outside_access_in extended permit object-group DM_INLINE_SERVICE_6 object Payback_Office object OLTP_Failover
    access-list outside_access_in remark Access for the 2nd IP from Eiresoft
    access-list outside_access_in extended permit object-group DM_INLINE_SERVICE_8 object Eiresoft_2nd object Development_OLTP
    access-list outside_access_in remark Access from the 2nd Eiresoft IP
    access-list outside_access_in extended permit object-group DM_INLINE_SERVICE_9 object Eiresoft_2nd object OLTP_Failover
    access-list outside_access_in extended permit object-group DM_INLINE_SERVICE_10 object Payback_Office object Dev_Test_Webserver
    access-list outside_access_in extended permit object-group DM_INLINE_SERVICE_11 object Payback_Office object External_TMC_OLTP
    access-list outside_access_in remark Access rules from Traget to CIX for testing
    access-list outside_access_in extended permit tcp object TargetMC object TMC_Webserver eq www
    access-list outside_access_in extended permit object-group DM_INLINE_SERVICE_12 object Rackspace_OLTP object OLTP_Failover
    access-list outside_access_in remark Topaz access to Target VM
    access-list outside_access_in extended permit tcp object Topaz_Target_VM object TMC_Webserver eq www
    access-list outside_access_in remark Opened up for Target for the weekend. Closing on Monday 20th
    access-list outside_access_in extended permit tcp any object TMC_Webserver eq www
    access-list outside_access_in remark Access for Eiresoft after their ISP changed their IP Address
    access-list outside_access_in extended permit object-group DM_INLINE_SERVICE_13 object EireSoft_NEW_IP object Development_OLTP
    access-list outside_access_in remark Eiresoft Access after ISP changed their IP Address
    access-list outside_access_in extended permit object-group DM_INLINE_SERVICE_14 object EireSoft_NEW_IP object OLTP_Failover
    access-list outside_cryptomap extended permit ip 192.168.100.0 255.255.255.0 object-group Payback_Intrernal
    pager lines 24
    logging enable
    logging console debugging
    logging asdm informational
    mtu outside 1500
    mtu inside 1500
    icmp unreachable rate-limit 1 burst-size 1
    no asdm history enable
    arp timeout 14400
    no arp permit-nonconnected
    nat (inside,outside) source static NETWORK_OBJ_192.168.100.0_24 NETWORK_OBJ_192.168.100.0_24 destination static Payback_Intrernal Payback_Intrernal no-proxy-arp route-lookup
    nat (inside,outside) source static CIX-Host-1 External_CIX-Host-1
    nat (inside,outside) source static Development_OLTP External_Development_OLTP
    nat (inside,outside) source static TMC_Webserver External_TMC_Web
    nat (inside,outside) source static TMC_OLTP External_TMC_OLTP
    nat (inside,outside) source static OLTP_Failover External_OLTP_Failover
    nat (inside,outside) source static Dev_Test_Webserver External_Dev_Test_Webserver
    nat (inside,outside) source dynamic LAN interface
    access-group outside_access_in in interface outside
    route outside 0.0.0.0 0.0.0.0 84.39.233.49 1
    timeout xlate 3:00:00
    timeout pat-xlate 0:00:30
    timeout conn 1:00:00 half-closed 0:10:00 udp 0:02:00 icmp 0:00:02
    timeout sunrpc 0:10:00 h323 0:05:00 h225 1:00:00 mgcp 0:05:00 mgcp-pat 0:05:00
    timeout sip 0:30:00 sip_media 0:02:00 sip-invite 0:03:00 sip-disconnect 0:02:00
    timeout sip-provisional-media 0:02:00 uauth 0:05:00 absolute
    timeout tcp-proxy-reassembly 0:01:00
    timeout floating-conn 0:00:00
    dynamic-access-policy-record DfltAccessPolicy
    user-identity default-domain LOCAL
    aaa authentication ssh console LOCAL
    http server enable
    http X.X.X.X 255.255.255.252 outside
    no snmp-server location
    no snmp-server contact
    snmp-server enable traps snmp authentication linkup linkdown coldstart warmstart
    crypto ipsec ikev1 transform-set ESP-AES-128-SHA esp-aes esp-sha-hmac
    crypto ipsec ikev1 transform-set ESP-AES-128-MD5 esp-aes esp-md5-hmac
    crypto ipsec ikev1 transform-set ESP-AES-192-SHA esp-aes-192 esp-sha-hmac
    crypto ipsec ikev1 transform-set ESP-AES-192-MD5 esp-aes-192 esp-md5-hmac
    crypto ipsec ikev1 transform-set ESP-AES-256-SHA esp-aes-256 esp-sha-hmac
    crypto ipsec ikev1 transform-set ESP-AES-256-MD5 esp-aes-256 esp-md5-hmac
    crypto ipsec ikev1 transform-set ESP-AES-128-SHA-TRANS esp-aes esp-sha-hmac
    crypto ipsec ikev1 transform-set ESP-AES-128-SHA-TRANS mode transport
    crypto ipsec ikev1 transform-set ESP-AES-128-MD5-TRANS esp-aes esp-md5-hmac
    crypto ipsec ikev1 transform-set ESP-AES-128-MD5-TRANS mode transport
    crypto ipsec ikev1 transform-set ESP-AES-192-SHA-TRANS esp-aes-192 esp-sha-hmac
    crypto ipsec ikev1 transform-set ESP-AES-192-SHA-TRANS mode transport
    crypto ipsec ikev1 transform-set ESP-AES-192-MD5-TRANS esp-aes-192 esp-md5-hmac
    crypto ipsec ikev1 transform-set ESP-AES-192-MD5-TRANS mode transport
    crypto ipsec ikev1 transform-set ESP-AES-256-SHA-TRANS esp-aes-256 esp-sha-hmac
    crypto ipsec ikev1 transform-set ESP-AES-256-SHA-TRANS mode transport
    crypto ipsec ikev1 transform-set ESP-AES-256-MD5-TRANS esp-aes-256 esp-md5-hmac
    crypto ipsec ikev1 transform-set ESP-AES-256-MD5-TRANS mode transport
    crypto ipsec ikev1 transform-set ESP-3DES-SHA esp-3des esp-sha-hmac
    crypto ipsec ikev1 transform-set ESP-3DES-MD5 esp-3des esp-md5-hmac
    crypto ipsec ikev1 transform-set ESP-3DES-SHA-TRANS esp-3des esp-sha-hmac
    crypto ipsec ikev1 transform-set ESP-3DES-SHA-TRANS mode transport
    crypto ipsec ikev1 transform-set ESP-3DES-MD5-TRANS esp-3des esp-md5-hmac
    crypto ipsec ikev1 transform-set ESP-3DES-MD5-TRANS mode transport
    crypto ipsec ikev1 transform-set ESP-DES-SHA esp-des esp-sha-hmac
    crypto ipsec ikev1 transform-set ESP-DES-MD5 esp-des esp-md5-hmac
    crypto ipsec ikev1 transform-set ESP-DES-SHA-TRANS esp-des esp-sha-hmac
    crypto ipsec ikev1 transform-set ESP-DES-SHA-TRANS mode transport
    crypto ipsec ikev1 transform-set ESP-DES-MD5-TRANS esp-des esp-md5-hmac
    crypto ipsec ikev1 transform-set ESP-DES-MD5-TRANS mode transport
    crypto ipsec ikev2 ipsec-proposal AES256
    protocol esp encryption aes-256
    protocol esp integrity sha-1 md5
    crypto ipsec ikev2 ipsec-proposal AES192
    protocol esp encryption aes-192
    protocol esp integrity sha-1 md5
    crypto ipsec ikev2 ipsec-proposal AES
    protocol esp encryption aes
    protocol esp integrity sha-1 md5
    crypto ipsec ikev2 ipsec-proposal 3DES
    protocol esp encryption 3des
    protocol esp integrity sha-1 md5
    crypto ipsec ikev2 ipsec-proposal DES
    protocol esp encryption des
    protocol esp integrity sha-1 md5
    crypto ipsec security-association pmtu-aging infinite
    crypto map outside_map 1 match address outside_cryptomap
    crypto map outside_map 1 set pfs
    crypto map outside_map 1 set peer X.X.X.X
    crypto map outside_map 1 set ikev1 transform-set ESP-AES-128-SHA ESP-AES-128-MD5 ESP-AES-192-SHA ESP-AES-192-MD5 ESP-AES-256-SHA ESP-AES-256-MD5 ESP-3DES-SHA ESP-3DES-MD5 ESP-DES-SHA ESP-DES-MD5
    crypto map outside_map 1 set ikev2 ipsec-proposal DES 3DES AES AES192 AES256
    crypto map outside_map interface outside
    crypto ca trustpool policy
    crypto ikev2 policy 1
    encryption aes-256
    integrity sha
    group 5 2
    prf sha
    lifetime seconds 86400
    crypto ikev2 policy 10
    encryption aes-192
    integrity sha
    group 5 2
    prf sha
    lifetime seconds 86400
    crypto ikev2 policy 20
    encryption aes
    integrity sha
    group 5 2
    prf sha
    lifetime seconds 86400
    crypto ikev2 policy 30
    encryption 3des
    integrity sha
    group 5 2
    prf sha
    lifetime seconds 86400
    crypto ikev2 policy 40
    encryption des
    integrity sha
    group 5 2
    prf sha
    lifetime seconds 86400
    crypto ikev2 enable outside
    crypto ikev1 enable outside
    crypto ikev1 policy 10
    authentication crack
    encryption aes-256
    hash sha
    group 2
    lifetime 86400
    crypto ikev1 policy 20
    authentication rsa-sig
    encryption aes-256
    hash sha
    group 2
    lifetime 86400
    crypto ikev1 policy 30
    authentication pre-share
    encryption aes-256
    hash sha
    group 2
    lifetime 86400
    crypto ikev1 policy 40
    authentication crack
    encryption aes-192
    hash sha
    group 2
    lifetime 86400
    crypto ikev1 policy 50
    authentication rsa-sig
    encryption aes-192
    hash sha
    group 2
    lifetime 86400
    crypto ikev1 policy 60
    authentication pre-share
    encryption aes-192
    hash sha
    group 2
    lifetime 86400
    crypto ikev1 policy 70
    authentication crack
    encryption aes
    hash sha
    group 2
    lifetime 86400
    crypto ikev1 policy 80
    authentication rsa-sig
    encryption aes
    hash sha
    group 2
    lifetime 86400
    crypto ikev1 policy 90
    authentication pre-share
    encryption aes
    hash sha
    group 2
    lifetime 86400
    crypto ikev1 policy 100
    authentication crack
    encryption 3des
    hash sha
    group 2
    lifetime 86400
    crypto ikev1 policy 110
    authentication rsa-sig
    encryption 3des
    hash sha
    group 2
    lifetime 86400
    crypto ikev1 policy 120
    authentication pre-share
    encryption 3des
    hash sha
    group 2
    lifetime 86400
    crypto ikev1 policy 130
    authentication crack
    encryption des
    hash sha
    group 2
    lifetime 86400
    crypto ikev1 policy 140
    authentication rsa-sig
    encryption des
    hash sha
    group 2
    lifetime 86400
    crypto ikev1 policy 150
    authentication pre-share
    encryption des
    hash sha
    group 2
    lifetime 86400
    telnet timeout 5
    ssh X.X.X.X  255.255.255.240 outside
    ssh X.X.X.X 255.255.255.252 outside
    ssh 192.168.40.0 255.255.255.0 outside
    ssh timeout 5
    console timeout 0
    dhcpd auto_config outside
    threat-detection basic-threat
    threat-detection statistics access-list
    no threat-detection statistics tcp-intercept
    group-policy GroupPolicy_92.51.193.158 internal
    group-policy GroupPolicy_92.51.193.158 attributes
    vpn-tunnel-protocol ikev1 ikev2
    username gordon password 6e6Djaz3W/XH59zX encrypted privilege 15
    tunnel-group 92.51.193.158 type ipsec-l2l
    tunnel-group 92.51.193.158 general-attributes
    default-group-policy GroupPolicy_92.51.193.158
    tunnel-group 92.51.193.158 ipsec-attributes
    ikev1 pre-shared-key *****
    ikev2 remote-authentication pre-shared-key *****
    ikev2 local-authentication pre-shared-key *****
    class-map inspection_default
    match default-inspection-traffic
    policy-map type inspect dns preset_dns_map
    parameters
      message-length maximum client auto
      message-length maximum 512
    policy-map global_policy
    class inspection_default
      inspect dns preset_dns_map
      inspect ftp
      inspect h323 h225
      inspect h323 ras
      inspect rsh
      inspect rtsp
      inspect esmtp
      inspect sqlnet
      inspect skinny
      inspect sunrpc
      inspect xdmcp
      inspect sip
      inspect netbios
      inspect tftp
      inspect ip-options
      inspect icmp
    service-policy global_policy global
    prompt hostname context
    no call-home reporting anonymous
    Cryptochecksum:78a7b9ccec2fa048306092eb29a2b769

  • Cisco asa 5505 issues ( ROUTING AND PAT)

    I have some issues with my cisco asa 5505 config. Please see details below:
    NETWORK SETUP:
    gateway( 192.168.223.191)   - cisco asa 5505 ( outside - 192.168.223.200 , inside - 192.168.2.253, DMZ - 172.16.3.253 )  -
    ISSUES:
    1)
    no route from DMZ to outside
    example:
    ping from 172.16.3201 to the gateway
    6          Jan 27 2014          11:15:33                    172.16.3.201          39728                              Failed to locate egress interface for ICMP from outside:172.16.3.201/39728 to 172.16.3.253/0
    2)
    not working access from external to DMZ AT ALL
    ASA DETAILS:
    cisco asa5505
    Device license          Base
    Maximum Physical Interfaces          8          perpetual
    VLANs          3      DMZ Restricted
    Inside Hosts          Unlimited          perpetual
    configuration:
    firewall200(config)# show run
    : Saved
    ASA Version 9.1(3)
    hostname firewall200
    domain-name test1.com
    enable password xxxxxxxxxxx encrypted
    xlate per-session deny tcp any4 any4
    xlate per-session deny tcp any4 any6
    xlate per-session deny tcp any6 any4
    xlate per-session deny tcp any6 any6
    xlate per-session deny udp any4 any4 eq domain
    xlate per-session deny udp any4 any6 eq domain
    xlate per-session deny udp any6 any4 eq domain
    xlate per-session deny udp any6 any6 eq domain
    passwd XXXXXXXXXXX encrypted
    names
    interface Ethernet0/0
    switchport access vlan 100
    interface Ethernet0/1
    switchport access vlan 200
    interface Ethernet0/2
    switchport access vlan 200
    interface Ethernet0/3
    switchport access vlan 200
    interface Ethernet0/4
    switchport access vlan 300
    interface Ethernet0/5
    switchport access vlan 300
    interface Ethernet0/6
    switchport access vlan 300
    interface Ethernet0/7
    switchport access vlan 300
    interface Vlan100
    nameif outside
    security-level 0
    ip address 192.168.223.200 255.255.255.0
    interface Vlan200
    mac-address 001b.539c.597e
    nameif inside
    security-level 100
    ip address 172.16.2.253 255.255.255.0
    interface Vlan300
    no forward interface Vlan200
    nameif DMZ
    security-level 50
    ip address 172.16.3.253 255.255.255.0
    boot system disk0:/asa913-k8.bin
    boot config disk0:/startup-config.cfg
    ftp mode passive
    clock timezone GMT/BST 0
    clock summer-time GMT/BDT recurring last Sun Mar 1:00 last Sun Oct 2:00
    dns server-group DefaultDNS
    domain-name test1.com
    same-security-traffic permit inter-interface
    same-security-traffic permit intra-interface
    object network office1-int
    host 172.16.2.1
    object network firewall-dmz-gateway
    host 172.16.3.253
    object network firewall-internal-gateway
    host 172.16.2.253
    object network com1
    host 192.168.223.227
    object network web2-ext
    host 192.168.223.201
    object network web2-int
    host 172.16.3.201
    object network gateway
    host 192.168.223.191
    object network office1-int
    host 172.16.2.1
    object-group network DMZ_SUBNET
    network-object 172.16.3.0 255.255.255.0
    object-group service www tcp
    port-object eq www
    port-object eq https
    access-list DMZ_access_in extended permit icmp any any
    access-list DMZ_access_in extended permit ip any any
    access-list outside_access_in extended permit tcp any object web2-ext eq www
    pager lines 24
    logging enable
    logging asdm informational
    mtu outside 1500
    mtu inside 1500
    mtu DMZ 1500 
    icmp unreachable rate-limit 1 burst-size 1
    asdm image disk0:/asdm-714.bin
    no asdm history enable
    arp DMZ 172.16.4.199 001b.539c.597e alias
    arp DMZ 172.16.3.199 001b.539c.597e alias
    arp timeout 14400
    no arp permit-nonconnected
    object network web2-int
    nat (DMZ,outside) static web2-ext service tcp www www
    access-group outside_access_in in interface outside
    access-group DMZ_access_in in interface DMZ
    route inside 172.168.2.0 255.255.255.0 192.168.223.191 1
    route inside 172.168.3.0 255.255.255.0 192.168.223.191 1
    timeout xlate 3:00:00
    timeout pat-xlate 0:00:30
    timeout conn 1:00:00 half-closed 0:10:00 udp 0:02:00 icmp 0:00:02
    timeout sunrpc 0:10:00 h323 0:05:00 h225 1:00:00 mgcp 0:05:00 mgcp-pat 0:05:00
    timeout sip 0:30:00 sip_media 0:02:00 sip-invite 0:03:00 sip-disconnect 0:02:00
    timeout sip-provisional-media 0:02:00 uauth 0:05:00 absolute
    timeout tcp-proxy-reassembly 0:01:00
    timeout floating-conn 0:00:00
    dynamic-access-policy-record DfltAccessPolicy
    user-identity default-domain LOCAL
    aaa authentication ssh console LOCAL
    http server enable
    http 192.168.223.227 255.255.255.255 outside
    http 172.163.2.5 255.255.255.255 outside
    http 172.163.2.5 255.255.255.255 inside
    no snmp-server location
    no snmp-server contact
    snmp-server enable traps snmp authentication linkup linkdown coldstart warmstart
    crypto ipsec security-association pmtu-aging infinite
    crypto ca trustpool policy
    telnet timeout 5
    ssh 192.168.223.227 255.255.255.255 outside
    ssh 172.163.2.5 255.255.255.255 outside
    ssh 172.163.2.5 255.255.255.255 inside
    ssh timeout 60
    ssh key-exchange group dh-group1-sha1
    console timeout 0
    dhcpd address 172.16.2.10-172.16.2.10 inside
    threat-detection basic-threat
    threat-detection statistics access-list
    no threat-detection statistics tcp-intercept
    ntp server 176.58.109.199 source outside prefer
    ntp server 81.150.197.169 source outside
    ntp server 82.113.154.206
    username xxxx password xxxxxxxxx encrypted
    class-map DMZ-class
    match any
    policy-map global_policy
    policy-map DMZ-policy
    class DMZ-class
      inspect icmp
    service-policy DMZ-policy interface DMZ
    prompt hostname context
    no call-home reporting anonymous
    call-home
    profile CiscoTAC-1
      no active
      destination address http https://tools.cisco.com/its/service/oddce/services/DDCEService
      destination address email [email protected]
      destination transport-method http
      subscribe-to-alert-group diagnostic
      subscribe-to-alert-group environment
      subscribe-to-alert-group inventory periodic monthly
      subscribe-to-alert-group configuration periodic monthly
      subscribe-to-alert-group telemetry periodic daily
    Cryptochecksum:9c73fa27927822d24c75c49f09c67c24
    : end

    Thank you one more time for everthing. It is workingin indeed
    Reason why maybe sometimes I had some 'weird' results was because I had all devices connected to the same switch.Separtated all networks to a different switches helped.Anyway if you could take a look one last time to my configuration and let me know if it's good enough to deploy it on live ( only www for all , ssh restricted from outside, lan to dmz) .Thanks one more time.
    show run
    : Saved
    ASA Version 9.1(3)
    hostname firewall200
    domain-name test1.com
    enable password xxxxxxxxxx encrypted
    xlate per-session deny tcp any4 any4
    xlate per-session deny tcp any4 any6
    xlate per-session deny tcp any6 any4
    xlate per-session deny tcp any6 any6
    xlate per-session deny udp any4 any4 eq domain
    xlate per-session deny udp any4 any6 eq domain
    xlate per-session deny udp any6 any4 eq domain
    xlate per-session deny udp any6 any6 eq domain
    passwd xxxxxxxxxxxx encrypted
    names
    interface Ethernet0/0
    switchport access vlan 100
    interface Ethernet0/1
    switchport access vlan 200
    interface Ethernet0/2
    switchport access vlan 200
    interface Ethernet0/3
    switchport access vlan 200
    interface Ethernet0/4
    switchport access vlan 300
    interface Ethernet0/5
    switchport access vlan 300
    interface Ethernet0/6
    switchport access vlan 300
    interface Ethernet0/7
    switchport access vlan 300
    interface Vlan100
    nameif outside
    security-level 0
    ip address 192.168.223.200 255.255.255.0
    interface Vlan200
    mac-address 001b.539c.597e
    nameif inside
    security-level 100
    ip address 172.16.2.253 255.255.255.0
    interface Vlan300
    no forward interface Vlan200
    nameif DMZ
    security-level 50
    ip address 172.16.3.253 255.255.255.0
    boot system disk0:/asa913-k8.bin
    boot config disk0:/startup-config.cfg
    ftp mode passive
    clock timezone GMT/BST 0
    clock summer-time GMT/BDT recurring last Sun Mar 1:00 last Sun Oct 2:00
    dns domain-lookup inside
    dns domain-lookup DMZ
    dns server-group DefaultDNS
    name-server 8.8.8.8
    name-server 8.8.4.4
    domain-name test1.com
    same-security-traffic permit inter-interface
    same-security-traffic permit intra-interface
    object network firewall-dmz-gateway
    host 172.16.3.253
    object network firewall-internal-gateway
    host 172.16.2.253
    object network com1
    host 192.168.223.227
    object network web2-ext
    host 192.168.223.201
    object network web2-int
    host 172.16.3.201
    object network gateway
    host 192.168.223.191
    object network office1-int
    host 172.16.2.1
    object-group network DMZ_SUBNET
    network-object 172.16.3.0 255.255.255.0
    object-group service www tcp
    port-object eq www
    port-object eq https
    access-list DMZ_access_in extended permit icmp any any
    access-list DMZ_access_in extended permit ip any any
    access-list DMZ_access_in extended permit tcp 172.16.3.0 255.255.255.0 interface outside eq ssh
    access-list outside_access_in extended permit tcp any object web2-int eq www
    access-list outside_access_in extended permit tcp any object web2-int eq ssh
    pager lines 24
    logging enable
    logging asdm informational
    mtu outside 1500
    mtu inside 1500
    mtu DMZ 1500
    icmp unreachable rate-limit 1 burst-size 1
    icmp permit any inside
    icmp permit any DMZ
    asdm image disk0:/asdm-714.bin
    no asdm history enable
    arp DMZ 172.16.4.199 001b.539c.597e alias
    arp DMZ 172.16.3.199 001b.539c.597e alias
    arp timeout 14400
    no arp permit-nonconnected
    object network web2-int
    nat (DMZ,outside) static web2-ext net-to-net
    access-group outside_access_in in interface outside
    access-group DMZ_access_in in interface DMZ
    route outside 0.0.0.0 0.0.0.0 192.168.223.191 1
    timeout xlate 3:00:00
    timeout pat-xlate 0:00:30
    timeout conn 1:00:00 half-closed 0:10:00 udp 0:02:00 icmp 0:00:02
    timeout sunrpc 0:10:00 h323 0:05:00 h225 1:00:00 mgcp 0:05:00 mgcp-pat 0:05:00
    timeout sip 0:30:00 sip_media 0:02:00 sip-invite 0:03:00 sip-disconnect 0:02:00
    timeout sip-provisional-media 0:02:00 uauth 0:05:00 absolute
    timeout tcp-proxy-reassembly 0:01:00
    timeout floating-conn 0:00:00
    dynamic-access-policy-record DfltAccessPolicy
    user-identity default-domain LOCAL
    aaa authentication ssh console LOCAL
    http server enable
    http 192.168.223.227 255.255.255.255 outside
    http 172.163.2.5 255.255.255.255 outside
    http 172.163.2.5 255.255.255.255 inside
    no snmp-server location
    no snmp-server contact
    snmp-server enable traps snmp authentication linkup linkdown coldstart warmstart
    crypto ipsec security-association pmtu-aging infinite
    crypto ca trustpool policy
    telnet timeout 5
    ssh 192.168.223.227 255.255.255.255 outside
    ssh 172.163.2.5 255.255.255.255 outside
    ssh 172.16.3.253 255.255.255.255 outside
    ssh 172.163.2.5 255.255.255.255 inside
    ssh timeout 60
    ssh key-exchange group dh-group1-sha1
    console timeout 0
    threat-detection basic-threat
    threat-detection statistics access-list
    no threat-detection statistics tcp-intercept
    ntp server 176.58.109.199 source outside prefer
    ntp server 81.150.197.169 source outside
    ntp server 82.113.154.206
    username xxxxx password xxxxxxxxx encrypted
    class-map DMZ-class
    match any
    policy-map global_policy
    policy-map DMZ-policy
    class DMZ-class
      inspect icmp
    service-policy DMZ-policy interface DMZ
    prompt hostname context
    no call-home reporting anonymous
    call-home
    profile CiscoTAC-1
      no active
      destination address http https://tools.cisco.com/its/service/oddce/services/DDCEService
      destination address email [email protected]
      destination transport-method http
      subscribe-to-alert-group diagnostic
      subscribe-to-alert-group environment
      subscribe-to-alert-group inventory periodic monthly
      subscribe-to-alert-group configuration periodic monthly
      subscribe-to-alert-group telemetry periodic daily
    Cryptochecksum:f264c94bb8c0dd206385a6b72afe9e5b
    : end

  • Cisco asa 5505 with Router 881w Configuration Help

    Hello all,
    I'm having trouble setting up a second vlan to route to the internet. I have a Cisco ASA 5505 connected to my ISP(OUTSIDE) and a Cisco 881w (INSIDE) router in the back of my firewall. My vlan 10 with the network 192.168.5.1 255.255.255.0 works with pat, however vlan 15 that is on my 881w router does not route to the internet at all. I can only ping from 192.168.15.15 network to 192.168.5.1 I would like some advice on how can I make this set up work. Attached with this discussion is a picture of my topology.
    Thanks in advance.
    here are the show runs:
    Cisco ASA 5505 show run:
    ASA Version 8.3(1)
    names
    interface Vlan1
     no nameif
     no security-level
     no ip address
    interface Vlan5
     mac-address xxxx.xxxx.xxxx
     nameif OUTSIDE
     security-level 0
     ip address dhcp setroute
    interface Vlan10
     nameif INSIDE
     security-level 100
     ip address 192.168.5.1 255.255.255.0
    interface Ethernet0/0
     switchport access vlan 5
    interface Ethernet0/1
     switchport access vlan 10
    interface Ethernet0/2
    interface Ethernet0/3
     shutdown
    interface Ethernet0/4
     shutdown
    interface Ethernet0/5
     shutdown
    interface Ethernet0/6
     shutdown
    interface Ethernet0/7
     shutdown
    ftp mode passive
    clock timezone CST -6
    clock summer-time CDT recurring
    object network INTERNAL_LAN
     subnet 192.168.5.0 255.255.255.0
    object network PRIVATE_LAN_192
     subnet 192.168.15.0 255.255.255.224
     description PRIVATE_LAN_192
    access-list INSIDE_access_in extended permit ip any any
    access-list INSIDE_access_in extended deny ip any any
    access-list OUTSIDE_access_in extended permit ip any any
    access-list OUTSIDE_access_in extended deny ip any any
    pager lines 24
    logging enable
    mtu OUTSIDE 1500
    mtu INSIDE 1500
    ip verify reverse-path interface OUTSIDE
    ip verify reverse-path interface INSIDE
    no failover
    icmp unreachable rate-limit 1 burst-size 1
    no asdm history enable
    arp timeout 14400
    object network INTERNAL_LAN
     nat (INSIDE,OUTSIDE) dynamic interface
    object network PRIVATE_LAN_192
     nat (INSIDE,OUTSIDE) dynamic interface
    access-group OUTSIDE_access_in in interface OUTSIDE
    access-group INSIDE_access_in in interface INSIDE
    route INSIDE 192.168.15.0 255.255.255.224 192.168.5.2 1
    dynamic-access-policy-record DfltAccessPolicy
    http server enable
    dhcpd dns 8.8.8.8 75.75.76.76
    dhcpd address 192.168.5.10-192.168.5.100 INSIDE
    dhcpd enable INSIDE
    Router 881w show run:
    Current configuration : 4912 bytes
    version 12.4
    no ip source-route
    ip dhcp excluded-address 192.168.15.1 192.168.15.10
    ip dhcp pool PRIVATE_LAN
       network 192.168.15.0 255.255.255.224
    interface FastEthernet0
     switchport trunk allowed vlan 1,15,1002-1005
     switchport mode trunk
    interface FastEthernet1
    interface FastEthernet2
    interface FastEthernet3
    interface FastEthernet4
     ip address 192.168.5.2 255.255.255.0
     duplex auto
     speed auto
    interface wlan-ap0
     description Service module interface to manage the embedded AP
     no ip address
     arp timeout 0
    interface Wlan-GigabitEthernet0
     description Internal switch interface connecting to the embedded AP
    interface Vlan1
     no ip address
    interface Vlan15
     ip address 192.168.15.1 255.255.255.224
    no ip forward-protocol nd
    ip route 0.0.0.0 0.0.0.0 FastEthernet4
    no ip http server
    ip http authentication local
    ip http secure-server

    The cable modem does not have any configuration. I cant add any to it. Its a cisco dpc3008. From vlan 10 i have no problem to get to the internet with the above  configuration. My problem is just vlan 15.

  • Cisco ASA 5505 L2TP VPN cannot access internal network

    Hi,
    I'm trying to configure Cisco L2TP VPN to my office. After successful connection I cannot access to internal network.
    Can you jhelp me to find out the issue?
    I have Cisco ASA:
    inside network - 192.168.1.0
    VPN network - 192.168.168.0
    I have router 192.168.1.2 and I cannot ping or get access to this router.
    Here is my config:
    ASA Version 8.4(3)
    interface Ethernet0/0
    switchport access vlan 2
    interface Ethernet0/1
    interface Ethernet0/2
    interface Ethernet0/3
    interface Ethernet0/4
    interface Ethernet0/5
    interface Ethernet0/6
    interface Ethernet0/7
    interface Vlan1
    nameif inside
    security-level 100
    ip address 192.168.1.1 255.255.255.0
    interface Vlan2
    nameif outside
    security-level 0
    ip address 198.X.X.A 255.255.255.248
    ftp mode passive
    same-security-traffic permit intra-interface
    object network net-all
    subnet 0.0.0.0 0.0.0.0
    object network vpn_local
    subnet 192.168.168.0 255.255.255.0
    object network inside_nw
    subnet 192.168.1.0 255.255.255.0
    access-list outside_access_in extended permit icmp any any echo-reply
    access-list outside_access_in extended deny ip any any log
    pager lines 24
    logging enable
    logging asdm informational
    mtu inside 1500
    mtu outside 1500
    ip local pool sales_addresses 192.168.168.1-192.168.168.254
    icmp unreachable rate-limit 1 burst-size 1
    no asdm history enable
    arp timeout 14400
    nat (inside,outside) source dynamic net-all interface
    nat (inside,outside) source static inside_nw inside_nw destination static vpn_local vpn_local
    nat (outside,inside) source static vpn_local vpn_local destination static inside_nw inside_nw route-lookup
    object network vpn_local
    nat (outside,outside) dynamic interface
    object network inside_nw
    nat (inside,outside) dynamic interface
    access-group outside_access_in in interface outside
    route outside 0.0.0.0 0.0.0.0 198.X.X.B 1
    timeout xlate 3:00:00
    timeout pat-xlate 0:00:30
    timeout conn 1:00:00 half-closed 0:10:00 udp 0:02:00 icmp 0:00:02
    timeout sunrpc 0:10:00 h323 0:05:00 h225 1:00:00 mgcp 0:05:00 mgcp-pat 0:05:00
    timeout sip 0:30:00 sip_media 0:02:00 sip-invite 0:03:00 sip-disconnect 0:02:00
    timeout sip-provisional-media 0:02:00 uauth 0:05:00 absolute
    timeout tcp-proxy-reassembly 0:01:00
    timeout floating-conn 0:00:00
    dynamic-access-policy-record DfltAccessPolicy
    user-identity default-domain LOCAL
    aaa authentication enable console LOCAL
    aaa authentication ssh console LOCAL
    aaa authentication http console LOCAL
    http server enable
    http 192.168.1.0 255.255.255.0 inside
    no snmp-server location
    no snmp-server contact
    snmp-server enable traps snmp authentication linkup linkdown coldstart warmstart
    crypto ipsec ikev1 transform-set my-transform-set-ikev1 esp-3des esp-sha-hmac
    crypto ipsec ikev1 transform-set my-transform-set-ikev1 mode transport
    crypto dynamic-map dyno 10 set ikev1 transform-set my-transform-set-ikev1
    crypto map vpn 20 ipsec-isakmp dynamic dyno
    crypto map vpn interface outside
    crypto isakmp nat-traversal 3600
    crypto ikev1 enable outside
    crypto ikev1 policy 10
    authentication pre-share
    encryption 3des
    hash sha
    group 2
    lifetime 86400
    telnet timeout 5
    ssh 192.168.1.0 255.255.255.0 inside
    ssh timeout 30
    console timeout 0
    management-access inside
    dhcpd address 192.168.1.5-192.168.1.132 inside
    dhcpd dns 75.75.75.75 76.76.76.76 interface inside
    dhcpd enable inside
    threat-detection basic-threat
    threat-detection statistics access-list
    no threat-detection statistics tcp-intercept
    webvpn
    group-policy sales_policy internal
    group-policy sales_policy attributes
    dns-server value 75.75.75.75 76.76.76.76
    vpn-tunnel-protocol l2tp-ipsec
    username ----------
    username ----------
    tunnel-group DefaultRAGroup general-attributes
    address-pool sales_addresses
    default-group-policy sales_policy
    tunnel-group DefaultRAGroup ipsec-attributes
    ikev1 pre-shared-key *****
    tunnel-group DefaultRAGroup ppp-attributes
    authentication ms-chap-v2
    class-map inspection_default
    match default-inspection-traffic
    policy-map type inspect dns preset_dns_map
    parameters
      message-length maximum client auto
      message-length maximum 512
    policy-map global_policy
    class inspection_default
      inspect dns preset_dns_map
      inspect ftp
      inspect h323 h225
      inspect h323 ras
      inspect rsh
      inspect rtsp
      inspect esmtp
      inspect sqlnet
      inspect skinny
      inspect sunrpc
      inspect xdmcp
      inspect sip
      inspect netbios
      inspect tftp
      inspect ip-options
    service-policy global_policy global
    prompt hostname context
    no call-home reporting anonymous
    Cryptochecksum:5d1fc9409c87ecdc1e06f06980de6c13
    : end
    Thanks for your help.

    You have to test it with "real" traffic to 192.168.1.2 and if you use ping, you have to add icmp-inspection:
    policy-map global_policy
      class inspection_default
        inspect icmp
    Don't stop after you've improved your network! Improve the world by lending money to the working poor:
    http://www.kiva.org/invitedby/karsteni

  • Cisco ASA 5505 and comodo SSL certificate

    Hey All,
    I am having an issue with setting up the SSL certificate piece of the Cisco AnyConnect VPN. I purchased the certificate and installed it via the ASDM under Configuration > Remote Access VPN > Certificate Management > Identity Certificates. I also placed the CA 2 piece under the CA Certificates. I have http redirect to https and under my browser it is green.
    Once the AnyConnect client installs and automatically connects i get no errors or anything. The minute I disconnect and try to reconnect again, I get the "Untrusted VPN Server Certificate!" which isn't true because the connection information is https://vpn.mydomain.com and the SSL Cert is setup as vpn.mydomain.com.
    On that note it lists the IP address instead of the vpn.mydomain.com as the untrusted piece of this. Now obviously I don't have the IP address as part of the SSL cert, just the web address. On the web side I have an A record setup to go from vpn.mydomain.com to the IP address of the Cisco ASA.
    What am I missing here? I can post config if anyone needs it.
    (My Version of ASA Software is 9.0 (2) and ASDM Version 7.1 (2))

    It's AnyConnect version 3.0. I don't know about the EKU piece. I didn't know that was required. I will attach my config.
    ASA Version 9.0(2)
    hostname MyDomain-firewall-1
    domain-name MyDomain.com
    enable password omitted
    xlate per-session deny tcp any4 any4
    xlate per-session deny tcp any4 any6
    xlate per-session deny tcp any6 any4
    xlate per-session deny tcp any6 any6
    xlate per-session deny udp any4 any4 eq domain
    xlate per-session deny udp any4 any6 eq domain
    xlate per-session deny udp any6 any4 eq domain
    xlate per-session deny udp any6 any6 eq domain
    passwd omitted
    names
    name 10.0.0.13.1 MyDomain-Inside description MyDomain Inside
    name 10.200.0.0 MyDomain_New_IP description MyDomain_New
    name 10.100.0.0 MyDomain-Old description Inside_Old
    name XXX.XXX.XX.XX Provider description Provider_Wireless
    name 10.0.13.2 Cisco_ASA_5505 description Cisco ASA 5505
    name 192.168.204.0 Outside_Wireless description Outside Wireless for Guests
    ip local pool MyDomain-Employee-Pool 192.168.208.1-192.168.208.254 mask 255.255.255.0
    ip local pool MyDomain-Vendor-Pool 192.168.209.1-192.168.209.254 mask 255.255.255.0
    interface Ethernet0/0
    switchport access vlan 2
    interface Ethernet0/1
    interface Ethernet0/2
    interface Ethernet0/3
    interface Ethernet0/4
    interface Ethernet0/5
    interface Ethernet0/6
    interface Ethernet0/7
    interface Vlan1
    nameif inside
    security-level 100
    ip address Cisco_ASA_5505 255.255.255.0
    interface Vlan2
    nameif outside
    security-level 0
    ip address Provider 255.255.255.252
    boot system disk0:/asa902-k8.bin
    ftp mode passive
    clock timezone PST -8
    clock summer-time PDT recurring
    dns domain-lookup inside
    dns server-group DefaultDNS
    name-server 10.0.3.21
    domain-name MyDomain.com
    object network obj_any
    subnet 0.0.0.0 0.0.0.0
    object network MyDomain-Employee
    subnet 192.168.208.0 255.255.255.0
    description MyDomain-Employee
    object-group network Inside-all
    description All Networks
    network-object MyDomain-Old 255.255.254.0
    network-object MyDomain_New_IP 255.255.192.0
    network-object host MyDomain-Inside
    access-list inside_access_in extended permit ip any4 any4
    access-list split-tunnel standard permit host 10.0.13.1
    pager lines 24
    logging enable
    logging buffered errors
    logging asdm informational
    mtu inside 1500
    mtu outside 1500
    no failover
    icmp unreachable rate-limit 1 burst-size 1
    asdm image disk0:/asdm-712.bin
    no asdm history enable
    arp timeout 14400
    no arp permit-nonconnected
    nat (inside,outside) source static Inside-all Inside-all destination static RVP-Employee RVP-Employee no-proxy-arp route-lookup
    object network obj_any
    nat (inside,outside) dynamic interface
    access-group inside_access_in in interface inside
    route outside 0.0.0.0 0.0.0.0 XXX.XXX.XX.XX 1
    route inside MyDomain-Old 255.255.254.0 MyDomain-Inside 1
    route inside MyDomain_New_IP 255.255.192.0 MyDomain-Inside 1
    route inside Outside_Wireless 255.255.255.0 MyDomain-Inside 1
    timeout xlate 3:00:00
    timeout pat-xlate 0:00:30
    timeout conn 1:00:00 half-closed 0:10:00 udp 0:02:00 icmp 0:00:02
    timeout sunrpc 0:10:00 h323 0:05:00 h225 1:00:00 mgcp 0:05:00 mgcp-pat 0:05:00
    timeout sip 0:30:00 sip_media 0:02:00 sip-invite 0:03:00 sip-disconnect 0:02:00
    timeout sip-provisional-media 0:02:00 uauth 0:05:00 absolute
    timeout tcp-proxy-reassembly 0:01:00
    timeout floating-conn 0:00:00
    dynamic-access-policy-record DfltAccessPolicy
    action terminate
    dynamic-access-policy-record "Network Access Policy Allow VPN"
    description "Must have the Network Access Policy Enabled to get VPN access"
    aaa-server LDAP_Group protocol ldap
    aaa-server LDAP_Group (inside) host 10.0.3.21
    ldap-base-dn ou=MyDomain,dc=MyDomainnet,dc=local
    ldap-group-base-dn ou=MyDomain,dc=MyDomainnet,dc=local
    ldap-scope subtree
    ldap-naming-attribute sAMAccountName
    ldap-login-password *****
    ldap-login-dn cn=Cisco VPN,ou=Special User Accounts,ou=MyDomain,dc=MyDomainNET,dc=local
    server-type microsoft
    user-identity default-domain LOCAL
    aaa authentication ssh console LOCAL
    http server enable
    http MyDomain_New_IP 255.255.192.0 inside
    http redirect outside 80
    no snmp-server location
    no snmp-server contact
    snmp-server enable traps snmp authentication linkup linkdown coldstart
    crypto ipsec ikev2 ipsec-proposal DES
    protocol esp encryption des
    protocol esp integrity sha-1 md5
    crypto ipsec ikev2 ipsec-proposal 3DES
    protocol esp encryption 3des
    protocol esp integrity sha-1 md5
    crypto ipsec ikev2 ipsec-proposal AES
    protocol esp encryption aes
    protocol esp integrity sha-1 md5
    crypto ipsec ikev2 ipsec-proposal AES192
    protocol esp encryption aes-192
    protocol esp integrity sha-1 md5
    crypto ipsec ikev2 ipsec-proposal AES256
    protocol esp encryption aes-256
    protocol esp integrity sha-1 md5
    crypto ipsec security-association pmtu-aging infinite
    crypto dynamic-map SYSTEM_DEFAULT_CRYPTO_MAP 65535 set ikev2 ipsec-proposal AES256 AES192 AES 3DES DES
    crypto map outside_map 65535 ipsec-isakmp dynamic SYSTEM_DEFAULT_CRYPTO_MAP
    crypto map outside_map interface outside
    crypto ca trustpoint LOCAL-CA-SERVER
    keypair LOCAL-CA-SERVER
    no validation-usage
    no accept-subordinates
    no id-cert-issuer
    crl configure
    crypto ca trustpoint VPN
    enrollment terminal
    fqdn vpn.mydomain.com
    subject-name CN=vpn.mydomain.com,OU=IT
    keypair vpn.mydomain.com
    crl configure
    crypto ca trustpoint ASDM_TrustPoint1
    enrollment terminal
    crl configure
    crypto ca trustpool policy
    crypto ca server
    shutdown
    crypto ca certificate chain LOCAL-CA-SERVER
    certificate ca 01
        omitted
      quit
    crypto ca certificate chain VPN
    certificate
        omitted
      quit
    crypto ca certificate chain ASDM_TrustPoint1
    certificate ca
        omitted
      quit
    crypto ikev2 policy 1
    encryption aes-256
    integrity sha
    group 5 2
    prf sha
    lifetime seconds 86400
    crypto ikev2 policy 10
    encryption aes-192
    integrity sha
    group 5 2
    prf sha
    lifetime seconds 86400
    crypto ikev2 policy 20
    encryption aes
    integrity sha
    group 5 2
    prf sha
    lifetime seconds 86400
    crypto ikev2 policy 30
    encryption 3des
    integrity sha
    group 5 2
    prf sha
    lifetime seconds 86400
    crypto ikev2 policy 40
    encryption des
    integrity sha
    group 5 2
    prf sha
    lifetime seconds 86400
    crypto ikev2 enable outside client-services port 443
    crypto ikev2 remote-access trustpoint VPN
    telnet timeout 5
    ssh MyDomain_New_IP 255.255.192.0 inside
    ssh timeout 5
    console timeout 0
    threat-detection basic-threat
    threat-detection statistics access-list
    threat-detection statistics tcp-intercept rate-interval 30 burst-rate 400 average-rate 200
    dynamic-filter updater-client enable
    dynamic-filter use-database
    dynamic-filter enable
    ssl encryption rc4-sha1 aes128-sha1 aes256-sha1 3des-sha1 rc4-md5 des-sha1
    ssl trust-point VPN outside
    webvpn
    enable outside
    anyconnect-essentials
    anyconnect image disk0:/anyconnect-macosx-i386-2.4.1012-k9.pkg 3
    anyconnect image disk0:/anyconnect-linux-2.4.1012-k9.pkg 4
    anyconnect image disk0:/anyconnect-win-3.1.01065-k9.pkg 5
    anyconnect profiles MyDomain-employee disk0:/MyDomain-employee.xml
    anyconnect enable
    tunnel-group-list enable
    group-policy DfltGrpPolicy attributes
    dns-server value 10.0.3.21
    vpn-tunnel-protocol ikev1 ikev2 l2tp-ipsec ssl-client
    default-domain value MyDomain.com
    group-policy MyDomain-Employee internal
    group-policy MyDomain-Employee attributes
    wins-server none
    dns-server value 10.0.3.21
    vpn-tunnel-protocol ssl-client
    split-tunnel-policy tunnelspecified
    split-tunnel-network-list value split-tunnel
    default-domain value MyDomain.com
    webvpn
      anyconnect profiles value MyDomain-employee type user
    username MyDomainadmin password omitted encrypted privilege 15
    tunnel-group MyDomain-Employee type remote-access
    tunnel-group MyDomain-Employee general-attributes
    address-pool MyDomain-Employee-Pool
    authentication-server-group LDAP_Group LOCAL
    default-group-policy MyDomain-Employee
    tunnel-group MyDomain-Employee webvpn-attributes
    group-alias MyDomain-Employee enable
    class-map inspection_default
    match default-inspection-traffic
    policy-map type inspect dns preset_dns_map
    parameters
      message-length maximum client auto
      message-length maximum 512
    policy-map global_policy
    class inspection_default
      inspect dns preset_dns_map
      inspect ftp
      inspect h323 h225
      inspect h323 ras
      inspect rsh
      inspect rtsp
      inspect esmtp
      inspect sqlnet
      inspect skinny 
      inspect sunrpc
      inspect xdmcp
      inspect sip 
      inspect netbios
      inspect tftp
      inspect ip-options
    service-policy global_policy global
    prompt hostname context
    no call-home reporting anonymous
    Cryptochecksum:1c7e3d7ff324e4fd7567aa21a96a8b22
    : end
    asdm image disk0:/asdm-712.bin
    asdm location MyDomain_New_IP 255.255.192.0 inside
    asdm location MyDomain-Inside 255.255.255.255 inside
    asdm location MyDomain-Old 255.255.254.0 inside
    no asdm history enable

  • Cisco ASA 5505 - 2 internal Networks

    Hi new to ASA's,
    Been trying to get the following setup working for ages but can't see what I am missing:
    (Got image from another post but exactly what I want but cannot get working)
    I can get ping between subnets but nothing else and Lan 2 cannot get to internet.
    The reolution for this guy was the following I believe; (from his config he has ASA v8.2)
    same-security-traffic permit intra-interface
    access-list NONAT permit ip 192.168.50.0 255.255.255.0 10.0.50.0255.255.255.0
    access-list NONAT permit ip 10.0.50.0 255.255.255.0 192.168.50.0 255.255.255.0
    nat (inside) 0 access-list NONAT
    I have tried this but I have ASA v8.4 and whilst commands 1 - 3 work command 4 doesn't.
    I get a message about the command being deprecated. I couldn't find a new version I could understand.
    Hope nothing stupid and simple but any help greatly appreciated.
    BTW, I have reset my ASA back to defaults except internet access is working and internet LAN as I made some many changes I feared one my conflict with the other.
    Many thanks for any views or help.

    Hi Jumora,
    Thanks for the reply.
    The 192 network behind the ASA can access the internet but the 10 network past the 1841 router can't.
    I have setup tcp bypass already as that got me at least remote access to the PC's on the 10 network from the 192 network.
    I had the 1841 router set to use the interface on the 192 subnet as the route to the 0.0.0.0 0.0.0.0 network but I couldn't get out but have just changed this to go to the inside interface of the ASA and can now ping 8.8.8.8 for example but still not internet access.
    Also I have found that the ASA seems to occasionally when it feels like it block pings from the 10 subnet to devices in the 192 subnet...... annoying for testing! but I can still access shares even though the ping fails.
    e.g. as per above yesterday it stopped when I enabled icmp error inspection but when I switched that off it worked again. Then suddenly again today with no changes it has stopped working again, drives me nuts the inconsistency!
    I couldn't find an attach option for the show tech so it has made this post massive.... apologies for that....
    ASA5505# show tech
    Cisco Adaptive Security Appliance Software Version 8.4(4)1
    Device Manager Version 6.4(9)
    Compiled on Thu 14-Jun-12 11:20 by builders
    System image file is "disk0:/asa844-1-k8.bin"
    Config file at boot was "startup-config"
    ASA5505 up 8 days 23 hours
    Hardware:   ASA5505, 512 MB RAM, CPU Geode 500 MHz
    Internal ATA Compact Flash, 128MB
    BIOS Flash M50FW016 @ 0xfff00000, 2048KB
    Encryption hardware device : Cisco ASA-5505 on-board accelerator (revision 0x0)
                                 Boot microcode   : CN1000-MC-BOOT-2.00
                                 SSL/IKE microcode: CNLite-MC-SSLm-PLUS-2.03
                                 IPSec microcode  : CNlite-MC-IPSECm-MAIN-2.06
                                 Number of accelerators: 1
    0: Int: Internal-Data0/0    : address is 4403.a7a2.e7c7, irq 11
    1: Ext: Ethernet0/0         : address is 4403.a7a2.e7bf, irq 255
    2: Ext: Ethernet0/1         : address is 4403.a7a2.e7c0, irq 255
    3: Ext: Ethernet0/2         : address is 4403.a7a2.e7c1, irq 255
    4: Ext: Ethernet0/3         : address is 4403.a7a2.e7c2, irq 255
    5: Ext: Ethernet0/4         : address is 4403.a7a2.e7c3, irq 255
    6: Ext: Ethernet0/5         : address is 4403.a7a2.e7c4, irq 255
    7: Ext: Ethernet0/6         : address is 4403.a7a2.e7c5, irq 255
    8: Ext: Ethernet0/7         : address is 4403.a7a2.e7c6, irq 255
    9: Int: Internal-Data0/1    : address is 0000.0003.0002, irq 255
    10: Int: Not used            : irq 255
    11: Int: Not used            : irq 255
    Licensed features for this platform:
    Maximum Physical Interfaces       : 8              perpetual
    VLANs                             : 3              DMZ Restricted
    Dual ISPs                         : Disabled       perpetual
    VLAN Trunk Ports                  : 0              perpetual
    Inside Hosts                      : 50             perpetual
    Failover                          : Disabled       perpetual
    VPN-DES                           : Enabled        perpetual
    VPN-3DES-AES                      : Enabled        perpetual
    AnyConnect Premium Peers          : 2              perpetual
    AnyConnect Essentials             : Disabled       perpetual
    Other VPN Peers                   : 10             perpetual
    Total VPN Peers                   : 12             perpetual
    Shared License                    : Disabled       perpetual
    AnyConnect for Mobile             : Disabled       perpetual
    AnyConnect for Cisco VPN Phone    : Disabled       perpetual
    Advanced Endpoint Assessment      : Disabled       perpetual
    UC Phone Proxy Sessions           : 2              perpetual
    Total UC Proxy Sessions           : 2              perpetual
    Botnet Traffic Filter             : Disabled       perpetual
    Intercompany Media Engine         : Disabled       perpetual
    This platform has a Base license.
    Serial Number: JMX3434343T
    Running Permanent Activation Key: 0x8509ef7f 0x2cff5895 0xa4675895 0x7989798 0xc1323132
    Configuration register is 0x1
    Configuration last modified by enable_15 at 16:21:28.863 UTC Wed Oct 23 2013
    ------------------ show disk0: controller ------------------
    Flash Model: SMART CF
    ------------------ show clock ------------------
    04:43:59.822 UTC Thu Oct 24 2013
    ------------------ show crashinfo ------------------
    No crash file found.
    ------------------ show module ------------------
    Mod Card Type                                    Model              Serial No.
      0 ASA 5505 Adaptive Security Appliance         ASA5505            JMX3434343T
    Mod MAC Address Range                 Hw Version   Fw Version   Sw Version    
      0 1255.a3a4.e3bf to 1233.a4a4.e4c4  0.1          1.0(12)13    8.4(4)1
    Mod SSC Application Name           Status           SSC Application Version
    Mod Status             Data Plane Status     Compatibility
      0 Up Sys             Not Applicable        
    ------------------ show memory ------------------
    Free memory:         283382600 bytes (53%)
    Used memory:         253488312 bytes (47%)
    Total memory:        536870912 bytes (100%)
    ------------------ show conn count ------------------
    76 in use, 704 most used
    ------------------ show xlate count ------------------
    80 in use, 814 most used
    ------------------ show vpn-sessiondb summary ------------------
    No sessions to display.
    ------------------ show blocks ------------------
      SIZE    MAX    LOW    CNT
         0    400    399    400
    4    100     99     99
        80    347    332    347
       256    200    192    195
      1550   6374   6306   6371
      2048   1200   1199   1200
      2560    264    264    264
      4096    100     99    100
      8192    100     99    100
    16384    100     99    100
    65536     16     15     16
    CORE  LIMIT  ALLOC   HIGH    CNT       FAILED
       0  24576     26     26     25            0
    ------------------ show blocks queue history detail ------------------
    History buffer memory usage: 2832 bytes (default)
    History analysis time limit: 100 msec
    Please see 'show blocks exhaustion snapshot' for more information
    ------------------ show interface ------------------
    Interface Ethernet0/0 "", is up, line protocol is up
      Hardware is 88E6095, BW 100 Mbps, DLY 100 usec
    Auto-Duplex(Full-duplex), Auto-Speed(100 Mbps)
    Input flow control is unsupported, output flow control is unsupported
    Available but not configured via nameif
    MAC address 4403.a7a2.e7bf, MTU not set
    IP address unassigned
    8257648 packets input, 9051289473 bytes, 0 no buffer
    Received 0 broadcasts, 0 runts, 0 giants
    0 input errors, 0 CRC, 0 frame, 0 overrun, 0 ignored, 0 abort
    0 pause input, 0 resume input
    0 L2 decode drops
    6222 switch ingress policy drops
    6399241 packets output, 1011134108 bytes, 0 underruns
    0 pause output, 0 resume output
    0 output errors, 0 collisions, 0 interface resets
    0 late collisions, 0 deferred
    0 rate limit drops
    0 switch egress policy drops
    0 input reset drops, 0 output reset drops
      Control Point Interface States:
    Interface number is 3
    Interface config status is active
    Interface state is active
    Interface Ethernet0/1 "", is up, line protocol is up
      Hardware is 88E6095, BW 100 Mbps, DLY 100 usec
    Auto-Duplex(Full-duplex), Auto-Speed(100 Mbps)
    Input flow control is unsupported, output flow control is unsupported
    Available but not configured via nameif
    MAC address 4403.a7a2.e7c0, MTU not set
    IP address unassigned
    1330699 packets input, 312264395 bytes, 0 no buffer
    Received 63097 broadcasts, 0 runts, 0 giants
    0 input errors, 0 CRC, 0 frame, 0 overrun, 0 ignored, 0 abort
    0 pause input, 0 resume input
    0 L2 decode drops
    0 switch ingress policy drops
    1738131 packets output, 637935280 bytes, 0 underruns
    0 pause output, 0 resume output
    0 output errors, 0 collisions, 0 interface resets
    0 late collisions, 0 deferred
    0 rate limit drops
    0 switch egress policy drops
    0 input reset drops, 0 output reset drops
      Control Point Interface States:
    Interface number is 4
    Interface config status is active
    Interface state is active
    Interface Ethernet0/2 "", is up, line protocol is up
      Hardware is 88E6095, BW 100 Mbps, DLY 100 usec
    Auto-Duplex(Full-duplex), Auto-Speed(100 Mbps)
    Input flow control is unsupported, output flow control is unsupported
    Available but not configured via nameif
    MAC address 4403.a7a2.e7c1, MTU not set
    IP address unassigned
    5028958 packets input, 693527818 bytes, 0 no buffer
    Received 28835 broadcasts, 0 runts, 0 giants
    0 input errors, 0 CRC, 0 frame, 0 overrun, 0 ignored, 0 abort
    0 pause input, 0 resume input
    0 L2 decode drops
    1 switch ingress policy drops
    7782140 packets output, 8316018900 bytes, 0 underruns
    0 pause output, 0 resume output
    0 output errors, 0 collisions, 0 interface resets
    0 late collisions, 0 deferred
    0 rate limit drops
    0 switch egress policy drops
    0 input reset drops, 0 output reset drops
      Control Point Interface States:
    Interface number is 5
    Interface config status is active
    Interface state is active
    Interface Ethernet0/3 "", is up, line protocol is up
      Hardware is 88E6095, BW 100 Mbps, DLY 100 usec
    Auto-Duplex(Full-duplex), Auto-Speed(100 Mbps)
    Input flow control is unsupported, output flow control is unsupported
    Available but not configured via nameif
    MAC address 4403.a7a2.e7c2, MTU not set
    IP address unassigned
    17048409 packets input, 21350059442 bytes, 0 no buffer
    Received 75081 broadcasts, 0 runts, 0 giants
    0 input errors, 0 CRC, 0 frame, 0 overrun, 0 ignored, 0 abort
    0 pause input, 0 resume input
    0 L2 decode drops
    18 switch ingress policy drops
    8319277 packets output, 5138543287 bytes, 0 underruns
    0 pause output, 0 resume output
    0 output errors, 0 collisions, 0 interface resets
    0 late collisions, 0 deferred
    0 rate limit drops
    0 switch egress policy drops
    0 input reset drops, 0 output reset drops
      Control Point Interface States:
    Interface number is 6
    Interface config status is active
    Interface state is active
    Interface Ethernet0/4 "", is down, line protocol is down
      Hardware is 88E6095, BW 100 Mbps, DLY 100 usec
    Auto-Duplex, Auto-Speed
    Input flow control is unsupported, output flow control is unsupported
    Available but not configured via nameif
    MAC address 4403.a7a2.e7c3, MTU not set
    IP address unassigned
    0 packets input, 0 bytes, 0 no buffer
    Received 0 broadcasts, 0 runts, 0 giants
    0 input errors, 0 CRC, 0 frame, 0 overrun, 0 ignored, 0 abort
    0 pause input, 0 resume input
    0 L2 decode drops
    0 switch ingress policy drops
    0 packets output, 0 bytes, 0 underruns
    0 pause output, 0 resume output
    0 output errors, 0 collisions, 0 interface resets
    0 late collisions, 0 deferred
    0 rate limit drops
    0 switch egress policy drops
    0 input reset drops, 0 output reset drops
      Control Point Interface States:
    Interface number is 7
    Interface config status is not active
    Interface state is active
    Interface Ethernet0/5 "", is down, line protocol is down
      Hardware is 88E6095, BW 100 Mbps, DLY 100 usec
    Auto-Duplex, Auto-Speed
    Input flow control is unsupported, output flow control is unsupported
    Available but not configured via nameif
    MAC address 4403.a7a2.e7c4, MTU not set
    IP address unassigned
    0 packets input, 0 bytes, 0 no buffer
    Received 0 broadcasts, 0 runts, 0 giants
    0 input errors, 0 CRC, 0 frame, 0 overrun, 0 ignored, 0 abort
    0 pause input, 0 resume input
    0 L2 decode drops
    0 switch ingress policy drops
    0 packets output, 0 bytes, 0 underruns
    0 pause output, 0 resume output
    0 output errors, 0 collisions, 0 interface resets
    0 late collisions, 0 deferred
    0 rate limit drops
    0 switch egress policy drops
    0 input reset drops, 0 output reset drops
      Control Point Interface States:
    Interface number is 8
    Interface config status is not active
    Interface state is active
    Interface Ethernet0/6 "", is down, line protocol is down
      Hardware is 88E6095, BW 100 Mbps, DLY 100 usec
    Auto-Duplex, Auto-Speed
    Input flow control is unsupported, output flow control is unsupported
    Available but not configured via nameif
    MAC address 4403.a7a2.e7c5, MTU not set
    IP address unassigned
    0 packets input, 0 bytes, 0 no buffer
    Received 0 broadcasts, 0 runts, 0 giants
    0 input errors, 0 CRC, 0 frame, 0 overrun, 0 ignored, 0 abort
    0 pause input, 0 resume input
    0 L2 decode drops
    0 switch ingress policy drops
    0 packets output, 0 bytes, 0 underruns
    0 pause output, 0 resume output
    0 output errors, 0 collisions, 0 interface resets
    0 late collisions, 0 deferred
    0 rate limit drops
    0 switch egress policy drops
    0 input reset drops, 0 output reset drops
      Control Point Interface States:
    Interface number is 9
    Interface config status is not active
    Interface state is active
    Interface Ethernet0/7 "", is up, line protocol is up
      Hardware is 88E6095, BW 100 Mbps, DLY 100 usec
    Auto-Duplex(Full-duplex), Auto-Speed(100 Mbps)
    Input flow control is unsupported, output flow control is unsupported
    Available but not configured via nameif
    MAC address 4403.a7a2.e7c6, MTU not set
    IP address unassigned
    7293552 packets input, 4521902362 bytes, 0 no buffer
    Received 6520 broadcasts, 0 runts, 0 giants
    0 input errors, 0 CRC, 0 frame, 0 overrun, 0 ignored, 0 abort
    0 pause input, 0 resume input
    0 L2 decode drops
    0 switch ingress policy drops
    16232858 packets output, 21234947011 bytes, 0 underruns
    0 pause output, 0 resume output
    0 output errors, 0 collisions, 0 interface resets
    0 late collisions, 0 deferred
    0 rate limit drops
    0 switch egress policy drops
    0 input reset drops, 0 output reset drops
      Control Point Interface States:
    Interface number is 10
    Interface config status is active
    Interface state is active
    Interface Internal-Data0/0 "", is up, line protocol is up
      Hardware is y88acs06, BW 1000 Mbps, DLY 10 usec
    (Full-duplex), (1000 Mbps)
    Input flow control is unsupported, output flow control is unsupported
    MAC address 4403.a2a2.e2c2, MTU not set
    IP address unassigned
    15222257 packets input, 10134321711 bytes, 0 no buffer
    Received 173531 broadcasts, 0 runts, 0 giants
    0 input errors, 0 CRC, 0 frame, 0 overrun, 0 ignored, 0 abort
    0 pause input, 0 resume input
    0 L2 decode drops, 0 demux drops
    15128507 packets output, 10256870512 bytes, 0 underruns
    0 pause output, 0 resume output
    0 output errors, 0 collisions, 0 interface resets
    0 late collisions, 0 deferred
    0 input reset drops, 0 output reset drops, 0 tx hangs
    input queue (blocks free curr/low): hardware (512/487)
    output queue (blocks free curr/low): hardware (512/450)
      Control Point Interface States:
    Interface number is 2
    Interface config status is active
    Interface state is active
    Interface Internal-Data0/1 "", is up, line protocol is up
      Hardware is 88E6095, BW 1000 Mbps, DLY 10 usec
    (Full-duplex), (1000 Mbps)
    Input flow control is unsupported, output flow control is unsupported
    MAC address 0000.0003.0002, MTU not set
    IP address unassigned
    15128465 packets input, 10256855882 bytes, 0 no buffer
    Received 1967 broadcasts, 0 runts, 0 giants
    0 input errors, 0 CRC, 0 frame, 0 overrun, 0 ignored, 0 abort
    0 pause input, 0 resume input
    0 switch ingress policy drops
    15222217 packets output, 10134318430 bytes, 0 underruns
    0 pause output, 0 resume output
    0 output errors, 0 collisions, 0 interface resets
    0 late collisions, 0 deferred
    0 switch egress policy drops
    0 input reset drops, 0 output reset drops
      Control Point Interface States:
    Interface number is 11
    Interface config status is active
    Interface state is active
    Interface Vlan1 "inside", is up, line protocol is up
      Hardware is EtherSVI, BW 100 Mbps, DLY 100 usec
    MAC address 4403.a7a2.e7c7, MTU 1500
    IP address 192.168.1.1, subnet mask 255.255.255.0
      Traffic Statistics for "inside":
    4183727 packets input, 523675346 bytes
    5702790 packets output, 5851485425 bytes
    142576 packets dropped
          1 minute input rate 22 pkts/sec,  2839 bytes/sec
          1 minute output rate 30 pkts/sec,  22751 bytes/sec
          1 minute drop rate, 0 pkts/sec
          5 minute input rate 33 pkts/sec,  3746 bytes/sec
          5 minute output rate 46 pkts/sec,  20906 bytes/sec
          5 minute drop rate, 1 pkts/sec
      Control Point Interface States:
    Interface number is 14
    Interface config status is active
    Interface state is active
    Interface Vlan2 "outside", is up, line protocol is up
      Hardware is EtherSVI, BW 100 Mbps, DLY 100 usec
    MAC address 4403.a7a2.e7c7, MTU 1492
    IP address 98.22.77.33, subnet mask 255.255.255.255
      Traffic Statistics for "outside":
    10541983 packets input, 11433817622 bytes
    3793777 packets output, 526586888 bytes
    13654 packets dropped
          1 minute input rate 47 pkts/sec,  41657 bytes/sec
          1 minute output rate 18 pkts/sec,  2802 bytes/sec
          1 minute drop rate, 0 pkts/sec
          5 minute input rate 80 pkts/sec,  38519 bytes/sec
          5 minute output rate 29 pkts/sec,  3749 bytes/sec
          5 minute drop rate, 0 pkts/sec
      Control Point Interface States:
    Interface number is 15
    Interface config status is active
    Interface state is active
    Interface Virtual0 "_internal_loopback", is up, line protocol is up
      Hardware is Virtual          MAC address 0000.0000.0000, MTU 1500
    IP address 127.0.0.1, subnet mask 255.255.255.0
      Traffic Statistics for "_internal_loopback":
    1 packets input, 28 bytes
    1 packets output, 28 bytes
    1 packets dropped
          1 minute input rate 0 pkts/sec,  0 bytes/sec
          1 minute output rate 0 pkts/sec,  0 bytes/sec
          1 minute drop rate, 0 pkts/sec
          5 minute input rate 0 pkts/sec,  0 bytes/sec
          5 minute output rate 0 pkts/sec,  0 bytes/sec
          5 minute drop rate, 0 pkts/sec
      Control Point Interface States:
    Interface number is 12
    Interface config status is active
    Interface state is active
    ------------------ show cpu usage ------------------
    CPU utilization for 5 seconds = 12%; 1 minute: 8%; 5 minutes: 8%
    ------------------ show cpu hogging process ------------------
    Process:      Unicorn Admin Handler, PROC_PC_TOTAL: 1, MAXHOG: 23, LASTHOG: 23
    LASTHOG At:   06:01:57 UTC Oct 15 2013
    PC:           0x0853e1f4 (suspend)
    Process:      Unicorn Admin Handler, NUMHOG: 1, MAXHOG: 23, LASTHOG: 23
    LASTHOG At:   06:01:57 UTC Oct 15 2013
    PC:           0x0853e1f4 (suspend)
    Call stack:   0x0853e1f4  0x0853ec36  0x0854182c  0x0869cc4b  0x08415ae7  0x0840ae40  0x0806e6cf
                  0x08aade2b  0x0806e6cf  0x084a0a44  0x0849986d  0x08499aac  0x08499dd6  0x084a0909
    Process:      Unicorn Admin Handler, PROC_PC_TOTAL: 2, MAXHOG: 18, LASTHOG: 18
    LASTHOG At:   06:01:57 UTC Oct 15 2013
    PC:           0x0853fb48 (suspend)
    Process:      Unicorn Admin Handler, NUMHOG: 2, MAXHOG: 18, LASTHOG: 18
    LASTHOG At:   06:01:57 UTC Oct 15 2013
    PC:           0x0853fb48 (suspend)
    Call stack:   0x0853fb48  0x0853fd1d  0x0853e1bc  0x0853ec36  0x0854182c  0x0869cc4b  0x08415ae7
          0x0840ae40  0x0806e6cf  0x08aade2b  0x0806e6cf  0x084a0a44  0x0849986d  0x08499aac
    Process:      Unicorn Admin Handler, PROC_PC_TOTAL: 2, MAXHOG: 24, LASTHOG: 24
    LASTHOG At:   06:01:57 UTC Oct 15 2013
    PC:           0x084167d2 (suspend)
    Process:      Unicorn Admin Handler, NUMHOG: 2, MAXHOG: 24, LASTHOG: 24
    LASTHOG At:   06:01:57 UTC Oct 15 2013
    PC:           0x084167d2 (suspend)
    Call stack:   0x08538afd  0x0853fa3a  0x0853fd1d  0x0853e1bc  0x0853ec36  0x0854182c  0x0869cc4b
                  0x08415ae7  0x0840ae40  0x0806e6cf  0x08aade2b  0x0806e6cf  0x084a0a44  0x0849986d
    Process:      Unicorn Admin Handler, PROC_PC_TOTAL: 1, MAXHOG: 12, LASTHOG: 12
    LASTHOG At:   06:01:57 UTC Oct 15 2013
    PC:           0x08ee9b4e (suspend)
    Process:      Unicorn Admin Handler, NUMHOG: 1, MAXHOG: 12, LASTHOG: 12
    LASTHOG At:   06:01:57 UTC Oct 15 2013
    PC:           0x08ee9b4e (suspend)
    Call stack:   0x08ee9e12  0x084a1032  0x0849986d  0x08499aac  0x08499dd6  0x084a0909  0x080689bc
    Process:      Dispatch Unit, PROC_PC_TOTAL: 2, MAXHOG: 12, LASTHOG: 12
    LASTHOG At:   06:01:57 UTC Oct 15 2013
    PC:           0x081e208a (suspend)
    Process:      Dispatch Unit, NUMHOG: 2, MAXHOG: 12, LASTHOG: 12
    LASTHOG At:   06:01:57 UTC Oct 15 2013
    PC:           0x081e208a (suspend)
    Call stack:   0x081e208a  0x080689bc
    Process:      Unicorn Admin Handler, NUMHOG: 1, MAXHOG: 180, LASTHOG: 180
    LASTHOG At:   07:24:33 UTC Oct 19 2013
    PC:           0x0806a8c2 (suspend)
    Call stack:   0x0806a8c2  0x08a8ebd7  0x08a8f7c8  0x08a914fa  0x080ddd6f  0x080df9db  0x080f4132
                  0x080f5b16  0x080dd956  0x080de0ef  0x080de876  0x080dea37  0xdd6e6c1c  0xdd6e71b5
    Process:      rtcli async executor process, NUMHOG: 14, MAXHOG: 94, LASTHOG: 82
    LASTHOG At:   07:28:06 UTC Oct 19 2013
    PC:           0x08f262e3 (suspend)
    Call stack:   0x0806a881  0x08f262e3  0x08f432a2  0x09064ba8  0x0903dfa9  0x0904f88d  0x0903ed70
                  0x09036221  0x0903d29b  0x0903d49f  0x09035ffa  0x09055321  0x0903dfa9  0x0904f88d
    Process:      rtcli async executor process, PROC_PC_TOTAL: 27, MAXHOG: 319, LASTHOG: 88
    LASTHOG At:   07:28:06 UTC Oct 19 2013
    PC:           0x08f4212d (suspend)
    Process:      rtcli async executor process, NUMHOG: 27, MAXHOG: 319, LASTHOG: 88
    LASTHOG At:   07:28:06 UTC Oct 19 2013
    PC:           0x08f4212d (suspend)
    Call stack:   0x08069faa  0x08f4212d  0x08f260b6  0x08f27b85  0x08f27c35  0xcb147b98
    Process:      rtcli async executor process, PROC_PC_TOTAL: 12, MAXHOG: 45, LASTHOG: 10
    LASTHOG At:   07:28:14 UTC Oct 19 2013
    PC:           0x08f2594b (suspend)
    Process:      rtcli async executor process, NUMHOG: 12, MAXHOG: 45, LASTHOG: 10
    LASTHOG At:   07:28:14 UTC Oct 19 2013
    PC:           0x08f2594b (suspend)
    Call stack:   0x0806a881  0x08f2594b  0x08f27b85  0x08f27c35  0xcb147b98
    Process:      Unicorn Admin Handler, NUMHOG: 4, MAXHOG: 11, LASTHOG: 11
    LASTHOG At:   07:28:14 UTC Oct 19 2013
    PC:           0x0806a8c2 (suspend)
    Call stack:   0x0806a8c2  0x08a8ebd7  0x08b9aa46  0x08b9ad0e  0x080dc76f  0xdd6e6961  0xdd6e71b5
                  0xdd6e7b07  0xdd6e8d5c  0xdd6e138d  0xdd6e247a  0x080dcb22  0x0849f899  0x084981c7
    Process:      rtcli async executor process, PROC_PC_TOTAL: 83, MAXHOG: 298, LASTHOG: 119
    LASTHOG At:   07:28:16 UTC Oct 19 2013
    PC:           0x08f262e3 (suspend)
    Process:      rtcli async executor process, NUMHOG: 47, MAXHOG: 298, LASTHOG: 119
    LASTHOG At:   07:28:16 UTC Oct 19 2013
    PC:           0x08f262e3 (suspend)
    Call stack:   0x0806a881  0x08f262e3  0x08f38fad  0x08f3acc0  0x0905a29e  0x0905b2ba  0x0903dfa9
                  0x0903ecb5  0x0904f6f5  0x0903ed70  0x09036221  0x0903d29b  0x0903d49f  0x09035ffa
    Process:      Unicorn Admin Handler, NUMHOG: 3, MAXHOG: 180, LASTHOG: 180
    LASTHOG At:   07:28:16 UTC Oct 19 2013
    PC:           0x0806a8c2 (suspend)
    Call stack:   0x0806a881  0x0806a8c2  0x0816261b  0x095302a7  0x0954abef  0x0954acc3  0x0815aabe
                  0x08134da6  0x08c64632  0x08ea8079  0x08ea8481  0x08ea85f7  0x08f41adc  0x0806e6cf
    Process:      Unicorn Admin Handler, NUMHOG: 3, MAXHOG: 15, LASTHOG: 15
    LASTHOG At:   07:28:20 UTC Oct 19 2013
    PC:           0x0806a8c2 (suspend)
    Call stack:   0x0806a881  0x0806a8c2  0x0947a399  0x0946d24d  0x0946d364  0x08c2b0e6  0x08c38f65
                  0x08ea810b  0x08ea8481  0x08ea85f7  0x08f41adc  0x0806e6cf  0x08f3cc48  0x092afca6
    Process:      Unicorn Admin Handler, NUMHOG: 3, MAXHOG: 64, LASTHOG: 64
    LASTHOG At:   07:28:20 UTC Oct 19 2013
    PC:           0x0806a8c2 (suspend)
    Call stack:   0x0806a881  0x0806a8c2  0x0947a3e4  0x09479cf9  0x094750eb  0x08c3f645  0x08c3fcab
                  0x08c2b235  0x08c38f65  0x08ea810b  0x08ea8481  0x08ea85f7  0x08f41adc  0x0806e6cf
    Process:      IP Thread, NUMHOG: 4, MAXHOG: 14, LASTHOG: 14
    LASTHOG At:   07:28:24 UTC Oct 19 2013
    PC:           0x0806a8c2 (suspend)
    Call stack:   0x0806a8c2  0x0947a399  0x0946d24d  0x0946d364  0x08c2b0e6  0x08c38f65  0x08ea810b
                  0x08ea8481  0x08ea85f7  0x08ea5f86  0x090e086e  0x090e0b6e  0x090b9a99  0x090b6b00
    Process:      Unicorn Admin Handler, PROC_PC_TOTAL: 22, MAXHOG: 180, LASTHOG: 64
    LASTHOG At:   07:28:24 UTC Oct 19 2013
    PC:           0x0806a8c2 (suspend)
    Process:      IP Thread, NUMHOG: 4, MAXHOG: 64, LASTHOG: 64
    LASTHOG At:   07:28:24 UTC Oct 19 2013
    PC:           0x0806a8c2 (suspend)
    Call stack:   0x0806a8c2  0x0947a3e4  0x09479cf9  0x094750eb  0x08c3f645  0x08c3fcab  0x08c2b235
                  0x08c38f65  0x08ea810b  0x08ea8481  0x08ea85f7  0x08ea5f86  0x090e086e  0x090e0b6e
    CPU hog threshold (msec): 10.240
    Last cleared: None
    ------------------ show process ------------------
        PC       SP       STATE       Runtime    SBASE     Stack Process
    Lwe 0x08058ba4 0xc82baf84 0x0a345788          0 0xc82b7078 15760/16384 block_diag
    Mrd 0x081e1e11 0xc82ed54c 0x0a346144     430188 0xc82cd6e0 120548/131072 Dispatch Unit
    Msi 0x087509a4 0xc82fdcb4 0x0a3458b0        713 0xc82f9da8 15688/16384 WebVPN KCD Process
    Msi 0x09200c7b 0xc839b3d4 0x0a3458b0       3466 0xc83974c8 15688/16384 y88acs06 OneSec Thread
    Mwe 0x080718dd 0xc83a3804 0x0a3458b0          0 0xc839f948 15808/16384 Reload Control Thread
    Mwe 0x080849b9 0xc83ae79c 0x0a346e2c          0 0xc83aabe0 15256/16384 aaa
    Mwe 0x08f4212d 0xc8d3d1e4 0x0a3458b0          9 0xc83aed78 15056/16384 UserFromCert Thread
    Mwe 0x08f4212d 0xc9003fe4 0x0a3458b0         14 0xc83b2f50 14528/16384 aaa_shim_thread
    Mwe 0x080b477c 0xc83bfa1c 0x0a347eb4          0 0xc83bbb20 15760/16384 CMGR Server Process
    Mwe 0x080b6ded 0xc83c3b64 0x0a3458b0          0 0xc83bfcb8 15832/16384 CMGR Timer Process
    Lwe 0x081e0474 0xc83d83bc 0x0a3568e0          0 0xc83d44b0 15488/16384 dbgtrace
    Mwe 0x084de0ed 0xc83ef574 0x0a3458b0          0 0xc83e76d8 31680/32768 idfw_proc
    Mwe 0x084ea35b 0xc83f75b4 0x0a3458b0          0 0xc83ef708 32216/32768 idfw_service
    Mwe 0x084f5fc5 0xc83fb70c 0x0a3458b0          0 0xc83f78a0 15524/16384 idfw_adagent
    Mwe 0x085351b5 0xc84038dc 0x0a3458b0         89 0xc83ffbd0 11568/16384 eswilp_svi_init
    Mwe 0x08f4212d 0xc8770564 0x0a3458b0          0 0xc8433aa0 15280/16384 netfs_thread_init
    Mwe 0x09576795 0xc844c10c 0x0a3458b0          0 0xc8448290 15848/16384 Chunk Manager
    Msi 0x08ae10be 0xc84508ac 0x0a3458b0       3523 0xc844c9c0 15656/16384 PIX Garbage Collector
    Mwe 0x08ac328a 0xc8461a0c 0x0a1d5d24          0 0xc845db00 16104/16384 IP Address Assign
    Mwe 0x08d0477a 0xc85f7534 0x0a251838          0 0xc85f3628 16104/16384 QoS Support Module
    Mwe 0x08b5c32a 0xc85fb70c 0x0a1d6c88          0 0xc85f7800 16104/16384 Client Update Task
    Lwe 0x095d54f5 0xc860009c 0x0a3458b0     109750 0xc85fc1f0 14448/16384 Checkheaps
    Mwe 0x08d093ed 0xc861080c 0x0a3458b0        454 0xc86089a0 19328/32768 Quack process
    Mwe 0x08d8569d 0xc86189c4 0x0a3458b0        533 0xc8610b38 31952/32768 Session Manager
    Mwe 0x08ed964d 0xc8620cd4 0xcadf5b08          8 0xc861ce68 15464/16384 uauth
    Mwe 0x08e66621 0xc8624f0c 0x0a264a10          0 0xc8621000 15632/16384 Uauth_Proxy
    Msp 0x08ea87de 0xc86313d4 0x0a3458b0        561 0xc862d4c8 15688/16384 SSL
    Mwe 0x08ed72d4 0xc863554c 0x0a26bc14          0 0xc8631660 15708/16384 SMTP
    Mwe 0x08ed170c 0xc86396a4 0x0a26af38      23255 0xc86357f8 13608/16384 Logger
    Mwe 0x08ecfd1d 0xc863d80c 0x0a3458b0          0 0xc8639990 15784/16384  Syslog Retry Thread
    Mwe 0x08ecadf5 0xc86419d4 0x0a3458b0          0 0xc863db28 15600/16384 Thread Logger
    Mwe 0x08ed50b4 0xc866457c 0x0a26b5e0          0 0xc8660680 15464/16384 syslogd
    Mwe 0x09132032 0xc8681094 0x0a2a5688          0 0xc867d1a8 15328/16384 vpnlb_thread
    Mwe 0x092037ec 0xc86916c4 0x0a2aa9e8          0 0xc868d808 16024/16384 pci_nt_bridge
    Mwe 0x082beb95 0xc8756e44 0x0a3458b0          0 0xc8752fb8 15864/16384 TLS Proxy Inspector
    Msi 0x08da221c 0xc87d44a4 0x0a3458b0       2749 0xc87d0598 15688/16384 emweb/cifs_timer
    Mwe 0x08852cc4 0xc88291f4 0x0a1c4c44          0 0xc88252f8 15712/16384 netfs_mount_handler
    Msi 0x086b4248 0xc8316454 0x0a3458b0      27304 0xc8312568 15312/16384 arp_timer
    Mwe 0x086bc58e 0xc8447fb4 0x0a371110          0 0xc84440f8 16024/16384 arp_forward_thread
    Mwe 0x08eddb77 0xc8f2e27c 0x0a26c680          0 0xc8f2a380 15672/16384 tcp_fast
    Mwe 0x08ee69a8 0xc8f3229c 0x0a26c680          0 0xc8f2e3b0 15656/16384 tcp_slow
    Mwe 0x08f1df34 0xc8f42fac 0x0a2745d0          0 0xc8f3f0b0 16000/16384 udp_timer
    Mwe 0x0814110d 0xc8fb133c 0xc83ca8d0          4 0xc8fad4a0 15664/16384 IPsec message handler
    Mwe 0x087515c6 0xc8fdc834 0x0a376060          1 0xc8fd8958 16056/16384 Lic TMR
    Mwe 0x087513bc 0xc8fe0884 0x0a1c0ea0        242 0xc8fdc988 16088/16384 Lic HA
    Msi 0x08153267 0xc84270dc 0x0a3458b0      54986 0xc8423440 13872/16384 CTM message handler
    Mwe 0x0811bd2d 0xc843bb8c 0x0a3458b0          0 0xc8437ce0 15832/16384 CTCP Timer process
    Mwe 0x090d3d95 0xc843fbac 0x0a3458b0          0 0xc843bd10 15816/16384 L2TP data daemon
    Mwe 0x090d6605 0xc9b5b24c 0x0a3458b0          0 0xc9b573b0 15816/16384 L2TP mgmt daemon
    Mwe 0x090c2b27 0xc9b9339c 0x0a29a3ec       2228 0xc9b8f4e0 15480/16384 ppp_timer_thread
    Msi 0x0913239d 0xc9b973ec 0x0a3458b0       4093 0xc9b93510 15640/16384 vpnlb_timer_thread
    Mwe 0x081c7708 0xc9c67c84 0x0a13ef88       2899 0xc9c47f18 118548/131072 tmatch compile thread
    Mwe 0x08d38b2d 0xcac940cc 0x0a3458b0          0 0xcac90210 15848/16384 ICMP event handler
    Mwe 0x0908081d 0xcac98254 0x0a3458b0          0 0xcac943a8 15832/16384 Dynamic Filter VC Housekeeper
    Mwe 0x08a1b612 0xcacc47f4 0x0a3458b0        819 0xcacc0938 13860/16384 IP Background
    Mwe 0x08c26e63 0xcaed904c 0x0a3458b0          0 0xcaed51a0 15832/16384 Crypto CA
    Mwe 0x08c60c18 0xcaedd1e4 0x0a3458b0          0 0xcaed9338 15896/16384 CERT API
    Mwe 0x08c257d5 0xcaee6e24 0x0a3458b0          0 0xcaee2f58 15928/16384 Crypto PKI RECV
    Mwe 0x0878dd85 0xc862d1cc 0x0a3458b0        187 0xc8629330 15272/16384 ESW_MRVL switch interrupt service
    Mwe 0x08cae62c 0xc866c89c 0x0a1ea7e0          0 0xc86689b0 15832/16384 lina_int
    Mrd 0x0959948b 0xc8684f1c 0x0a346144   28493079 0xc8681340 13824/16384 esw_stats
    Lsi 0x08af3199 0xc86958bc 0x0a3458b0        152 0xc86919a0 15704/16384 uauth_urlb clean
    Lwe 0x08acbd76 0xc83ff8b4 0x0a3458b0       4432 0xc83fba38 14308/16384 pm_timer_thread
    Mwe 0x08555f8d 0xc8418b0c 0x0a3458b0          0 0xc8414c60 15832/16384 IKE Common thread
    Mwe 0x0858cecd 0xcaf8688c 0x0a3458b0          0 0xcaf82a60 15704/16384 IKE Timekeeper
    Mwe 0x0857bad1 0xcaf8ccc4 0x0a1bc678          1 0xcaf890e8 12116/16384 IKE Daemon
    Mwe 0x08629eb3 0xcaf90c64 0x0a3458b0        964 0xcaf8d118 14744/16384 IKEv2 Daemon
    Mwe 0x08628e7c 0xcaf94ff4 0x0a3458b0       1095 0xcaf91148 15640/16384 IKEv2 DPD Client Process
    Mwe 0x08e7d2e4 0xcafafd7c 0x0a2690f4          0 0xcafabe90 16072/16384 RADIUS Proxy Event Daemon
    Mwe 0x08e41f35 0xcafb3d74 0xcb07e358          7 0xcafb0028 14912/16384 RADIUS Proxy Listener
    Mwe 0x08e7ca0d 0xcafb806c 0x0a3458b0          0 0xcafb41c0 15832/16384 RADIUS Proxy Time Keeper
    Mwe 0x086a1e44 0xcafbc184 0x0a3710c8          0 0xcafb8358 15264/16384 Integrity FW Task
    Mrd 0x082c923a 0xcaffce54 0x0a346144          0 0xcaff8f98 14552/16384 CP Threat-Detection Processing
    Mwe 0x081fb74e 0xcb0cc4bc 0x09c4a8bc       2497 0xcb0acd60 122448/131072 ci/console
    Msi 0x08b0ea8c 0xcb0d0e14 0x0a3458b0     217583 0xcb0ccef8 14004/16384 update_cpu_usage
    Mwe 0x08ef5ff5 0xcb0d4ecc 0x0a3458b0         77 0xcb0d1090 15360/16384 npshim_thread
    Msi 0x08b0eb14 0xcb0e1224 0x0a3458b0          0 0xcb0dd428 13104/16384 NIC status poll
    Mwe 0x08dd5f2c 0xcb0e54bc 0x0a259ec8        228 0xcb0e15c0 15540/16384 SNMP Notify Thread
    Mwe 0x086aba0e 0xcb12ebe4 0x0a37170c     235813 0xcb126d08 25428/32768 IP Thread
    Mwe 0x086b31fe 0xcb132d9c 0x0a371100       9150 0xcb12eea0 9700/16384 ARP Thread
    Mwe 0x084be3ae 0xcb136f8c 0x0a3716c8       1743 0xcb1331b0 12696/16384 icmp_thread
    Mwe 0x08f1f443 0xcb13b1e4 0x0a3458b0        158 0xcb137348 15728/16384 udp_thread
    Mwe 0x08ee0f44 0xcb13f0bc 0x0a37178c          0 0xcb13b4e0 15288/16384 tcp_thread
    Mwe 0x08f4212d 0xcb1bccd4 0x0a3458b0      12848 0xcb13fd70 26600/32768 rtcli async executor process
    Mwe 0x090e408d 0xcb4dff64 0x0a3458b0          0 0xcb4dc0a8 14608/16384 PPPOE background daemon
    Mwe 0x090e53c4 0xcb4e3fb4 0x0a29aa4c          1 0xcb4e00d8 14656/16384 PPPOE CLI daemon
    Mwe 0x0824ff45 0xcb501e4c 0x0a3458b0        258 0xcb4fdf90 15624/16384 Timekeeper
    Mwe 0x08e41f35 0xcb89a6d4 0xcb89eb10          7 0xcb896998 15392/16384 EAPoUDP-sock
    Mwe 0x0822323d 0xcb89e544 0x0a3458b0          0 0xcb89a9c8 15016/16384 EAPoUDP
    Mwe 0x08204371 0xcb3df9dc 0x0a3458b0        149 0xcb3dbb20 15168/16384 DHCPD Timer
    Mwe 0x082066a1 0xcb3e6404 0x0a3458b0       1286 0xcb3e25a8 7172/16384 dhcp_daemon
    Mwe 0x0910dfd4 0xcbc3b4e4 0x0a2a5380          0 0xcbc335e8 32472/32768 vpnfol_thread_msg
    Msi 0x09116252 0xcbc3fac4 0x0a3458b0       2657 0xcbc3bbd8 15656/16384 vpnfol_thread_timer
    Mwe 0x09114882 0xcbc44074 0x0a2a53c0          0 0xcbc401c8 16008/16384 vpnfol_thread_sync
    Msi 0x09115fdc 0xcbc486b4 0x0a3458b0      11061 0xcbc447b8 15672/16384 vpnfol_thread_unsent
    Mwe 0x0869e365 0xc8689384 0x0a3458b0          0 0xc86854d8 15832/16384 Integrity Fw Timer Thread
    Msi 0x08852fd6 0xc868d55c 0x0a3458b0        206 0xc8689670 15656/16384 netfs_vnode_reclaim
    Mwe 0x08f4212d 0xcb2a1914 0x0a3458b0       1277 0xcbd38510 15008/16384 Unicorn Proxy Thread
    Mwe 0x0825afcb 0xcbc61254 0x0a3458b0        335 0xcbc5d788 14272/16384 emweb/https
    Mwe 0x08eef828 0xcbd4dd0c 0xcbd4fd7c          0 0xcbd49fd0 14888/16384 listen/telnet
    Mwe 0x08aac530 0xcbdbd754 0xcbd6c9fc        102 0xcbd9def8 127432/131072 Unicorn Admin Handler
    Mwe 0x08aab345 0xcbddd644 0x0a3458b0        105 0xcbdbdf28 123712/131072 Unicorn Admin Handler
    Mwe 0x08cd7c6f 0xcaf358cc 0x0a49edc8          0 0xcaf31bb0 15384/16384 qos_metric_daemon
    Mwe 0x08218c82 0xcb2693fc 0x0a3458b0          3 0xcb265560 13248/16384 DHCP Client
    Mwe 0x08f1d929 0xcb4bb0fc 0xc8f3ece4          0 0xcb4b3300 31552/32768 DHCPC Receiver
    M*  0x08a86f55 0xdcc1df2c 0x0a346144        274 0xcb34deb8 19696/32768 telnet/ci
    -           -          -          -          0          -      -      DATAPATH-0-455
    -           -          -          -  744377118          -      -      scheduler
    -           -          -          -  774156778          -      -      total elapsed
    ------------------ show kernel process ------------------
    PID PPID PRI NI      VSIZE      RSS      WCHAN STAT  RUNTIME COMMAND
      1    0  20  0    2080768      616 3725686580    S      630 init
      2    0  15 -5          0        0 3725738556    S        0 kthreadd
      3    2  15 -5          0        0 3725692956    S        0 ksoftirqd/0
      4    2  15 -5          0        0 3725728656    S        0 events/0
      5    2  15 -5          0        0 3725728656    S        0 khelper
    50    2  15 -5          0        0 3725728656    S        0 kblockd/0
    53    2  15 -5          0        0 3726777703    S        0 kseriod
    99    2  20  0          0        0 3725848262    S        0 pdflush
    100    2  20  0          0        0 3725848262    S        0 pdflush
    101    2  15 -5          0        0 3725861131    S        0 kswapd0
    102    2  15 -5          0        0 3725728656    S        0 aio/0
    103    2  15 -5          0        0 3725728656    S        0 nfsiod
    214    2  15 -5          0        0 3725728656    S        0 hid_compat
    215    2  15 -5          0        0 3725728656    S        0 rpciod/0
    240    1  16 -4    1789952      600 3725997327    S        4 udevd
    272  240  18 -2    1785856      564 3725997327    S        0 udevd
    277  240  18 -2    1785856      552 3725997327    S        0 udevd
    421    1  20  0    5201920     1600 4294967295    S       11 lwsmd
    423  421  20  0   16736256     3600 4294967295    S      102 lwregd
    448    1  20  0    2084864      512 3725686580    S        1 sh
    449  448  20  0   10186752      528 4294967295    S        2 lina_monitor
    451  449   0 -20  440270848    53000 4294967295    S 77713055 lina
    ------------------ show kernel cgroup-controller detail ------------------
    memory controller:
    memory.limit_in_bytes: unlimited
    memory.usage_in_bytes: 61665280   (11%)
    memory.max_usage_in_bytes: 64245760   (12%)
    memory.failcnt: 0
    tasks:
    group "normal"
      memory.limit_in_bytes: unlimited
      memory.usage_in_bytes: 77824   (0%)
      memory.max_usage_in_bytes: 544768   (0%)
      memory.failcnt: 0
      tasks:
           PID         RSS COMMAND                      
             1      630784 init                         
             2           0 kthreadd                     
             3           0 ksoftirqd/0                  
             4           0 events/0                     
             5           0 khelper                      
            50           0 kblockd/0                    
            53           0 kseriod                      
            99           0 pdflush                      
           100           0 pdflush                      
           101           0 kswapd0                      
           102           0 aio/0                        
           103           0 nfsiod                       
           214           0 hid_compat                   
           215           0 rpciod/0                     
           240      614400 udevd                        
           272      577536 udevd                        
           277      565248 udevd                        
           448      524288 sh                           
    group "privileged"
      memory.limit_in_bytes: unlimited
      memory.usage_in_bytes: 22327296   (4%)
      memory.max_usage_in_bytes: 22515712   (4%)
      memory.failcnt: 0
      tasks:
           PID         RSS COMMAND                      
           449      540672 lina_monitor                 
           450           0 lina_monitor                 
           451    54280192 lina                         
           452           0 lina                         
           453           0 lina                         
           454           0 lina                         
           455           0 lina                         
    group "restricted"
      memory.limit_in_bytes: 23068672   (4%)
      memory.usage_in_bytes: 1724416   (0%)
      memory.max_usage_in_bytes: 1900544   (0%)
      memory.failcnt: 0
      tasks:
           PID         RSS COMMAND                      
           421     1638400 lwsmd                        
           422           0 lwsmd                        
           423     3686400 lwregd                       
           425           0 lwregd                       
           426           0 lwregd                       
           427           0 lwregd                       
           428           0 lwregd                       
           429           0 lwregd                       
           430           0 lwsmd                        
           431           0 lwsmd                        
           432           0 lwsmd                        
           433           0 lwsmd                        
           434           0 lwsmd                        
    cpu controller:
    cpu.shares: 1024
    cpuacct.usage: 777015353084076
    tasks:
    group "normal"
      cpu.shares: 1024
      cpuacct.usage: 53525955783   (0%)
    tasks:
           PID         RSS COMMAND                      
             1      630784 init                         
             2           0 kthreadd                     
             3           0 ksoftirqd/0                  
             4           0 events/0                     
             5           0 khelper                      
            50           0 kblockd/0                    
            53           0 kseriod                      
            99           0 pdflush                      
           100           0 pdflush                      
           101           0 kswapd0                      
           102           0 aio/0                        
           103           0 nfsiod                       
           214           0 hid_compat                   
           215           0 rpciod/0                     
           240      614400 udevd                        
           272      577536 udevd                        
           277      565248 udevd                        
           448      524288 sh                           
           449      540672 lina_monitor                 
           450           0 lina_monitor                 
           451    54280192 lina                         
           452           0 lina                         
           453           0 lina                         
           454           0 lina                         
    group "privileged"
      cpu.shares: 16384
      cpuacct.usage: 776952528547140   (100%)
      tasks:
           PID         RSS COMMAND                      
           455           0 lina                         
    group "restricted"
      cpu.shares: 1024
      cpuacct.usage: 1291957168   (0%)
      tasks:
           PID         RSS COMMAND                      
           421     1638400 lwsmd                        
           422           0 lwsmd                        
           423     3686400 lwregd                       
           425           0 lwregd                       
           426           0 lwregd                       
           427           0 lwregd                       
           428           0 lwregd                       
           429           0 lwregd                       
           430           0 lwsmd                        
           431           0 lwsmd                        
           432           0 lwsmd                        
           433           0 lwsmd                        
           434           0 lwsmd                        
    ------------------ show traffic ------------------
    inside:
    received (in 422169.300 secs):
    4183910 packets          523687951 bytes
    9 pkts/sec          1006 bytes/sec
    transmitted (in 422169.300 secs):
    5702974 packets          5851550584 bytes
    3 pkts/sec          13006 bytes/sec
          1 minute input rate 22 pkts/sec,  2839 bytes/sec
          1 minute output rate 30 pkts/sec,  22751 bytes/sec
          1 minute drop rate, 0 pkts/sec
          5 minute input rate 33 pkts/sec,  3746 bytes/sec
          5 minute output rate 46 pkts/sec,  20906 bytes/sec
          5 minute drop rate, 1 pkts/sec
    outside:
    received (in 422169.300 secs):
    10542135 packets          11433861540 bytes
    4 pkts/sec          27002 bytes/sec
    transmitted (in 422169.300 secs):
    3793870 packets          526596330 bytes
    8 pkts/sec          1003 bytes/sec
          1 minute input rate 47 pkts/sec,  41657 bytes/sec
          1 minute output rate 18 pkts/sec,  2802 bytes/sec
          1 minute drop rate, 0 pkts/sec
          5 minute input rate 80 pkts/sec,  38519 bytes/sec
          5 minute output rate 29 pkts/sec,  3749 bytes/sec
          5 minute drop rate, 0 pkts/sec
    _internal_loopback:
    received (in 422168.950 secs):
    0 packets          0 bytes
    0 pkts/sec          0 bytes/sec
    transmitted (in 422168.950 secs):
    0 packets          0 bytes
    0 pkts/sec          0 bytes/sec
          1 minute input rate 0 pkts/sec,  0 bytes/sec
          1 minute output rate 0 pkts/sec,  0 bytes/sec
          1 minute drop rate, 0 pkts/sec
          5 minute input rate 0 pkts/sec,  0 bytes/sec
          5 minute output rate 0 pkts/sec,  0 bytes/sec
          5 minute drop rate, 0 pkts/sec
    Aggregated Traffic on Physical Interface
    Ethernet0/0:
    received (in 776992.730 secs):
    8257731 packets          9051312645 bytes
    5 pkts/sec          11002 bytes/sec
    transmitted (in 776992.730 secs):
    6399342 packets          1011145708 bytes
    2 pkts/sec          1002 bytes/sec
          1 minute input rate 26 pkts/sec,  24481 bytes/sec
          1 minute output rate 20 pkts/sec,  3472 bytes/sec
          1 minute drop rate, 0 pkts/sec
          5 minute input rate 40 pkts/sec,  20147 bytes/sec
          5 minute output rate 29 pkts/sec,  4280 bytes/sec
          5 minute drop rate, 0 pkts/sec
    Ethernet0/1:
    received (in 776992.730 secs):
    1330771 packets          312271947 bytes
    1 pkts/sec          3 bytes/sec
    transmitted (in 776992.730 secs):
    1738316 packets          638003030 bytes
    2 pkts/sec          3 bytes/sec
          1 minute input rate 4 pkts/sec,  405 bytes/sec
          1 minute output rate 11 pkts/sec,  3333 bytes/sec
    <--- More --->
          1 minute drop rate, 0 pkts/sec
          5 minute input rate 7 pkts/sec,  735 bytes/sec
          5 minute output rate 13 pkts/sec,  4410 bytes/sec
          5 minute drop rate, 0 pkts/sec
    Ethernet0/2:
    received (in 776993.220 secs):
    5028958 packets          693527818 bytes
    0 pkts/sec          2 bytes/sec
    transmitted (in 776993.220 secs):
    7782202 packets          8316039741 bytes
    4 pkts/sec          10000 bytes/sec
          1 minute input rate 1 pkts/sec,  153 bytes/sec
          1 minute output rate 2 pkts/sec,  391 bytes/sec
          1 minute drop rate, 0 pkts/sec
          5 minute input rate 1 pkts/sec,  187 bytes/sec
          5 minute output rate 3 pkts/sec,  1011 bytes/sec
          5 minute drop rate, 0 pkts/sec
    Ethernet0/3:
    received (in 776993.220 secs):
    17219822 packets          21609826615 bytes
    0 pkts/sec          27005 bytes/sec
    transmitted (in 776993.220 secs):
    8373382 packets          5142266559 bytes
    5 pkts/sec          6004 bytes/sec
    <--- More --->
          1 minute input rate 8384 pkts/sec,  12695156 bytes/sec
          1 minute output rate 2657 pkts/sec,  203156 bytes/sec
          1 minute drop rate, 0 pkts/sec
          5 minute input rate 8010 pkts/sec,  12112337 bytes/sec
          5 minute output rate 2525 pkts/sec,  188122 bytes/sec
          5 minute drop rate, 0 pkts/sec
    Ethernet0/4:
    received (in 776993.680 secs):
    0 packets          0 bytes
    0 pkts/sec          0 bytes/sec
    transmitted (in 776993.680 secs):
    0 packets          0 bytes
    0 pkts/sec          0 bytes/sec
          1 minute input rate 0 pkts/sec,  0 bytes/sec
          1 minute output rate 0 pkts/sec,  0 bytes/sec
          1 minute drop rate, 0 pkts/sec
          5 minute input rate 0 pkts/sec,  0 bytes/sec
          5 minute output rate 0 pkts/sec,  0 bytes/sec
          5 minute drop rate, 0 pkts/sec
    Ethernet0/5:
    received (in 776993.690 secs):
    0 packets          0 bytes
    0 pkts/sec          0 bytes/sec
    transmitted (in 776993.690 secs):
    <--- More --->
    0 packets          0 bytes
    0 pkts/sec          0 bytes/sec
          1 minute input rate 0 pkts/sec,  0 bytes/sec
          1 minute output rate 0 pkts/sec,  0 bytes/sec
          1 minute drop rate, 0 pkts/sec
          5 minute input rate 0 pkts/sec,  0 bytes/sec
          5 minute output rate 0 pkts/sec,  0 bytes/sec
          5 minute drop rate, 0 pkts/sec
    Ethernet0/6:
    received (in 776994.140 secs):
    0 packets          0 bytes
    0 pkts/sec          0 bytes/sec
    transmitted (in 776994.140 secs):
    0 packets          0 bytes
    0 pkts/sec          0 bytes/sec
          1 minute input rate 0 pkts/sec,  0 bytes/sec
          1 minute output rate 0 pkts/sec,  0 bytes/sec
          1 minute drop rate, 0 pkts/sec
          5 minute input rate 0 pkts/sec,  0 bytes/sec
          5 minute output rate 0 pkts/sec,  0 bytes/sec
          5 minute drop rate, 0 pkts/sec
    Ethernet0/7:
    received (in 776994.140 secs):
    7328915 packets          4524298170 bytes
    <--- More --->
    3 pkts/sec          5004 bytes/sec
    transmitted (in 776994.140 secs):
    16345245 packets          21405489647 bytes
    4 pkts/sec          27001 bytes/sec
          1 minute input rate 2330 pkts/sec,  158045 bytes/sec
          1 minute output rate 7422 pkts/sec,  11264540 bytes/sec
          1 minute drop rate, 0 pkts/sec
          5 minute input rate 2481 pkts/sec,  168427 bytes/sec
          5 minute output rate 7977 pkts/sec,  12105867 bytes/sec
          5 minute drop rate, 0 pkts/sec
    Internal-Data0/0:
    received (in 776994.640 secs):
    15222548 packets          10134365294 bytes
    3 pkts/sec          13004 bytes/sec
    transmitted (in 776994.640 secs):
    15128813 packets          10256961010 bytes
    2 pkts/sec          13001 bytes/sec
          1 minute input rate 45 pkts/sec,  24860 bytes/sec
          1 minute output rate 49 pkts/sec,  26647 bytes/sec
          1 minute drop rate, 0 pkts/sec
          5 minute input rate 73 pkts/sec,  24918 bytes/sec
          5 minute output rate 75 pkts/sec,  26334 bytes/sec
          5 minute drop rate, 0 pkts/sec
    Internal-Data0/1:
    <--- More --->
    received (in 776994.640 secs):
    15128721 packets          10256943282 bytes
    2 pkts/sec          13001 bytes/sec
    transmitted (in 776994.640 secs):
    15222455 packets          10134357062 bytes
    3 pkts/sec          13004 bytes/sec
          1 minute input rate 48 pkts/sec,  26530 bytes/sec
          1 minute output rate 45 pkts/sec,  24826 bytes/sec
          1 minute drop rate, 0 pkts/sec
          5 minute input rate 75 pkts/sec,  26323 bytes/sec
          5 minute output rate 73 pkts/sec,  24908 bytes/sec
          5 minute drop rate, 0 pkts/sec
    ------------------ show perfmon ------------------
    PERFMON STATS:                     Current      Average
    Xlates                                0/s          0/s
    Connections                           0/s          0/s
    TCP Conns                             0/s          0/s
    UDP Conns                             0/s          0/s
    URL Access                            0/s          0/s
    URL Server Req       

Maybe you are looking for